Hematology & Oncology- UWORLD

¡Supera tus tareas y exámenes ahora con Quizwiz!

-Which curve best corresponds to lung cancer (learn don't answer) a) A b) B c) C d) D e) E

answer: A -Lung cancer has been the leading cause of cancer mortality in both women and men in the US since the 1980s -Tobacco use (primary and secondhand) is the most important cause of lung cancer, particularly non-small cell lung cancer; female use of cigarettes peaked in 1955, and mortality rates increased 20-50 years after smoking onset -Lung cancer mortality began to decrease after 2000, corresponding to a decline in tobacco use

52 years man -Progressively enlarging neck mass, fatigue, weight loss (past 2 months) -Exam: enlarged, firm, nontender cervical lymph nodes -Enlarged tonsils, bilateral axillary lymphadenopathy, and splenomegaly -Excisional lymph node biopsy= diffuse sheets of atypical, large B cells that have replaced normal tissue architecture -In situ hybridization= positive for EBV; risk factor most strongly associated with development of patient's condition? a)Advanced HIV infection b)Aspirin and nonsteroidal anti-inflammatory drug use c)Cigarette smoking d)Radiation exposure e)Socioeconomic status

answer: Advanced HIV infection -Likely has non-Hodgkin lymphoma (NHL); depending on the subtype, NHL may present with a rapidly progressive mass, lymphadenopathy, splenomegaly and B symptoms (e.g., night sweats, weight loss); diagnosis= excisional lymph node biopsy (demonstrate loss of normal tissue architecture with expansion of abnormal lymphocytes; most often B cells, as in this patient with large, atypical B cells) -Lymphoma is frequently associated with EBV, a ubiquitous herpesvirus that primarily infects B cells and causes persistent latent infections; although viral reactivation is uncommon, the latent EBV genome still transcribes viral gene products that can result in malignant transformation of infected cells; EBV is particularly associated with nasopharyngeal carcinoma, Hodgkin lymphoma, and some forms of NHL (e.g., Burkitt lymphoma) -Patients with HIV are at greatest risk for EBV-associated lymphomas (up to 60 fold greater); likely due to HIV-related immune dysregulation, which decreases recognition of EBV-infected cells and promotes B-cell proliferation -Some types of NHL (e.g., primary central nervous system lymphoma) are considered AIDS-defining conditions and can sometimes be presenting manifestation of HIV option B= decrease risk of colorectal cancer option C= strong risk factor for lung, bladder, pancreas cancer, etc.; but not linked to NHL option D= link between radiation and NHL is controversial; but not strong risk factor option E= may be associated with worse outcomes in patients with NHL; this marker has not been closely linked to development of NHL

20 years woman -Comes to ED due to bloody stools; hour ago= bowel movement that appeared grossly bloody -Past month= decreased energy -High pulse, patient appears tired; exam: mild tachycardia; abdomen is soft without organomegaly; skin exam: pallor and scattered bruises in various stages of healing throughout the trunk -Low Hb (7.2g/dl), normal MCV, low platelets (10,000/mm3), low WBCs (1,050/mm3); neutrophils (5%), lymphocytes (95%) -Cause? a)Autoimmune-induced loss of self-renewing hematopoietic stem cells b)Bone marrow replacement by constitutively active tyrosine kinase-stimulated cells c)Splenic hyperactivity with cell trapping by the reticuloendothelial system d)Increased destruction of cells opsonized by glycoprotein IIb/IIIa-targeted autoantibodies e)Ineffective hematopoiesis secondary to micronutrient deficiency

answer: Autoimmune-induced loss of self-renewing Patient has GI bleeding and ecchymosis secondary to thrombocytopenia; CBC shows pancytopenia, could be due to: -Bone marrow aplasia: hematopoietic stem cells in bone marrow are unable to proliferate and differentiate into mature blood cells due to aplastic anemia, nutritional impairment, infection (e.g,. Parvovirus, HIV), or cytotoxic medications -Bone marrow infiltration: cancer, fibrosis or infection (e.g., TB) fills bone marrow and crowds out hematologic cells (preventing blood cell replication and maturation) -Mature blood cell destruction: circulating mature blood cells may be destroyed due to DIC or TTP; extravascular mature blood cells may be destroyed or sequestered in the spleen due to hypersplenism Most likely cause is aplastic anemia (see why other options were ruled out); linked to certain medications, infections, toxins, or radiation, most cases are idiopathic (thought to be caused by autoimmune-induced loss of multipotent hematologic stem cells) In idiopathic AA, underlying insult (e.g., mutation, virus) causes alteration of surface antigens on multipotent stem cells, making them appear foreign and triggering a cytotoxic T cell response; Cytokines released by T-helper cells (type 1 cytokines) also contribute to pathogenesis, most notably interferon gamma, which triggers apoptotic cell death due to stimulation of destructive cytokine cascade and increased expression of Fas receptor on hematologic stem cell surface option B= CML causes leukocytosis, increase in myeloid cell line, so unlikely option C= hypersplenism can cause pancytopenia, but abdominal exam would show splenomegaly option D= immune thrombocytopenia, marked by thrombocytopenia only, no change in other cell lines option E= vitamin B12/folate deficiency causes pancytopenia, but MCV would be high

68 years man -Severe fatigue, weight loss (months); exam: mucosal pallor and no scleral icterus -Exam: mild hepatomegaly and markedly enlarged spleen; stool guaiac is negative; labs: pancytopenia, and peripheral smear= nucleated RBCs and teardrop cells -Bone marrow aspiration is attempted by yields no marrow, and bone marrow biopsy is performed; findings of biopsy? a)Diffusely fibrotic marrow with clusters of megakaryocytes b)Hypercellular marrow composed of clusters of plasma cells c)Increased staining for storage iron in marrow macrophages d)Marrow replaced by dysplastic cells of all 3 lineages e)Severely hypocellular marrow replaced by adipose cells

answer: Diffusely fibrotic marrow with clusters of megakaryocytes Pancytopenia, hepatosplenomegaly, teardrop cells on smear= primary myelofibrosis, a hematopoietic stem cell malignancy associated with clonal expansion of megakaryocytes Neoplastic megakaryocytes secrete the cytokine transforming growth factor-beta, which stimulates bone marrow fibroblasts to fill the medullary space with collagen. Subsequent bone marrow fibrosis usually leads to the following: -Extramedullary hematopoiesis- because bone marrow is destroyed by fibrotic tissue, hematopoiesis occurs in secondary hematopoietic tissue such as the spleen and liver. This typically results in marked splenomegaly and a palpable liver edge on examination -Cytopenias- extramedullary hematopoiesis is much less efficient than medullary hematopoiesis. Therefore, patients often have deficits of 1 or more cell lines (e.g., anemia, pancytopenia) -Dacrocytes on peripheral smear- RBC membrane is damaged when squeezing out of the fibrotic bone marrow or passing through enlarged spleen, which leads to formation of teardrop-shaped red cells The diagnosis is of primary myelofibrosis requires bone marrow examination. Bone marrow aspiration frequently results in a dry tap due to significant bone marrow fibrosis. Therefore, bone marrow biopsy is required; presence of fibrotic marrow with occasional clusters of atypical megakaryocytes confirms the diagnosis option B= multiple myeloma, a clonal disorder of plasma cells, is marked by a monoclonal spike (M-spike) on protein electrophoresis. Patients typically have anemia, hypercalcemia, bone pain (osteolytic lesions), and renal insufficiency. Bone marrow biopsy usually shows clusters of monoclonal plasma cells Option C= anemia of chronic disease is associated with hepcidin-induced changes in iron metabolism that cause iron trapping within macrophages. Bone marrow biopsy with Prussian blue stain demonstrates increased iron within macrophages. Hepatosplenomegaly and teardrop cells would be unexpected option D= myelodysplastic syndrome is a malignant stem cell cancer associated with production of dysplastic, atypical blood cells. Although most patients with myelodysplasia have cytopenias, bone marrow is usually hypercellular with multi-lineage dysplasia option E= aplastic anemia results from injury (e.g., drugs, radiation, viruses) to multipotent hematologic stem cells. It is usually associated with pancytopenia and a profoundly hypocellular bone marrow predominantly composed of adipose tissue. Extramedullary hematopoiesis is minimal and hepatosplenomegaly and dacrocytes are not typical

28 years man -Episodic fevers, night sweats, and weight loss (several months) -Emigrated from Kenya with his family at age of 14; works as driving instructor and volunteers at homeless shelter -Exam: normal except cervical lymphadenopathy -Lymph node biopsy; owl's eye apperance; diagnosis? a)Burkitt lymphoma b)Follicular lymphoma c)Hodgkin lymphoma d)Large B-cell lymphoma e)Multiple myeloma f)Tuberculosis

answer: Hodgkin lymphoma -Most likely has classic Hodgkin lymphoma; typical presentation= either nontender lymphadenopathy or lymphadenopathy incidentally detected on routine chest X-ray; most patients develop associated systemic B symptoms (fevers, night sweats, weight loss) -HL has bimodal age distribution with a peak in the 20s (or younger in some countries) and another in the 60s; CBC and PBS= normal -Lymph node biopsy= Reed-Sternberg cell on H&E; RS cells have ample cytoplasm, multilobulated nucleus or multiple nuclei, and inclusion-like nucleoli; RS cells are seen against a background of lymphocytes, histiocytes, and eosinophils in classic HL option A= presents as a mass in the abdomen/pelvis (or jaw in endemic/African form, which has peak incidence in boys around age 5); histology= medium-sized lymphoid cells with many tangible body macrophages, giving a "starry sky" appearance option B= aggregates of closely packed, neoplastic lymph node follicles; two major cell types, centrocytes (small cleaved cells) and centroblasts (large noncleaved cells) are observed; older adults are affected; waxing and waning painless lymphadenopathy (without B symptoms) is common option D= characterized by diffuse sheets of large, atypical lymphoid cells with nuclei at least 5 times the size of small lymphocytes option E= plasma cell dyscrasia characterized by proliferation of clonal plasma cells, is predominantly a disease of the elderly; characterized by osteolytic lesions, although spread to nodal and extranodal sites can occur option F= D/D for this patient, who is from Kenya, volunteers at homeless shelters, and has night sweats and weight loss; however, histology= caseating granulomas

23 years man -Comes to ED due to 4 days of cramping abdominal pain -Weakness (2 weeks), tried over counter antacids (without relief); industrial laborer with no significant medical history or known allergies -Parents have hypertension; exam: normal; peripheral smear= basophilic stippling -Cause? a)Acute intermittent porphyria b)Acute leukemia c)Acute gastritis d)Metal poisoning e)Non-Hodgkin lymphoma f)Poor nutrition

answer: Metal poisoning -Lead poisoning. In the US, lead poisoning (plumbism) is typically a pediatric condition that results from children ingesting lead-containing paint chips. However, lead poisoning can occur in adults -Affected individuals are usually miners or industrial workers (especially those in battery manufacturing) who inhale particulate lead while working. Adults with lead poisoning present with weakness, abdominal pain and constipation. In severe cases, there may be neurologic manifestations (e.g., headache, cognitive symptoms, peripheral neuropathy). On exam: patients may have blue "lead lines" at the junction of teeth and gingivae -Classic diagnostic finding= coarse basophilic stippling on a background of hypochromic microcytic anemia (PBS); basophilic stippling results from abnormal degradation of ribosomal RNA (due to lead-induced inhibition of a nucleotidase) -The hypochromic microcytic anemia results from inhibition of delta-aminolevulinate dehydratase and resultant reduced incorporation of iron into heme; the net effect of these is decreased Hb synthesis option A= can cause attacks of abdominal pain without tenderness due to autonomic neuropathy; erythropoiesis is not affected, and PBS is normal option B= Auer rods in AML option C= autoimmune destruction of gastric parietal cells and/or intrinsic factor= pernicious anemia option E= can invade bone marrow, causing myelophthisic anemia; PBS would show leukoerythrocytosis= immature granulocytes and nucleated teardrop shaped RBCs option F= folate/vitamin B12 deficiency= megaloblastic anemia with hypersegmented neutrophils

20 years man -Brought to ED due to fever, headache, neck pain (last day); fever, high pulse, and RR -Neck stiffness and petechial rash on trunk; CSF analysis reveals: low glucose, high protein, high leukocytes (neutrophils= 70%) -CSF gram stain= gram negative diplococci; in ED, the patient's hemodynamic status deteriorates rapidly; BP drops to 80/50mmHg, venous access sites are oozing blood. Findings on peripheral smear? a)Atypical lymphocytes b)Howell-Jolly bodies c)Hypersegmented neutrophils d)Reactive thrombocytosis e)Schistocytes f)Severe pancytopenia

answer: Schistocytes -Clinical findings (fever, headache, neck stiffness, rash) and CSF findings (elevated protein, low glucose, leukocytosis, gram-negative diplococci)= meningococcal meningitis; he subsequently developed septic shock (severe hypotension) while in the ED -His bleeding from venous puncture sites is highly suggestive of DIC, a consumptive coagulopathy associated with sepsis, malignancy, trauma, and obstetric complications -In DIC due to gram-negative sepsis, the coagulation cascade is activated by bacterial endotoxins, leading to widespread fibrin deposition and consumption of coagulation factors and platelets, with eventual bleeding. Deposition of fibrin strands in small vessels can cause shearing of circulating RBCs, resulting in schistocytes on peripheral smear. Labs: show decreased platelet count and fibrinogen level and prolonged PT and PTT option A= reactive lymphocytes with abundant, pale-blue cytoplasm and large nuclei; often seen in viral infections, particular in infectious mononucleosis option B= are round, dark, purple/red inclusions within RBCs; these represent nuclear fragments that are typically removed by the spleen; they can be seen in patients with splenectomy or reduced splenic function (e.g., sickle cell) option C= megaloblastic anemia option D= common in infection due to cytokine-mediated megakaryocyte proliferation and release of platelets in peripheral bloodstream. In contrast, patients with DIC have thrombocytopenia due to platelet consumption option F= often seen with aplastic anemia= hypocellular marrow due to destruction of stem cells

45 years man -3 months of exertional fatigue, weight loss; sexually active with multiple male partners. -Temp (37.9); patient appears chronically ill and has mucosal pallor -Thick, white plaques over the buccal mucosa and soft palate; anterior and posterior cervical lymphadenopathy is present; no organomegaly -Labs: Hematocrit (28%), MCV (76), platelets (160,000), leukocytes (4,100), iron (low), total iron binding capacity (low); cause of anemia? a)Abnormal iron utilization b)Decreased synthesis of globin chains c)Deposition of fibrin in the microcirculation d)Gastrointestinal blood loss e)Deficiency of a heme synthesis enzyme

answer: Abnormal iron utilization -patient's high risk sexual history, weight loss, lymphadenopathy, chronically ill appearance, oral candidiasis suggest undiagnosed HIV infection. Patients with chronic infections such as HIV often develop anemia of chronic disease (ACD) due to high circulating levels of inflammatory cytokines, which cause maladaptive changes to iron utilization -Although ACD is mediated by a wide range of inflammatory cytokines (e.g., IL-1, IL-6, TNF-alpha, IFN-gamma), the major player is hepcidin, a small peptide released by the liver in response to inflammation. Hepcidin binds to and inhibits iron channels on enterocytes and reticuloendothelial macrophages, leading to reduced iron absorption in the gut and reduced iron recycling in the reticuloendothelial system (site of senescent RBC destruction). This significantly reduces circulating serum iron, which limits iron availability for erythropoiesis -Patients with ACD typically develop normocytic or slightly microcytic anemia with a low reticulocyte response. Total iron binding capacity is generally reduced due to cytokine mediated suppression of transferrin option B= thalassemia option C= DIC= microangiopathic hemolytic anemia and severe thrombocytopenia option D= iron deficiency anemia; would have high total iron binding capacity option E= porphyrias

15 years patient -age: 15 years patient -Progressive dyspnea; Chest X-ray= large mediastinal mass and bilateral pleural effusions -Elevated WBC count, PBS reveals abnormal WBCs (blast cells with large, irregular nuclei and granular chromatin) -Cells are positive for terminal deoxynucleotidyl transferase and CD3 and negative for CD19; most likely diagnosis? a)Acute lymphoblastic leukemia b)Acute myeloid leukemia c)Burkitt lymphoma d)Hodgkin lymphoma e)Mantle cell lymphoma

answer: Acute lymphoblastic leukemia -Patient's PBS demonstrates blast cells with large, irregular nuclei and granular chromatin (blast cells could be myeloid or lymphoid) -Presence of terminal deoxynucleotidyl transferase (TdT), a DNA polymerase involved in V(D)J recombination that is expressed only by lymphoid progenitor cells -Negative for CD19= not B cells; positive for CD3= T cells -Thus, most likely acute lymphoblastic leukemia; unlike B-cell ALL (occurs in early childhood), T-cell ALL is most common in the teenage years or early adulthood -Tumors classically develop in the thymus, organ charged w/ T-cell development, and often result in formation of mediastinal mass= compresses airways and nearby vessels= cough & dyspnea; SVC syndrome; pleural effusions are also common due to inflammation from tumor growth/cytokine release option B= Auer rods are specific for it option C= aggressive B-cell, non-Hodgkin lymphoma; strongly associated with EBV; jaw/facial bone and abdominal involvement are common, mediastinal mass would be atypical, express B cell markers, and not associated with circulating blasts option D= arises from germinal or post-germinal center B cells; most patients= painless, peripheral lymphadenopathy, particularly in the cervical region; Reed-Sternberg cells (large cells with multiple or multilobulated nuclei and eosinophilic nucleoli) are characteristic option E= mature B-cell, non-Hodgkin lymphoma, usually presents with lymphadenopathy, splenomegaly, cytopenia; would express B cell markers, and peripheral blasts would be atypical

41 years woman -Weakness, easy fatigability; several episodes of epistaxis and gum bleeding -Exam: mucosal pallor; evaluation: clonal proliferation of WBCs containing an abnormal protein -Experiment: abnormal cells from patient are purified and cultured in 2 different plates, one with vitamin A derivative (plate 1) and other with control plate (plate 2) -After several days of incubation, cells in plate 1 are well differentiated compared to those in plate 2, and clonal proliferation is inhibited; diagnosis? a)Acute lymphocytic leukemia b)Acute myeloid leukemia c)Chronic lymphocytic leukemia d)Chronic myeloid leukemia e)High-grade non-Hodgkin lymphoma

answer: Acute myeloid leukemia -Acute promyelocytic leukemia (APL), type M3 of acute myelogenous leukemia in the FAB classification system, results from a t(15;17) mutation whereby the gene for retinoid acid receptor alpha (RAR-alpha) is transferred form chromosome 17 to chromosome 15, where it fuses with the promyelocytic leukemia (PML) gene -This leads to formation of a new gene called PML/RAR-alpha; normally, RAR-alpha plays a role in the proper differentiation of myeloid precursors, as the receptor interacts with retinoid acid to affect transcription of genes required for maturation -However, the t(15;17) mutation in APML causes transcription repression and produces an abnormal receptor that is unable to signal for differentiation of myeloid precursors at physiologic doses of retinoic acid -Treatment with all-trans retinoid acid (ATRA), vitamin A derivative, overcomes this (partly by inducing PML/RAR-alpha proteolysis) and stimulates differentiation of myeloblasts into mature granulocytes; ATRA therapy induces remission in 90% of patients with APL; APL is associated with DIC option A= affects children mainly; blast cells are positive for CD10 antigen, which indicates good prognosis option C= seen in elderly; presents with lymphadenopathy, hepatosplenomegaly, and anemia, has indolent course; neoplastic cells resemble mature B lymphocytes option D= manifests in elderly with fatigue, leukocytosis, splenomegaly; myeloid precursors in different stages of differentiation proliferate. Caused by t(9;22); BCR-ABL fusion gene, codes for abnormal tyrosine kinase option E= t(8;14), t(2;8), t(8;22) are typical for Burkitt lymphoma; these involve c-myc oncogene; it is associated with EBV infection and classically has a "starry sky" histologic appearance

63 years man -Fatigue, easy bruising, no lymphadenopathy; labs: Hb (8.0g/dl), platelets (40,000/mm3), leukocytes (20,500/mm3) -Peripheral smear: cells with Auer rods; diagnosis? a)Acute lymphoblastic leukemia b)Acute myeloid leukemia c)Chronic lymphocytic leukemia d)Chronic myeloid leukemia e)Hairy cell leukemia f)Hodgkin lymphoma g)Infectious mononucleosis

answer: Acute myeloid leukemia -Peripheral smear= very large nucleated cells with scant cytoplasm; these are blast cells= makes acute leukemia likely. Closer examination: linear, purple-red inclusions in some of cells, called Auer rods -These represent fused granules and may be single or multiple within immature myeloid precursors. Auer rods are highly suggestive of acute myeloid leukemia (AML), but not ALL (choice A); Although Auer rods are most commonly associated with M1, M2, and M3 subtypes, they me be found in any type of AML -Majority of AML occurs in adults, with median age of 65, and median WBC count of about 15,000/mm3 at diagnosis. Most patients present with complications of pancytopenia (anemia, thrombocytopenia, and functional neutropenia); diagnostic criteria generally requires presence of >20% myeloblasts in bone marrow or peripheral blood option C and D= chronic leukemia causes prevalence of mature cells in peripheral blood. In CML peripheral smear shows many mature granulocytes and few blasts (generally <2%); in CLL, smear shows many mature lymphocytes option E= have splenomegaly, cytopenias, and circulating hairy cells. On peripheral smear, hairy cells take appearance of small to medium sized lymphoid cells with circumferential hairy projections option F= present with mass (e.g., mediastinal mass, enlarged lymph nodes). Histologically, Reed-Sternberg cells are characteristic; these are large cells with abundant basophilic cytoplasm and >2 nuclear lobes or nuclei option G= have fatigue, lymphadenopathy, and splenomegaly. Thrombocytopenia can be a complication of this infection. Peripheral smear would show atypical lymphocytes

34 years man -Brought to ED after motor vehicle collision. He has blunt abdominal trauma and bilateral femur fractures. -Drinks 8-12 alcoholic beverages a day. Low BP and high pulse. Receives transfusion of several units of packed RBCs -During transport to ICU, the patient reports tingling sensation in his fingers and toes. Serum ionized calcium level is low. Cause? a)antibody-mediated red blood cell membrane damage b)Calcium chelation by a substance in the transfused blood c)Electrolyte leakage from red blood cells during pretransfusion storage d)Increased renal tubular excretion of calcium e)Release of intracellular contents from injured muscle

answer: Calcium chelation by a substance in the transfused blood -The patient has symptomatic hypocalcemia (e.g., paresthesias) following high-volume blood transfusion. Prior to storage, donated blood is generally mixed with solutions containing citrate anticoagulant. Following transfusion citrate is rapidly metabolized to bicarbonate in the liver; however, when large volumes of blood are transfused rapidly, the excess citrate can chelate calcium in the plasma, leading to hypocalcemia due to decreased serum ionized calcium concentration. This is most common with very high transfusion rates (e.g., >9 units/hour), but it can also be seen at lower rates in patients with underlying hepatic insufficiency (e.g., alcohol associated liver disease) -Other complications of high-volume transfusion include depletion of coagulation factors and volume overload option A and C= antibody-mediated hemolysis is typically caused by transfusion of ABO-incompatible blood. Characteristic clinical findings include fever, back or flank pain, acute kidney injury, and hemoglobinuria. The primary electrolyte complication is hyperkalemia due to release of intracellular potassium. Hyperkalemia can also be seen due to leakage of potassium from RBCs during storage. However, hypocalcemia wouldn't occur option D= acute hypocalcemia due to increased renal calcium excretion can be seen in hypoparathyroidism. This is commonly encountered as a complication of thyroid or other neck surgery with inadvertent injury or removal of the parathyroid glands. Abdominal and lower extremity trauma wouldn't affect parathyroid function option E= rhabdomyolysis typically presents with muscle pain, red urine (i.e., myoglobinuria), and elevated creatinine kinase. Release of intracellular phosphate from muscle cells can precipitate calcium salts in damaged muscle and lead to hypocalcemia, but this is typically a delayed finding: 24-72 hours after trauma

12 years girl -Brought to ED due to several days of worsening exertional dyspnea and lethargy -Patient has a history of sickle cell disease and takes hydroxyurea -Exam: mucosal pallor and a systolic ejection murmur -Labs: Hb (5.6g/dl); marrow biopsy demonstrates decreased erythroid precursors and giant pronormoblasts containing inclusions as shown -Cause of patient's current condition? (learn don't answer) a)Acute viral infection b)Dietary folate deficiency c)Medication adverse effect d)Secondary iron overload e)Toxic chemical exposure

answer: Acute viral infection -Symptomatic anemia (lethargy, exertional dyspnea, mucosal pallor, ejection murmur) and bone marrow biopsy showing decreased erythropoiesis (reduced erythrocyte precursors) raise suspicion for transient aplastic crisis due to acute parvovirus B19 infection -Parvovirus is a small, nonenveloped, DNA virus transmitted via the respiratory route that primarily attacks erythroid progenitor cells due to tropism for an erythrocyte cell surface receptor (P blood group antigen). Infection of erythroid progenitor cells prevents RBC maturation, leading to formation of abnormal giant pronormoblasts (several times larger than surrounding RBCs) with glassy, intranuclear viral inclusions. -The drop in RBC production often leads to a transient (1-2 weeks) drop in hematocrit. Although many patients are asymptomatic, individuals with underlying hemoglobin disorders (e.g., sickle cell anemia) sometimes develop symptomatic anemia option B= can be seen in patients with sickle cell disease due to increased erythropoiesis in response to chronic anemia; however, folate deficiency is characterized by macrocytic anemia and hypersegmented neutrophils option C= can be seen in patients with sickle cell disease due to increased erythropoiesis in response to chronic anemia; however, folate deficiency is characterized by macrocytic anemia and hypersegmented neutrophils option D= patients who require multiple transfusions, but biopsy would show increased iron stores option E= exposure to toxic chemicals, e.g., industrial solvents/chemicals, insecticides, can lead to aplastic anemia. Bone marrow biopsy would show hypocellular marrow with a decrease in all cell types, the remaining hematopoietic cells would be morphologically normal

59 years man -3 months of progressive fatigue, back pain; back pain mainly occurs with movement or positional changes -Exam: midline tenderness over middle and lower back -Labs: Hb (10.2g/dl) and hypercalcemia; bone marrow biopsy= numerous plasma cells (abundant basophilic cytoplasm, eccentric nuclei, well-developed Golgi apparatus (perinuclear paleness) and "clock-face" peripheral chromatin) -Greatest risk for developing? a)Amyloidosis b)Cardiac tamponade c)Hepatic failure d)Hyperthyroidism e)Meningeal carcinomatosis f)Splenic rupture

answer: Amyloidosis Bone marrow aspirate= numerous plasma cells (abundant basophilic cytoplasm, eccentric nuclei, well-developed Golgi apparatus (perinuclear paleness) and "clock-face" peripheral chromatin) Bone marrow sample with >10% plasma cells is strongly suggestive of multiple myeloma (MM), clonal plasma cell malignancy; In MM, neoplastic plasma cells: -Replicate in bone marrow and choke out normal hematopoiesis, leading to normocytic, normochromic anemia and increased risk of infection -Secrete osteolytic cytokines= bone pain, osteolytic (radiolucent) bone lesions, and hypercalcemia -Produce large quantities of monoclonal immunoglobulin (paraprotein) composed of heavy chain and light chains (e.g., IgG, IgA) or light chains alone. Light chains can deposit in renal tubules, leading to light-chain cast nephropathy, which is characterized by mild renal insufficiency, and waxy, laminated urinary casts (Bence Jones) -Light chains can also form insoluble fibrils and deposit in major organs, leading to amyloid light-chain amyloidosis. This can contribute to the already elevated risk of renal failure, as well as heart failure and neurologic dysfunction; Amyloidosis on biopsy 1) eosinophilic extracellular deposits (H&E) or 2) apple-green birefringence (Congo red under polarized light)

55 years man -Several months of progressive fatigue. History: hypertension, obesity; high BMI; exam: mucosal pallor -Labs: Hb (8.6g/dl), MCV (84), reticulocytes (0.2%), platelets (200,000), WBCs (7,500), BUN and creatinine (normal); cause of anemia? a)Anemia of chronic inflammation b)Extravascular hemolysis c)Microangiopathic hemolytic anemia d)Occult blood loss e)Vitamin B12 deficiency

answer: Anemia of chronic inflammation -Patient with morbid obesity has normocytic anemia with low reticulocyte count= anemia of chronic disease (ACD). Because ACD is triggered by long-standing elevations in serum inflammatory cytokines, it is classically associated with underlying rheumatologic disease, persistent infection, or malignancy. However, ACD is also commonly caused by less obvious sources of inflammation such as type 2 diabetes, CHF, severe traumatic injury, and morbid obesity -Major inflammatory cytokine that drives ACD is hepcidin, a small peptide released from the liver in response to inflammation of bacterial LPS. Hepcidin binds and inhibits iron channels (ferroportin) on the surface of enterocytes and reticuloendothelial macrophages, leading to decreased iron absorption in the gut and decreased iron release by the reticuloendothelial system (which supplies 95% of daily iron for erythropoiesis). Because heme synthesis is impaired without sufficient iron, patients with ACD typically develop a normocytic (or slightly microcytic) anemia with a minimal reticulocyte count option B= occurs with formation of IgG autoantibodies against RBC membrane, which results in RBC destruction by splenic macrophages. Patients generally develop an acute normocytic anemia with increased reticulocyte count option C= patients would have normocytic anemia, significant reticulocytosis, and severe thrombocytopenia due to platelet consumption option D= iron deficiency anemia; low reticulocyte count, initially normocytic, but by the time it is severe= Hb (8.6)= microcytic anemia option E= megaloblastic anemia

26 years woman -Gravida 2 para 1, at 8 weeks of gestation comes to office due to pain and swelling of her left leg for the past day -Pulmonary embolism during her previous pregnancy and prophylactic low molecular weight heparin therapy was initiated 6 days ago -No other medication conditions and takes prenatal vitamins; exam: left lower extremity edema and calf tenderness -Venous duplex ultrasonography= acute left femoral vein thrombosis -Platelet count decreased (normal prior to anticoagulant therapy); other blood cell counts, liver and renal function= normal -Cause to her current condition? a)Acquired protein C deficiency b)Anti-platelet factor 4/heparin antibodies c)Anti-platelet glycoprotein IIb/IIIa antibodies d)Cold-precipitable immunoglobulins e)Decreased ADAMTS13 level

answer: Anti-platelet factor 4/heparin antibodies -Significant drop in platelet count and acute venous thromboembolism following recent exposure to LWMH likely has heparin-induced thrombocytopenia and thrombosis (HITT), also known as heparin-induced thrombocytopenia type 2; HITT typically occurs 5-10 days following exposure to heparin products and is characterized by a large drop in platelet count (more common following exposure to unfractioned heparin but can occur following exposure to LMWH as well) -Platelet factor 4 (PF4) is a protein released from the alpha granules of platelets that plays a role in platelet aggregation; also binds heparin and helps inactivate the molecule -Mechanism of HITT involves generation of IgG antibodies to these complexes of heparin and PF4; The Fc component of the activated IgG antibodies then binds to additional platelets, resulting in further PF4 release and widespread platelet activation this leads to a prothrombotic state that places patients at high risk for both arterial and venous thrombosis option A= occurs early in the course of warfarin therapy, as the inhibition of protein C by warfarin occurs more rapidly than inhibition of other factors, e.g., factors II, VII, IX and X; if not bridged with heparin when starting therapy, patients may develop warfarin-induced skin necrosis due to localized cutaneous thrombus formation option C= idiopathic thrombocytopenic purpura results from splenic destruction of platelets labeled by IgG antibodies to glycoprotein IIb/IIIa receptors; ITP often causes very low platelet levels and is associated with bleeding complications rather than thrombosis option D= cryoglobulinemia can occur in setting of autoimmune disease (e.g., SLE) or viral infection (e.g., hepatitis C) and causes systemic vasculitis characterized by fatigue, arthralgia, and pruritic rash; marked thrombocytopenia is not typical option E= thrombotic thrombocytopenic purpura (TTP) results from decreased levels of the vWF-cleaving protease ADAMTS13; classical presentation of TTP is the pentad of fever, thrombocytopenia, microangiopathic hemolytic anemia, renal insufficiency and neurologic dysfunction

44 years man -Recently diagnosed with idiopathic membranous nephropathy after developing edema= comes to office due to sudden-onset left flank pain and gross hematuria; takes furosemide and ramipril -Exam: left flank tenderness; patient's edema improved -Left-sided varicocele (not noticed before); urinalysis= proteinuria and new hematuria; serum LDH is elevated; urinary loss of which of the following substances predisposed this patient to acute condition? a)Albumin b)Alpha-1 antitrypsin c)Antithrombin III d)Ceruloplasmin e)Immunoglobulins f)Lipoproteins

answer: Antithrombin III -Patient with membranous nephropathy, flank pain, hematuria, and left varicocele likely has renal vein thrombosis (RVT) as a result of nephrotic syndrome -Due to increased glomerular capillary wall permeability in nephrotic syndrome, many important substances are lost in urine; causing a number of complications. Loss of anticoagulant factors, especially antithrombin III, leads to a hypercoagulable state, of which RVT can be a manifestation. Patients with RVT can develop sudden-onset abdominal or flank pain and gross hematuria with elevated LDH as a result of renal infarction -Left-sided varicoceles are relatively common in healthy pubertal men as the aorta and superior mesenteric artery can compress left renal vein ("nutcracker effect"), resulting in increased intravascular pressure in the left gonadal vein with retrograde blood flow and varicocele formation -However, a new onset left varicocele associated with ipsilateral flank pain and hematuria should raise suspicion for RVT causing impaired left gonadal venous drainage option A= also lost in massive quantities in urine in nephrotic syndrome, leading to hypoproteinemia, decreased intravascular oncotic pressure, and fluids shifts into interstitium. Albuminuria explains patient's edema (improved now), but not his acute flank pain, hematuria, and left-sided varicocele, which are more likely due to thrombosis option B= deficiency= inherited condition= early emphysema and cirrhosis option D= decreased in Wilson disease (hepatolenticular degeneration), which is clinically characterized by liver disease, motor abnormalities, and psychiatric symptoms. Ceruloplasmin has no role in pathogenesis of nephrotic syndrome option E= loss of Ig's and low-molecular weight components of complement (such as factor B) makes patients with nephrotic syndrome vulnerable to infections, especially pneumococcal infections option F= lipiduria is common in nephrotic syndrome as a result of increased lipoprotein synthesis by the liver and increased glomerular capillary wall permeability; leading to lipid loss in urine in the form of free fat and oval fat bodies (with characteristic Maltese cross appearance under polarized light). However, lipiduria doesn't contribute to hypercoagulable state

24 years African American male -Comes to office after rapid HIV test was positive -Additional lab evaluation: high viral load and low CD4 count (180 cells/mm3) -Given prescription for antiretroviral therapy; also given dapsone for pneumocystis jiroveci pneumonia prophylaxis due to history of rash with TMP-SMX -Pneumococcal and influenza vaccines administered; few days later, patient returns to office with fatigue, jaundice and dark urine -Labs: Hb (9.0g/dl), MCV (normal), reticulocytes (7.1%), platelets (234,000/mm3), leukocytes (5,500/mm3) -Coagulation studies= normal; peripheral smear= RBC fragments, microspherocytes, and bite cells; cause? a)Abnormal sickling due to medication b)Disseminated intravascular coagulation c)Enzyme-deficiency hemolytic anemia d)Red blood cell cytoskeleton abnormality e)Vaccine-induced hemolysis

answer: Enzyme-deficiency hemolytic anemia -Developed symptomatic anemia (e.g., fatigue) and signs of hemolysis (e.g., jaundice, dark urine) after taking dapsone for a few days, indicating likely G6PD deficiency -G6PD catalyzes the first step of NADPH production. Because NADPH is required to generate the reduced form of glutathione (potent antioxidant), patients with G6PD deficiency lack sufficient reductive capacity to handle significant oxidative stressors and typically develop erythrocyte damage/lysis with acute infection, oxidative foods (e.g., fava beans) and oxidative medications (e.g., dapsone, primaquine) -Oxidative injury to erythrocytes causes Hb precipitation on erythrocyte membrane (Heinz bodies) and impaired RBC deformation while passing through narrow sinusoids of spleen= damaged RBCs are identified and removed by splenic macrophages= leading to extravascular hemolysis -Patients typically develop manifestations 2-4 days after oxidative injury, including acute anemia and signs of elevated bilirubin (e.g., dark urine, jaundice); labs= significant reticulocytosis because the bone marrow increases RBCs to compensate for anemia -PBS= Heinz bodies on periphery of RBCs, and bite cells due to phagocytic removal of precipitated Hb by splenic macrophages (mild intravascular hemolysis also occurs due to intravascular RBC lysis from oxidative damage, schistocytes, and microspherocytes are frequently seen option A= sickle cell anemia, genetic mutation associated with RBC sickling in setting of deoxygenation and dehydration; blood smear= sickled RBCs option B= consumptive coagulopathy characterized by intravascular clot formation, which shears RBCs and can cause schistocytes, these patients also have significant thrombocytopenia and prolonged coagulation times option D= hereditary spherocytosis, genetic disorder marked by RBC cytoskeleton abnormalities; marked by abundant spherocytes, but not bite cells or schistocytes; in addition, this patient developed symptoms after drug administration, G6PD deficiency is more likely option E= do not trigger hemolysis; can cause Guillain-Barre syndrome or rash

7 years boy -Easy bruising, frequent epistaxis (2 weeks) -Exam: pale lips with mucositis in oropharynx; neck is supple -Abdomen is soft and pulses are 2+; labs: Hb (8.2g/dl), platelets (40,000/mm3), WBCs (2,100/mm3) -Biopsy= hypocellular marrow filled with fat; diagnosis? a)Acute lymphoblastic leukemia b)Aplastic anemia c)Myelodysplastic syndrome d)Myelofibrosis e)Parvovirus B19 infection

answer: Aplastic anemia -Patient with pancytopenia has bone marrow findings consistent with aplastic anemia, form of acquired bone marrow failure caused by direct toxic injury or cytotoxic T cell destruction of multipotent hematopoietic stem cells. Because these stem cells generate all mature blood cells for the body, their destruction dramatically decreases mature and immature blood cell populations in the bone marrow; Therefore, bone marrow biopsy= profoundly hypocellular marrow with abundant lipid cells and small clusters of morphologically normal hematopoietic cells -Although most cases are idiopathic, a minority are triggered by viruses (e.g., EBV, hepatitis), exposures (e.g., toxic chemicals, radiation), or medications (e.g., chemotherapy, antiseizure drugs) -Patients usually present with manifestations of pancytopenia (e.g., pale lips= anemia, easy bruising= thrombocytopenia, infections= leukopenia) -In contrast to many other causes of pancytopenia, extramedullary hematopoiesis (e.g., splenomegaly) is usually absent because stem cells are damaged and unable to generate cells in any organ option A= common childhood malignancy; can present with pancytopenia; most patients have lymphadenopathy and hepatosplenomegaly; biopsy would be different option C= is a clonal hematopoietic neoplasm that crowds out the bone marrow, leading to pancytopenia; bone marrow biopsy= hypercellular marrow with abnormally differentiated cells (e.g., hypogranular neutrophils, erythroid precursors with misshapen nuclei), most cases occur in patients age >65 years option D= primary myelofibrosis is a myeloproliferative neoplasm that usually occurs in older adults; patients may be cytopenic, but bone marrow biopsy would show marrow fibrosis; in addition, splenomegaly is usually prominent due to extramedullary hematopoiesis option E= infects erythroid precursors and can cause an aplastic crisis in patients with preexisting hematologic disease, e.g., sickle cell anemia, hereditary spherocytosis; leukocytes and thrombocytes are not affected

35 years man -Nontender cervical lymphadenopathy; biopsy shown -Cytogenic analysis would demonstrate? (learn don't answer) a)BCL-2 overexpression b)BCR-ABL rearrangement c)C-MYC overexpression d)Constitutive tyrosine kinase activation e)N-MYC overexpression

answer: BCL-2 overexpression -Image demonstrates classic histology of follicular lymphoma (a type of non-Hodgkin's lymphoma) -Follicular lymphoma is a B-cell tumor composed predominantly of centrocytes (small cleaved cells; shown in bottom pic) and fewer number of centroblasts (large noncleaved cells) -At low magnification, aggregates of packed follicles that obscure the normal lymph node architecture are seen -The t(14;18) translocation is found in 80-90% of follicular lymphomas; result= overexpression of antiapoptotic gene product, BCL-2, and tumor formation option B and D= t(9;22) is found in CML; CML is a myeloproliferative disorder, not a lymphoproliferative disorder; BCR-ABL fusion product directs the synthesis of a constitutively active protein tyrosine kinase that affects multiple intracellular signaling pathways, leading to uncontrolled proliferation option C= All Burkitt lymphomas are associated with translocation of the C-MYC gene on chromosome 8, usually onto the Ig heavy chain region of chromosome 14; t(8;14); histologically, Burkitt lymphoma shows a diffuse population of medium-sized lymphocytes with a "starry-sky" appearance due to high proliferation index and high rate of apoptosis option E= is common in neuroblastoma and small cell carcinoma of the lung

34 years man -Lump in neck (grown slowly over months); smoker (one pack daily); denies dysphagia, chest pain, weight loss or fever. -Biopsy of mass= abnormal cells with t(14;18) chromosomal translocation; leads to? a)BCL2 overexpression b)BCR-ABL hybrid formation c)C-MYC overexpression d)ERB-B2 overexpression e)p53 inactivation

answer: BCL2 overexpression -Follicular lymphoma; non-Hodgkin lymphoma of the cleaved and noncleaved B cells of the follicular center. -Classic abnormality: t(14;18) translocation, moves BCL-2 (B-cell lymphoma-2) protooncogene from chromosome 18 to chromosome 14, near site of Ig heavy chain enhancer element -BCL-2 is considered a protooncogene because it has anti-apoptotic effects (prevents release of pro-apoptotic factors within affected cells). When this protooncogene is positioned near the Ig enhancer element= resultant BCL2 overexpression allows for cell immortality option B= t(9;22)= AML option C= t(8;14)= Burkitt lymphoma option D= or HER2/neu are all names for the same epidermal growth factor receptor= overexpressed in breast cancer option E= occurs as one of the two "hits" in many human malignancies. Normally, it functions as a tumor suppressor; Li-Fraumeni syndrome is an autosomal dominant cancer syndrome caused by an inherited p53 mutation

24 years woman -3 days of fever, dyspnea, productive cough (yellow sputum) -Exam: fever, high pulse, bronchial breath sounds and crackles over right lower lung -Labs: Hb (13g/dl), platelets (350,000/mm3), leukocytes (54,000/mm3), neutrophils (65%), band forms (10%), myelocytes (3%), metamyelocytes (1%), lymphocytes (15%) -Leukocyte alkaline phosphatase is elevated, finding on peripheral smear? a)Basophilic oval inclusions in mature neutrophils b)Basophilic stippling of red blood cells c)Mature neutrophils with hypersegmented nuclei d)Numerous blasts with azurophilic rods e)Small lymphoid cells with cleaved nuclei

answer: Basophilic oval inclusions in mature neutrophils -Most likely has pneumonia with sepsis and an associated leukemoid reaction, a significant leukocytosis (may exceed 50,000/mm3) that occurs in response to underlying inflammatory condition (e.g., severe infection, hemorrhage, solid tumors). Release of colony-stimulating factors and inflammatory mediators into circulation causes bone marrow to increase production of leukocytes= leukocytosis -Blood smear= numerous mature neutrophils, which may have reactive morphologic features, such as Dohle bodies (blue cytoplasmic inclusion of RER), toxic granulation and cytoplasmic vacuoles. Increased neutrophil precursors (e.g., bands, metamyelocytes, myelocytes) are also typically present (referred to as left shift) due to early release from marrow in response to increased demand of inflammatory condition -Leukocyte alkaline phosphatase (enzyme in neutrophils) can be used to distinguish marked leukocytosis due to leukemoid reaction from CML; values are normal or increased in leukemoid reaction; and low in CML option B= refers to small, blue granules (ribosomal precipitates) in the cytoplasm of RBCs; most often seen in thalassemia, alcohol use disorder, lead/heavy metal poisoning option C= show >6 nuclear lobes and are a feature of megaloblastic anemia; folate/vitamin B12 deficiency option D= AML; would have decreased leukocyte alkaline phosphatase and leukemic failure (no intermediate forms of neutrophils) option E= follicular lymphoma, a mature B-cell neoplasm; centrocyte

45 years man -Progressive fatigue (4 months); works as construction worker; past 2 weeks unable to climb past second floor of building his company is renovating; progressive dyspnea (with minor activities) -Exam: constipation, decreased concentration; overweight male with conjunctival pallor; Labs: hematocrit (26%), MCV (72), ferritin (normal), iron (normal), total iron binding capacity (normal); peripheral smear? a)Basophilic stippling b)Heinz bodies c)Howell-Jolly bodies d)Ring sideroblasts e)Schistocytes

answer: Basophilic stippling -Patient has microcytic anemia, constipation, and mental status changes in the setting of construction work= lead poisoning. Lead is a soft, heavy metal used for manufacturing of batteries, alloys, and ammunition. Individuals working in these industries or others (e.g., construction, mining, smelting, chemical processing, recycling, spray painting, radiator repair) can be regularly exposed to lead -Lead inhibits the heme synthesis pathway (delta-aminolevulinate dehydratase and ferrochelatase), causing a microcytic, hypochromic anemia. Dyspnea particularly with exertion, occurs due to decreased O2 delivery to peripheral tissues. Iron studies are normal unless co-existing iron deficiency is present as well. Basophilic stippling is frequently seen on peripheral smear. Inhibition of erythrocyte 5' nucleotidase by lead results in ribosome aggregation due to inability to degrade RNA, which is represented by coarse blue punctate dots in the cytoplasm of RBCs. Basophilic stippling is nonspecific (thalassemia, myelodysplastic syndrome too) -Adult lead poisoning can also have: 1) bluish pigment (lead line) at gum-tooth line 2) wrist droop or foot drop due to peripheral neuropathy option B= inclusions of denatured hemoglobin; commonly seen after oxidant stress in patients with G6PD deficiency option C= are nuclear DNA remnant inclusions in peripheral RBCs that are normally removed by spleen during circulation; presence of them would indicate decreased/absent spleen function option D= are formed from precipitation of iron granules in developing erythrocytes due to defects in the heme synthesis pathway, including enzymatic inhibition induced by lead poisoning. Ringed sideroblasts are seen in the bone marrow but not peripheral blood smear option E= from mechanical hemolysis; and HUS/TTP/DIC

15 years girl -Persistent fever, fatigue, sore throat. Exam: splenomegaly, and symmetric posterior cervical lymphadenopathy -Peripheral smear= atypical lymphocytes; atypical cells seen are activated form of which cells? a)CD4+ Helper lymphocytes b)CD8+ Cytotoxic lymphocytes c)CD14+ Monocytes d)CD21+ Lymphocytes e)CD27+ Plasma cells

answer: CD8+ Cytotoxic lymphocytes -fever, fatigue, sore throat, splenomegaly, symmetric posterior cervical lymphadenopathy and atypical lymphocytosis= infectious mononucleosis -After entering the bloodstream through the pharyngeal mucosa and tonsillar crypts, the EBV preferentially infects B lymphocytes by binding to CD21 cell surface receptors. EBV- infected B cells then activate cytotoxic T lymphocytes (CD8+) through the presentation of viral antigens on MHC class I molecules. These reactive (atypical) CD8+ T cells are the primary immune response to EBV and clonally expand to destroy virus-infected cells -Atypical lymphocytes make up more than 10% of the cells in peripheral smear in IM, and most (>95%) of those cells are CD8+ T cells; they classically appear as irregular cells much larger than quiescent lymphocytes, with abundant basophilic cytoplasm and a cell membrane that appears to conform to the borders of neighboring cells. Although activated CD21+ B cells and CD4+ T cells can have similar appearance in response to EBV infection, they make up <5% of the atypical lymphocytes in peripheral circulation (choice A and D) option C= e.g., juvenile myelomonocytic leukemia) can be associated with persistent fever, fatigue, splenomegaly; however, peripheral smear would show proliferation of blasts and dysplastic monocytes rather than atypical lymphocytes option E= activated plasma cells appear on histology as ovoid cells with abundant cytoplasm and an eccentric nucleus with a spoke-wheel chromatin pattern; a zone of perinuclear clearing within the cytoplasm may also be noted, corresponding to active Golgi body

60 years man -Lymph node biopsy due to persistent cervical lymphadenopathy -Histology= lymphoid cells arranged in follicular pattern -Cells demonstrate overexpression of BCL2 gene; protein encoded by this gene normally inhibits? a)Cell death cascade b)DNA mismatch accumulation c)Nuclear transcription d)RAS-induced signal transmission e)Telomere shortening

answer: Cell death cascade -Apoptosis (i.e., programmed cell death) is a strictly controlled method of cell death that allows for degradation and removal of cells without evoking an inflammatory response. BCL2 protein functions as an inhibitor of the intrinsic (i.e., mitochondrial) apoptotic pathway -Apoptosis involves triggering a cascade of proteolytic caspase enzymes; in mitochondrial pathway, caspase activation occurs via the movement of cytochrome c protein from the mitochondrial intermembrane space to the cytoplasm; BCL2 protein family regulates the ability of cytochrome c to permeate the mitochondrial outer membrane; this protein family includes both proapoptotic members (e.g., BAX, BAK), which increase the permeability of membrane via pore formation, and antiapoptotic members (e.g., BCL2, BCL-XL), which preserve membrane integrity and prevent release of cytochrome C -Characteristic cytogenic abnormality associated with follicular lymphoma, a mature B-cell lymphoma, is a translocation involving BCL2 gene on chromosome 18 and Ig heavy chain gene on chromosome 14, t(14;18)= this results in overexpression of BCL2 protein, which allows neoplastic cells to evade apoptosis option B= mutations of genes responsible for DNA mismatch repair, e.g., MSH2 can be seen in nonpolyposis colon cancer, i.e., Lynch syndrome; familial predisposition to colon cancer and other visceral malignancies, e.g., endometrial cancer option C= retinoblastoma is a tumor suppressor protein that regulate cell cycle progression; when active, Rb binds E2F transcription factors, prevents transcription, and blocks progression through the G1/S checkpoint option D= RAS protein is a component of the mitogen-activated protein (MAP) kinase pathway, a signaling system that regulates cell proliferation via transmission of stimuli from cell surface receptor to nucleus; neurofibromin, encoded by the NF1 gene, is a tumor suppressor protein that inhibits RAS function option E= telomeres are DNA sequences found at ends of chromosomes that are shortened with cellular division, eventually leading to growth arrest; telomerase is an RNA-dependent DNA polymerase that prevents shortening of telomeres. The majority of cancer cells show increased telomerase activity, thereby avoiding growth arrest and facilitating continuous cell proliferation

12 months boy -Prolonged bleeding from mouth after slipping and hitting his face on table. Mother died shortly after birth due to bleeding complications following home delivery -Blood oozing from gums and marked bruises along trunk and thighs. Labs: decreased vWF antigen levels and activity, measured via the ristocetin cofactor assay; lab values? a)Platelets (decreased); PT (normal); PTT (normal) b)Platelets (decreased); PT (prolonged); PTT (prolonged) c)Platelets (normal); PT (normal); PTT (prolonged) d)Platelets (normal); PT (prolonged); PTT (normal) e)Platelets (normal); PT (prolonged); PTT (prolonged)

answer: Choice C Platelets (normal); PT (normal); PTT (prolonged) Von Willebrand disease (vWD) is caused by genetic mutations that lead to decreased quantity or impaired function of von Willebrand factor (vWF), a multimeric glycoprotein produced by platelets and endothelial cells. vWF has 2 important hemostatic functions -It binds platelets to vascular subendothelial components at sites of injury (and platelets to each other), which results in the formation of hemostatic platelet plugs. Therefore, patients with vWD have impaired platelet aggregation but normal platelet count (as platelet consumption/production is unaffected) -vWF acts as a carrier protein for factor VIII and protects it from degradation. Because factor VIII is part of the intrinsic (contact activation) coagulation pathway, patients with vWD sometimes have prolonged PTT (due to increased destruction of factor VIII), but PT will be normal vWD is typically transmitted in autosomal dominant fashion, so a family history of bleeding is often an important diagnostic clue. Most cases are asymptomatic, but a minority (1%) have easy bruising, skin bleeding, and bleeding from mucous membranes (e.g., oropharyngeal, GI, uterine). Testing for vWF level and activity (e.g., ristocetin cofactor assay) can help confirm the diagnosis -Choice A= ITP; choice B= DIC; choice D= warfarin and vitamin K deficiency; choice E= liver disease

4 years male -Dies in hospital from overwhelming infection -Autopsy= bone deformities, hepatosplenomegaly; clumps of erythroid precursor cells in liver and spleen -Presence of these precursors are related to? a)Frequent transfusions b)Immune deficiency c)Erythropoietin deficiency d)Chronic hemolysis e)Iron deficiency f)Cyanocobalamin deficiency g)Portal hypertension

answer: Chronic hemolysis -Presence of erythroid precursors in liver and spleen= extramedullary hematopoiesis= condition characterized by erythropoietin-stimulated, hyperplastic marrow cell invasion of extramedullary organs; most frequently caused by severe chronic hemolytic anemias (e.g., beta-thalassemia) -Extramedullary hematopoiesis can cause a range of skeletal abnormalities; expanding mass of progenitor cells in the bone marrow thins the bony cortex and impairs bone growth -Pathologic fracture are common in the most symptomatic of children -Maxillary overgrowth and frontal bossing are associated with the characteristic "chipmunk facies" observed in the pediatric population -all other options are not related with extramedullary hematopoiesis -Note: option G= extramedullary hematopoiesis can cause portal hypertension, but not other way around

67 years woman -Worsening fatigue, exertional dyspnea; exam: mucosal pallor -Stool testing for occult blood=negative; labs: Hb (6.7g/dl), WBCs (35,000/mm3), platelets (45,000/mm3); peripheral blood flow cytometry of WBC population= CD20 positive and CD3 negative; diagnosis? a)Adult T-cell leukemia b)Aplastic anemia c)Chronic lymphocytic leukemia d)Chronic myeloid leukemia e)Classic Hodgkin lymphoma

answer: Chronic lymphocytic leukemia -Symptomatic anemia, thrombocytopenia, leukocytosis; predominance of CD20+ cells (B-cell marker)= CLL -In CLL, mature B cells progressively accumulate in the bone marrow and peripheral blood due to oncogenic mutations that inhibit apoptosis. Although most patients are asymptomatic for years, accumulation of B cells in bone marrow eventually chokes out normal hematopoiesis, leading to symptomatic anemia, thrombocytopenia, and infections (neutropenia) -Patients with CLL usually have dramatic elevations in peripheral leukocyte count (often >100,000/mm3) and characteristic findings on peripheral blood smear (e.g., numerous small lymphocytes, smudge cells). The diagnosis is supported by flow cytometry of peripheral blood revealing a clonal population of leukocytes with expression of B-cell surface markers such as CD19, CD20, CD23 option A= would stain positive for CD2, CD3 and CD4 option B=bone marrow failure and is usually triggered by an infection, medication, radiation, toxic substance or autoimmune disease. Patients have pancytopenia (not leukocytosis) and bone marrow biopsy= hypocellular fat filled marrow with normal morphology of hematopoietic cells option D= myeloproliferative neoplasm characterized by peripheral leukocytosis due to numerous circulating granulocytes (e.g., neutrophils in different stages of maturation). Diagnosis= reveals BCR-ABL fusion gene; predominance of CD20+ B cells wouldn't be seen option E= causes localized peripheral lymphadenopathy and B symptoms (e.g., fever, night sweats); diagnosis= lymph node excision, which reveals Reed-Sternberg cells that are characteristically positive for CD15 and CD30; anemia, thrombocytopenia and leukocytosis is atypical

64 years man -Brought to hospital by ambulance (found unresponsive by brother); he dies after failed resuscitative efforts. He had 2 months of progressive fatigue and weight loss prior to episode -Autopsy= massive pulmonary embolus, and liver shows large, solid, yellow-green nodule with multiple smaller nodules -Genomic analysis of hepatic lesions= intranuclear fragments of foreign DNA. These fragments belong to? a)Cytomegalovirus b)Entamoeba histolytica c)Epstein-Barr virus d)Hepatitis B virus e)Hepatitis C virus f)Human papilloma virus

answer: Hepatitis B virus Cross-section of liver= large, solid, yellow-green nodule with multiple smaller nodules= hepatocellular carcinoma (HCC); HBV and HCV infections dramatically increase risk of HCC. Ongoing infection with either virus leads to increased hepatocyte turnover and generation of local inflammatory cytokines, which can result in genetic mutations that lead to malignant transformation. However, HBV has several additional mechanisms that promote HCC, including: -Integration into host genome- nearly 90% of patients with chronic HBV who develop HCC have evidence of HBV DNA in chromosomes of tumor cells. HBV is partially double-stranded DNA virus that is repaired by into a closed circular DNA strand. HBV DNA is often inserted into the cellular genome, although this is not required for viral replication (unlike HIV). Integration into regions that control cell growth and differentiation can result in tumorigenesis -Production of oncogenic viral proteins- HBV produces a viral protein called HBx that is a transcriptional activator of several genes associated with cellular growth. Also, interferes with function of p53 (important tumor suppressor) option A and C= CMV and EBV are members of the Herpesviridae family. Both have double-stranded DNA genomes and cause acute infection followed by latent infection. CMV is not associated with cancer, but EBV is linked to nasopharyngeal carcinoma and hematologic malignancies option B= is a protozoal infection that is typically acquired by ingesting contaminated food or water in developing countries. A minority of patients develop invasive disease with liver abscesses. This usually presents with fever, RUQ pain; organism doesn't incorporate into genome of host cells option E= increases risk of HCC; but it is an RNA virus, doesn't have reverse transcriptase and is unable to integrate into host genome option F= double-stranded DNA virus of the papillomavirus family. HPV produces oncogenic proteins and is able to integrate into host chromosome. However, HPV is not linked with liver cancer; it typically causes cervical and oropharyngeal cancer

-Certain patients with non-small cell lung cancer develop constitutive tumor kinase activity due to production of echinoderm microtubule-associated protein-like 4- anaplastic lymphoma kinase (EML4-ALK), a fusion protein that contributes to carcinogenesis -Most similar to molecular pathophysiology of which disorder? a)Burkitt lymphoma b)Chronic myelogenous leukemia c)Follicular lymphoma d)Li-Fraumeni syndrome e)Mantle cell lymphoma

answer: Chronic myelogenous leukemia -4% of patients with non-small cell lung carcinoma (NSCLC) have an inversion of the short arm of chromosome 2 that creates fusion gene between EML4 (echinoderm microtubule-associated protein-like 4) and ALK (anaplastic lymphoma kinase)= results in constitutively active tyrosine kinase that causes malignancy (usually in young, non-smokers, often with adenocarcinoma, who lack mutations in either the epidermal growth factor receptor gene or the K-ras gene (kinase activity of this fusion protein is a target of the protein kinase inhibitor, crizotinib) -Pathophysiology of EML4-ALK NSCLC is most similar to the pathophysiology of CML; in CML, the classic and most common cause is a translocation between chromosome 9 and 22; the ABL proto-oncogene is transported from chromosome 9 to chromosome 22 where it is placed adjacent to the BCR gene; the resulting oncogene, BCR-ABL, codes for a fusion protein with constitutive tyrosine kinase activity (stimulates proliferation of granulocytic precursors and leads to development of CML); kinase activity is the target of protein kinase inhibitor, imatinib option A= t(8;14); c-myc (8) to Ig heavy chain (14)= overproduction of c-myc, nuclear phosphoprotein that functions as transcription activator option C= t(14;18); overexpression of antiapoptotic BCL2 option D= Autosomal dominant; predisposition to sarcomas and tumors of breast, brain, and adrenal cortex; associated with mutation in tumor suppressor gene p53 option E= t(11;14); cyclin D1 (11) to Ig heavy chain (14)= overproduction of cyclin D1= promoter of the G1 to S phase transition

45 years man -Week of purulent nasal discharge, headache, sore throat, and nonproductive cough -No significant past medical history, except for an episode of infectious mononucleosis at age 22 -Patient smokes a pack of cigarettes daily -Fever, maxillary sinus tenderness, pharyngeal erythema, tender anterior cervical lymphadenopathy -Labs: leukocytosis (neutrophils= 42%; myelocytes= 30%; metamyelocytes= 8%; band forms= 1%; blast cells= 1%; eosinophils= 6%; basophils= 4%) -Leukocyte alkaline phosphatases score is low; diagnosis? a)Acute lymphoblastic leukemia b)Acute myelogenous leukemia c)Burkitt lymphoma d)Chronic lymphocytic leukemia e)Chronic myelogenous leukemia f)Diffuse large B-cell lymphoma g)Follicular lymphoma h)Fungal superinfection i)Leukemoid reaction j)Parasitic superinfection

answer: Chronic myelogenous leukemia -Sinusitis + elevated WBC count with increase in myeloid precursor forms on peripheral smear; The differential diagnosis is chronic myeloid leukemia (CML) (uncontrolled mature granulocyte production, mostly neutrophils but also basophils and eosinophils) or leukemoid reaction (over-exuberant WBC response associated with bacterial infection or malignancy, among others); both cause an elevated WBC count (>50,000/mm3) with an increase in precursor forms (e.g., bands, metamyelocytes, myelocytes); however, the enzyme leukocyte (neutrophil) alkaline phosphatase is decreased in CML, because the WBCs are cytochemically abnormal, by contrast, it is normal or elevated in a leukemoid reaction (Choice I) -Other clues to the diagnosis of CML= predominance of myelocytes compared to more mature forms such as metamyelocytes ("myelocytic bulge") and the absolute basophilia and eosinophilia; CML is confirmed by demonstration of Philadelphia chromosome (t(9;22); BCR-ABL1 fusion gene or mRNA); immature blast cells (e.g., myeloblasts, promyelocytes) are typically <2%; management= tyrosine kinase inhibitor option A= has abnormality of myeloid, not lymphoid cells and his smear shows only 1% of blasts; in general, >25% bone marrow lymphoblasts are seen in ALL, more common in children option B= most common acute leukemia in adults; mean age= 65 years; most patients have WBC counts of 15,000-20,000/mm3 with significant increase in blast cells, e.g., >20%, rather than 1% seen here option C, F, G= lymph node biopsy abnormalities, not peripheral blood option D= increased circulating mature lymphoid cells, not myeloid cells

64 years man -Constant, dull, low back pain, worse with movements (3 months) -Intermittent episodes of difficulty urinating (past year); no medications; smoker (15 years, quit 10 years ago) -Exam: diffusely enlarged prostate with no nodules -Labs: elevated creatinine, urine dipstick negative but 24-hour urinary protein excretion is elevated; urine microscopy: waxy, laminated casts -Imaging of spine: diffuse osteopenia, multiple irregular radiolucent lesions in the thoracic and lumbar vertebrae; biopsy shows? a)Branching papillae lined by cuboidal cells with overlapping nuclei b)Infiltrating glandular cells with substantial adjacent osteoblasts c)Large, malignant cells containing keratin and intercellular bridges d)Clusters of mature plasma cells and plasmablasts e)Sheets of round or polygonal cells with abundant clear cytoplasm

answer: Clusters of mature plasma cells and plasmablasts -Older patient with back pain, several irregularly-shaped vertebral lesions, raising strong suspicion for metastatic disease; Differentiation of the underlying tumor is often assisted by whether the lesion is sclerotic (osteoblastic) or radiolucent (osteolytic) -Multiple myeloma, a clonal plasma cell malignancy, is one of the most common causes of radiolucent bone lesions in adults; the tumor multiplies in the bone marrow and generates osteolytic cytokines, causing bone pain, bone destruction, pathologic fractures and hypercalcemia -Myeloma cells also produce excessive monoclonal immunoglobulin (paraprotein) that can clog the renal tubules and subsequently cause light-chain cast nephropathy; this is typically characterized by mild renal insufficiency, normal urine dipstick for protein (only detects albumin), elevated 24-hour urine protein (detects light-chain proteins), and waxy, laminated casts -Excessive light chains can also form fibrils and deposit in tissues, leading to amyloid light-chain amyloidosis. Monoclonal immunoglobulins are identified by urine or serum protein electrophoresis (M spike) -The diagnosis of myeloma is confirmed by bone marrow aspirate or biopsy, which will demonstrate >10% plasma cells. Plasma cells are identified based upon "clock-face" chromatin, abundant basophilic cytoplasm, and prominent perinuclear clear areas (Golgi apparatus) option A= papillary thyroid cancer is characterized by branching papillae lined by cuboidal cells with overlying nuclei (often calcified psammoma bodies). Most cases progress slowly and present with a thyroid mass that may invade adjacent structures, bone lesions are relatively uncommon option B= prostate adenocarcinoma is a glandular tumor that often spreads to the bones; however, it generates sclerotic (osteoblastic) bone lesions, not radiolucent lesions. Prostate cancer does not generally impinge on the urethra (no urinary symptoms) and is usually associated with a nodule on prostate examination. This patient's intermittent difficulty urinating and diffusely enlarged prostate with no nodules likely indicate benign prostatic hyperplasia option C and E= squamous cell lung cancer is associated with large, malignant cells containing keratin with intercellular bridges. Renal cell carcinoma is associated with sheets of round or polygonal cells with abundant clear cytoplasm; both cause osteolytic lesions but not associated with renal insufficiency, elevated urine protein and waxy urinary casts

Question 2 of 2 -Further evaluation= decreased vWF activity in serum; this protein normally binds to which of the following? a)Collagen b)Fibrin polymer c)Prostacyclin d)Protein C e)Thrombin

answer: Collagen vWF serves 2 important roles in hemostasis: -vWF promotes platelet adhesion at sites of vascular injury by binding to and crosslinking platelet glycoprotein (primarily GpIb) with exposed collagen underneath damaged endothelium. vWF also enhances platelet aggregation, particularly under conditions of high shear stress (such as in small vessels) -vWF functions as a protective carrier protein for factor VIII that increases its plasma half-life. In the absence of vWF, factor VIII is rapidly degraded in the circulation via proteolytic inactivation option B= hemostatic fibrin polymer is the end result of the coagulation cascade. Fibrin also plays a role in inflammation and coagulation regulation option C= functions both as an inhibitor of platelet aggregation and a vasodilator option D= vitamin K-dependent anticoagulant; activated by bound thrombin in the presence of endothelial thrombomodulin. Protein C acts together with protein S to inactivate the activated forms of factors V, and VIII, which are necessary for the production of thrombin option E= produced from prothrombin by activated factor X. Thrombin, in turn, convers fibrinogen to fibrin. Thrombin also activates factors V, VIII and XIII

34 years man -Being treated for acute leukemia, develops oliguria. Serum creatinine is high. Renal biopsy reveals multiple uric acid crystals obstructing renal tubular lumen. The principal site of uric acid precipitation would be which of the following? a)Proximal tubules due to high solute concentration b)Proximal tubules due to impaired uric acid transport c)Loop of henle due to urine hyposmolarity d)Distal tubules due to high urine flow rate e)Collecting ducts due to low urine pH

answer: Collecting ducts due to low urine pH -Tumor lysis syndrome is an oncologic emergency. It often develops during chemotherapy for high-grade lymphomas, leukemias, and often tumors that have rapid cell turnover and high sensitivity to chemotherapy. -When a large number of tumor cells are destroyed during chemotherapy, intracellular ions, such as potassium phosphorus, and uric acid (a metabolite of tumor nucleic acid), are released into the serum and are then filtered by the kidney -Uric acid (pKa= 5.4) is soluble at physiologic pH, but precipitates in an acidic environment. The lowest pH along the nephron is found in the distal tubules and collecting ducts; so these are the segments of the nephron that become obstructed by uric acid crystals. Obstructive uropathy and acute renal failure follow -The risk of tumor lysis syndrome can be reduced by urine alkalinization and hydration. Additionally, allopurinol (a xanthine oxidase inhibitor) is used to reduce uric acid production during the breakdown of tumor cells

-Study: assess acute traumatic blood loss; younger patients (35 years); older patients (70 years) -Older patients with acute traumatic blood loss have much lower reticulocyte count and a much longer time to recovery to pretrauma hemoglobin levels (compared to young) -Age related process that explains this? a)Decreased bone marrow mass b)Decreased medullary cavity size c)Decreased red blood cell life span d)Increased extramedullary hematopoiesis e)Increased marrow fibrous tissue

answer: Decreased bone marrow mass -Although patients with advanced age typically have normal serum blood counts, age-related changes in cytokine signaling and the cellular consumption of the bone marrow impairs the generation of new cells in response to acute stressors such as blood loss or hypoxia. When compared to younger patients, the bone marrow of older patients has a higher quantity of fat and a reduced overall mass, which limits the functional reserve of hematopoietic cells -In addition, hematopoietic progenitor cell diversity and the response to stimulatory cytokines (e.g., stem-cell factor, granulocyte macrophage, colony-stimulating factor) are also impaired, thereby limiting the generation of new cells in response to stress. option B= increases with age due to loss of endosteal cortical bone, this increases risk of fractures

30 years woman -Sudden onset abdominal pain and ascites. Labs: anemia, reticulocytosis, leukopenia and thrombocytopenia. Flow cytometry= CD55 and CD59 deficiency. CT scan= hepatic vein thrombosis. Cause of anemia? a)Complement activation b)Factor V mutation c)Intracellular dehydration d)Mutation in beta globin chain e)Splenic sequestration

answer: Complement activation -Hemolytic anemia, hypercoagulability (e.g., hepatic vein thrombosis) and pancytopenia are characteristic of paroxysmal nocturnal hemoglobinuria (PNH). Despite the name, hemolysis has been shown to occur at low baseline levels throughout the day, with exacerbations caused by a variety of inflammatory triggers (e.g., infections, surgery) -PNH is caused by an acquired mutation of the PIGA gene within a clonal population of multipotent hematopoietic stem cells. This gene is involved in the synthesis of the glycosylphosphatidylinositol (GPI) anchor, a glycolipid necessary for the attachment of several cell-surface proteins, including CD55 (decay-accelerating factor) and CD59 (MAC inhibitory protein). These proteins help inactivate complement and prevent the membrane attack complex (MAC) from forming on normal cells -Absence of the GPI anchor results in CD55 and CD59 deficiency and complement mediated hemolysis. The hemolysis occurs more often at night because complement activity is increased during sleep due to lower blood pH. Patients also develop thrombotic complications (e.g., Budd Chiari syndrome), likely due to release of free Hb and other prothrombotic factors from lysed RBCs and platelets. PNH is often associated with pancytopenia and aplastic anemia (possibly due to autoimmune attack against GPI-antigens on stem cells). Flow cytometry is the gold standard= shows absence of the GPI anchor and CD55 and CD59 deficiency option B= Factor V Leiden, which is resistant to cleavage by activated protein C. Patients with factor V Leiden are at increased risk of thrombotic events., but hemolytic anemia and pancytopenia are not seen Option C= occurs in the pathogenesis of sickle cell anemia and spherocytosis option D= beta-thalassemia option E= complication of sickle cell anemia

50 years woman -History of SLE; admitted with fever, chills, vomiting, burning pain when urinating -Exam: fever, BP (80/50mmhg), high pulse; RR (20/min) -Exam: costovertebral angle tenderness; given IV antibiotics -Over next several hours, she has decreased urine output despite aggressive IV hydration, blood is oozing around central venous catheter -Labs: Hb (9.0g/dl), platelets (68,000/mm3), WBCs (24,500/mm3) -Increased PT and PTT; decreased plasma fibrinogen; cause? a)Accumulation of anticardiolipin antibodies b)Accumulation of ultra-large von Willebrand factor multimers c)Consumptive coagulopathy d)Immune-mediated platelet destruction e)Impaired hepatic synthetic activity

answer: Consumptive coagulopathy -Costovertebral angle tenderness, dysuria, leukocytosis= acute pyelonephritis complicated by septic shock (hypotension, tachycardia); labs show consumptive coagulopathy (thrombocytopenia, prolonged PT/PTT, low fibrinogen), which, along with bleeding from the central catheter site, are likely due to disseminated intravascular coagulation (DIC) -DIC= common complication of sepsis (particularly with gram negative organisms) because lipopolysaccharide is a procoagulant that triggers the coagulation cascade. This leads to formation of fibrin- and platelet-rich thrombi in vasculature, which consumes platelets (thrombocytopenia), coagulation factors (prolonged PT/PTT), and fibrinogen -Fibrinolysis is then triggered to degrade the clots, which elevates D-dimer (fibrin degradation product) and depletes protein C, protein S, and antithrombin; the thrombi also often shear RBCs= leading to microangiopathic hemolytic anemia (MAHA) -Most patients with acute DIC have signs of bleeding, such as oozing from vascular catheters, mucocutaneous bleeding, ecchymosis, or petechiae; end-organ damage (e.g., renal insufficiency, pulmonary hemorrhage) are also common option A= an antiphospholipid antibody often seen in patients with lupus erythematosus; it can cause antiphospholipid syndrome, which is marked by thromboembolism and fetal loss (not bleeding); although patients often have prolonged PTT, PT is normal option B= TTP is caused by reduced activity of von Willebrand factor-cleaving protein ADAMTS13, leading to accumulation of ultra-large von Willebrand factor multimers that trap platelets and generate platelet-rich thrombi in microvasculature; although patients usually develop thrombocytopenia, MAHA, fever, confusion and organ damage (e.g., acute renal failure), the coagulation cascade is unaffected, and bleeding doesn't occur option D= ITP is marked by autoantibodies to platelets/megakaryocytes; thrombocytopenia is the only peripheral blood abnormality; spontaneous bleeding is very uncommon option E= liver generates thrombopoietin and majority of clotting factors; therefore, patients with significantly impaired hepatic function (e.g., cirrhosis) sometimes have mild thrombocytopenia and prolonged coagulation studies; however, fibrinogen level is usually normal, and this patient has no other signs of advanced hepatic dysfunction

70 years man -Palpitations (irregular and fast heartbeat on multiple occasions over last month); no chest pain, dyspnea, dizziness, or weakness -History: hypertension, type 2 diabetes. Pulse is high and irregular, BP is high; ECG= irregularly irregular rhythm with absent P waves -Patient started on metoprolol and apixaban. Which of the following is likely to be directly affected as a result of this therapy? a)Conversion of factor X to Xa b)Conversion of fibrinogen to fibrin c)Conversion of plasminogen to plasmin d)Conversion of prothrombin to thrombin e)Gamma-carboxylation of coagulation factors

answer: Conversion of prothrombin to thrombin -Patient has palpitations, an irregular pulse, and ECG findings consistent with atrial fibrillation. Treatment with a rate control medication (e.g., metoprolol) reduces AV conduction and limits ventricular tachycardia, which helps prevent symptoms (e.g., palpitations, dizziness) and risk of cardiomyopathy. However, rate control agents do not inhibit uncoordinated atrial contraction, so patients remain at risk for atrial thrombus and embolic stroke. As a result, most patients are also initiated on anticoagulation Direct factor Xa inhibitors (e.g., apixaban) are often used for stroke prevention in atrial fibrillation as they are administered orally and do not require monitoring of drug levels (unlike warfarin). This class of medications blocks the active site of factor Xa, which prevents it from convertingprothrombintothrombin.DirectfactorXainhibitors are denoted by names that end in "Xa-ban" option A= beginning of final common pathway of coagulation cascade and is triggered by factor VIIIa (intrinsic pathway) and tissue factor (extrinsic pathway) option B= direct thrombin inhibtors option C= tissue plasminogen activator option E= warfarin

60 years man -Comes to ED due to 10 days worsening cough, dyspnea, right-sided chest pain; low grade fever, fatigue, and weight loss -Month ago= hospitalized due to right lower lobe pneumonia; oral antibiotics were prescribed at discharge, he di not get prescription filled because symptoms improved -Exam: fever, high pulse, decreased breath sounds at right lung base; imaging: loculated pleural fluid on right side -Labs: Hb (10.4g/dl), MCV (96), platelets (580,000/mm3), leukocytes (17,200/mm3; 80% neutrophils) -Blood cell counts were normal a month ago at time of discharge; underlying cause of patient's increased platelet count? a)Autonomous clonal proliferation of megakaryocytes b)Chronic hypoxia-induced erythropoietin release c)Cytokine-mediated megakaryocyte proliferation and maturation d)Decreased platelet sequestration by the spleen e)Increased platelet production from extramedullary hematopoiesis

answer: Cytokine-mediated megakaryocyte proliferation and maturation -Patient with partially treated pneumonia developed progressive pulmonary symptoms and pleural effusion indicating likely empyema; although labs were normal a month ago, CBC now reveals ongoing infection (neutrophilic leukocytosis) and inflammation (anemia, thrombocytosis) -Infections are associated with release of inflammatory cytokines (e.g., IL-6, TNF-alpha) from activated immune cells; elevated circulating concentrations of IL-6 stimulate the liver to increase production of thrombopoietin, which triggers megakaryocyte proliferation and maturation in bone marrow -Subsequent increase in platelet count is called reactive thrombocytosis because it is secondary process that typically resolves when infection is treated; reactive thrombocytosis is seen in a wide range of inflammatory (e.g., malignancy, rheumatologic disease, burns) and noninflammatory (e.g., iron deficiency, hemolysis, blood loss, postsplenectomy) conditions= elevated platelet counts -Elevated circulating levels of inflammatory cytokines also trigger liver to generate hepcidin, which down-regulates iron absorption in gut and iron release from reticuloendothelial system= leads to anemia of chronic disease (normocytic anemia) due to reduced iron availability for reticulocytes option A and E=essential thrombocythemia, myeloproliferative disorder characterized by autonomous clonal proliferation of megakaryocytes, is marked by thrombocytosis, vasomotor symptoms (e.g., dizziness), and increased risk of thrombosis. Primary myelofibrosis is a chronic myeloproliferative disorder characterized by significant extramedullary hematopoiesis (e.g., hepatosplenomegaly), it can occasionally be associated with thrombocytosis; unlikely this patient developed myeloproliferative disorder in the month since he was admitted to the hospital option B= chronic hypoxia can trigger release of EPO from interstitial cells in kidneys, leading to increased erythrocytosis and subsequent polycythemia option D= patients with congestive splenomegaly often have thrombocytosis due to reduced platelet sequestration in the spleen, less likely here

53 years man -Frequent headaches, dizziness; history of hypertension, peptic ulcer disease -Medications: chlorthalidone and antacids as needed -High BP, high BMI; exam: facial plethora and moderate splenomegaly -High hemoglobin, hematocrit, erythrocytes, platelets and WBCs -Mutation in? a)Cytoplasmic tyrosine kinase b)Growth-stimulating transcription factor c)Intrinsic receptor tyrosine kinase d)Tumor suppressor protein e)Vascular growth factor

answer: Cytoplasmic tyrosine kinase -Has polycythemia vera, myeloproliferative disorder characterized by uncontrolled erythrocyte production -Present with nonspecific symptoms (e.g., headache, weakness; diaphoresis), aquagenic pruritus, facial plethora (reddish complexion), and splenomegaly -Associated conditions include: peptic ulcer disease (altered mucosal blood flow due to increased viscosity) and gouty arthritis (higher red cell turnover); Labs= increased erythrocyte mass, thrombocytosis, leukocytosis and low EPO levels -Cause: abnormal transduction of EPO growth signals; erythropoietin receptor has no intrinsic kinase activity and must interact with Janus kinase 2 (JAK2), a cytoplasmic (non-receptor) tyrosine kinase, to initiate downstream signaling -Almost all of patients have JAK2 mutation= constitutive activation of its kinase domain= clonal proliferation of myeloid cells; JAK2 mutations= implicated in essential thrombocythemia, primary myelofibrosis, and other myeloproliferative disorders option B=Burkitt lymphoma; t(8;14); c-myc oncogene (8) translocated to Ig heavy chain region (14)= constitutive expression of c-Myc, a growth-stimulating transcription factor option C= receptors w/ intrinsic kinase activity that autophosphorylated and induce a downstream signaling cascade upon ligand binding; e.g., receptors for insulin, insulin-like growth factor, and epidermal growth factor option D= mutations of p53; associated with Li-Fraumeni syndrome, autosomal dominant disorder with predisposition to a variety of cancer, particularly sarcomas and tumors of breast, brain, and adrenal cortex option E= expression of VEGF is required for tumor angiogenesis, critical step in tumor growth and metastasis

24 years woman -Gravida 1 para 0, at 28 weeks gestation comes to labor and delivery unit due to intense abdominal pain, vaginal bleeding, and absent fetal movement; high BP, high pulse and RR -Pulse oximetry= 98%; exam: tense abdomen and firm, tender uterus -Ultrasound= hematoma between placenta and uterine wall and no fetal cardiac activity; labs: Hb (9.2g/dl), platelets (60,000), AST and ALT are normal -Hospitalized for management; bleeding from gums, IV catheter sites and urinary catheter; vital signs unchanged; underlying cause? a)Embolization of amniotic fluid into the maternal circulation b)Ascending polymicrobial infection of the endometrial lining c)Decidual tissue factor release into the maternal circulation d)Hepatic fatty infiltration with decreased clotting factor production e)Complete rupture of the endometrium, myometrium, and serosa

answer: Decidual tissue factor release into the maternal circulation -Pregnant patient with new bleeding from gums and catheter sites likely has DIC, a life-threatening complication that can occur with postpartum hemorrhage, abruptio placentae, or amniotic fluid embolism. This patient has DIC due to abruptio placentae (i.e., placental bleeding b/w the uterine wall and placenta), which results in premature placental separation, fetal hypoxia, and in severe cases, fetal demise. Patients with abruptio placentae typically have vaginal bleeding, uterine rigidity and/or tenderness, and contractions. Maternal hypertension (e.g., preeclampsia) and smoking are risk factors -Severe placental abruption (e.g., retroplacental hematoma) causes decidual damage and ischemia, thereby triggering tissue factor release into the maternal circulation. Tissue factor, a procoagulant, then activates the extrinsic pathway of the coagulation cascade to cause excessive thrombin generation with formation of circulating platelet-fibrin microthrombi. As clotting factors and platelets are rapidly consumed, a consumptive coagulopathy (e.g., thrombocytopenia, prolonged PT& aPTT) develops that results in bleeding (from mucosal surfaces and IV line sites) option A= amniotic fluid embolism introduces fetal antigens and tissue factor from amniotic fluid into maternal circulation, triggering a massive immune response and increased release of procoagulant factors that can lead to DIC. However, AFE typically causes maternal circulatory collapse with severe hypotension, making this diagnosis unlikely here option B= can cause uterine tenderness, vaginal bleeding, and sepsis, thereby precipitating DIC; however, patients typically also have fever and hypotension option D= acute fatty liver of pregnancy, characterized by microvesicular fatty infiltration of hepatocytes, can impair maternal hepatic function, reducing production of clotting factors and impairing clearance of fibrin degradation products, which may result in DIC; however, liver transaminases would be elevated option E= complete uterine rupture can cause vaginal bleeding, abdominal pain, and fetal compromise. However, patients usually have had a prior uterine surgery and present with palpable fetal parts protruding through the disrupted uterine wall rather than a firm, tense, uterus

29 years woman -Comes to ED due to fever, headache (last week); patient has generalized tonic-clonic seizure while being evaluated -Labs: Hb (6.1g/dl), platelets (16,000/mm3), creatinine (2.2mg/dl), total bilirubin (4.3mg/dl), serum haptoglobin (undetectable), PT (11secs; INR=1.1); activated PTT (30 secs) -Peripheral blood smear= schistocytes; cause of current condition? a)Bacterial product-mediated overactivation of coagulation cascade b)Decreased activity of von Willebrand factor-cleaving protease c)Expansion of neoplastic myeloid precursor cells in bone marrow d)Inherited deficiency of glucose-6-phosphate dehydrogenase enzyme e)Missense mutation at the sixth position of the hemoglobin beta chain

answer: Decreased activity of von Willebrand factor-cleaving protease Patient has classic pentad of manifestation for acquired thrombotic thrombocytopenic purpura (TTP) -Severe thrombocytopenia (platelet rich clots form in microvasculature, leading to rapid platelet consumption) -Microangiopathic hemolytic anemia (MAHA): RBCs are mechanically sheared by microvascular thrombi leading to intravascular hemolytic anemia; typically causes an elevated indirect bilirubin level and an undetectable haptoglobin level, but confirmation of MAHA requires identification of schistocytes (e.g., triangle cells, helmet cells) on PBS -Neurologic manifestations (e.g., headache, seizure), renal insufficiency and fever= these 3 occur in minority of patients TTP is triggered by the formation of autoantibody inhibitors against ADAMTS13, a protease that cleaves ultralarge vWF multimers in circulation, reducing their prothrombotic activity Patients with TTP have very low levels of ADAMTS13, which increases proportion of ultralarge vWF multimers and leads to aggregation and activation of platelets; plasma exchange, which removes the autoantibody inhibitors against ADAMTS13 is generally curative option A= Unlike patients who have DIC (consumptive coagulopathy also associated with MAHA and thrombocytopenia), those with TTP have normal coagulation studies because coagulation factors are not significantly consumed by the formation of platelet rich clots option C= seen in AML which can cause anemia and thrombocytopenia due to bone marrow infiltration, however, PBS would show leukemic blast cells with Auer rods option D= G6PD deficiency causes hemolytic anemia during times of oxidative stress, e.g., infection, medication exposure, certain foods. High bilirubin and low haptoglobin are often present; however, thrombocytopenia is not seen; PBS= bite cells and Heinz bodies option E= sickle cell anemia, marked by periods of acute on chronic hemolytic anemia (e.g., elevated bilirubin, low haptoglobin); however, PBS= sickled RBCs; in addition, thrombocytopenia is generally not seen

56 years man -Evaluated for increased fatigability; past medical history: diabetes mellitus, osteoarthritis, severe aortic stenosis that required aortic valve replacement -Peripheral blood smear= schistocytes; lab findings? a)Increased total serum iron level b)Decreased serum haptoglobin level c)Increased mean corpuscular volume d)Decreased reticulocyte count e)Decreased serum albumin level

answer: Decreased serum haptoglobin level -Fragmented RBCs on smear= schistocytes or helmet cells -Formed by mechanical trauma to RBCs as they circulate through vasculature; in this patient, most likely caused by his aortic valve prosthesis -Artificial (mechanical) valves are more traumatic for RBCs than porcine prostheses and frequently cause hemolysis -Schistocytes can also be formed from narrowing of the microvasculature spaces, as seen in DIC, hemolytic uremic syndrome, and thrombotic thrombocytopenic purpura -Hemolytic anemia due to intravascular RBC destruction results in lab findings similar to those in other types of hemolytic anemias (including increase in serum indirect bilirubin) -Intravascular RBC damage= free hemoglobin in serum (hemoglobinemia) and urine (hemoglobinuria) as well as increased LDH; haptoglobin is a serum protein that binds to free hemoglobin and promotes its uptake by the reticuloendothelial system -Haptoglobin levels decrease when significant quantities of Hb are released into circulation as occurs with intravascular hemolysis option A= occurs in hemochromatosis most classically, but increased serum iron can also be iatrogenic; hemochromatosis doesn't cause anemia, but is associated with cirrhosis and increased incidence of HCC; hemolysis does not usually have significant effect on serum iron because iron released from lysed RBCs remains bound to hemoglobin; increased total serum iron can also occur with sideroblastic anemia; but peripheral smear would show ringed sideroblasts option C= occurs in megaloblastic anemia (e.g., folate/vitamin B12 deficiency) due to presence of enlarged ovoid erythrocytes; increased MCV may also occur with other forms of anemia due to reactive increase in reticulocytes (large, immature RBCs); however, anemia caused by mechanical damage usually results in decreased MCV due to presence of small RBC fragments option D= characteristic of aplastic anemia; in hemolytic anemias, the reticulocyte count is increased to compensate for the increased destruction of RBCs option E= associated with cirrhosis; nephrotic syndrome; and protein-wasting enteropathy

3 years boy -Diagnosed with thalassemia major; receives multiple transfusions to maintain his blood Hb level. -Exam: liver and spleen mildly enlarged; glucose intolerance (oral glucose tolerance test); therapy that would prevent CHF here? a)Erythropoietin b)Cobalamin c)Deferoxamine d)Ascorbic acid e)Digoxin

answer: Deferoxamine -Many hemolytic diseases (e.g., thalassemia major) requires frequent, regular treatment with RBC transfusions for maintenance of adequate Hb level. -Over time, these chronic RBC transfusions cause iron to accumulate within the reticuloendothelial system and organs such as liver and heart -Because iron cardiotoxicity may result in arrhythmias and CHF, it is important to implement chelation therapy within 1-2 years of beginning RBC transfusions -Chelation therapy allows for the removal of excessive iron from the body; and can delay or prevent the development of cardiac disease. Deferoxamine is one of the more effective and safe iron chelators, as it complexes with ferric irons to form ferrioxamine, which is removed by the kidneys.

45 years man -Comes to ED due to epigastric abdominal pain, fatigue; similar episodes of pain in past -Exam: mild epigastric tenderness and hepatomegaly; no jaundice or lymphadenopathy -Labs: Hb (9.0g/dl), MCV (115fl), platelets (130,000/mm3), leukocytes (5,500/mm3) -CT= pancreatic calcifications; RUQ ultrasound= no gallstones; cause of anemia? a)Diminished thymidine synthesis b)Impaired globin chain synthesis c)Impaired pyruvate decarboxylation d)Low rate of intracellular transamination reactions e)Reduced iron availability for hemoglobin synthesis

answer: Diminished thymidine synthesis -patient's recurrent abdominal pain and pancreatic calcifications= chronic pancreatitis, most common cause of which is ethanol abuse -His hepatomegaly is likely due to alcohol-associated hepatic steatosis. The patient also has macrocytic anemia, which often develops in patients with chronic alcohol use due to folic acid deficiency (within months) or vitamin B12 deficiency (within years) -Both deficiencies lead to megaloblastosis, the underlying biochemical feature of which is a defect in DNA synthesis. Folic acid is a single carbon donor required for synthesis of purine and pyrimidine (e.g., thymidine) bases incorporated in DNA. RNA and protein synthesis may continue relatively unaltered, leading to a state of unbalanced cytoplasmic growth with impaired cell division -Pancreatic insufficiency is also associated with vitamin B12 deficiency as pancreatic enzymes normally cleave R factor from B12, allowing B12 to bind to intrinsic factor and be absorbed option B= thalassemias are disorders presenting with reduced or absent globin chain production. Almost all thalassemias cases result in hypochromia and microcytosis option C= oxidative decarboxylation of pyruvate by pyruvate dehydrogenase requires thiamine pyrophosphate (TPP), a derivative of thiamine (vitamin B1). Patients with alcohol use disorder may be prone to thiamine deficiency and could therefore suffer from pyruvate decarboxylation, but this does not cause macrocytic anemia option D= vitamin B6 catalyzes the transaminase reactions by acting as a carrier of the amino group. B6 deficiency can cause anemia that is hypochromic, microcytic and sideroblastic option E= iron deficiency anemia is hypochromic and microcytic

54 years man -Pain and swelling in the right leg; smokes daily for 30 years -Just returned from an overseas trip. Exam: pitting edema of the right leg and tenderness on deep palpation of the calf muscles -Started on medication that prolongs aPTT and PT time in a dose-dependent manner but has no effect on thrombin time; agent administered? a)Cyclooxygenase inhibitor b)Direct factor VIIa inhibitor c)Direct factor Xa inhibitor d)Direct thrombin inhibitor e)Unfractioned heparin

answer: Direct factor Xa inhibitor -Patient has DVT, which is typically treated with anticoagulation therapy for >3 months to prevent recurrent thrombosis. Initial treatment is often with heparin agents (e.g,. Unfractioned heparin, LMWH, fondaparinux), followed by bridging to an oral anticoagulant (most commonly warfarin). Additional treatment options include direct factor Xa inhibitors (e.g., apixaban, rivaroxaban) and direct thrombin inhibitors (e.g., dabigatran) -In general, medications that affect the extrinsic pathway prolong the PT, whereas inhibition of the intrinsic pathway mainly prolongs PTT -Thrombin time is prolonged with medication that directly or indirectly inhibit thrombin. Because factor Xa is located at the junction of the intrinsic and extrinsic pathways, direct factor Xa inhibitors prolong both PTT and PT option B= prolong PT option D= prolong PT, PTT, and TT option E= Binds to antithrombin and causes inactivation of several coagulation factors, most significantly thrombin and factor Xa; PTT and TT will be prolonged; theoretically PT should be prolonged by action of heparin on the final common pathway, in practice, the PT reagent contains heparin neutralizers that minimize this effect

66 years woman -Worsening fatigue, dyspnea w/ moderate exertion (2 months); history of breast cancer (treated w/ surgery and chemotherapy) -Exam: normal; labs: Hb (7.2g/dl), MCV (108), reticulocytes (1%); platelets (90,000), leukocytes (3,800) -Smear: oval macrocytic red cells and hyposegmented neutrophils; biopsy? a)Deposits of atypical gland-forming cells b)Dysplastic erythroid and myeloid progenitor cells c)Hypercellular marrow containing sheets of plasma cells d)Increased macrophage iron with absent sideroblasts e)Large collagen bands replacing the entire marrow

answer: Dysplastic erythroid and myeloid progenitor cells Patient's previous chemotherapy, pancytopenia, and cellular dysplasia (e.g., hyposegmented neutrophils, oval macrocytic erythrocytes)= myelodysplastic syndrome (MDS). MDS is a clonal hematologic malignancy associated with the development of driver mutations due to advanced age, previous chemo-/radiotherapy, or exposure to environmental toxins. MDS is marked by the following: ->1 cytopenias: normal hematopoiesis is impaired due to neoplastic cell replication in the bone marrow, therefore, patients usually present with symptoms of >1 cytopenia such as fatigue/dyspnea on exertion (anemia), bleeding/bruising (thrombocytopenia), or infections (leukopenia). Because erythrocyte production is impaired, reticulocyte count will be low despite significant anemia. However, significant extramedullary hematopoiesis does not occur, so hepatosplenomegaly is rare -Dysplasia of RBCs and neutrophils: peripheral blood smear usually shows normocytic or macrocytic RBCs with a variety of abnormalities (e.g., oval macrocytes). Neutrophils are typically hypolobulated and hypogranular Bone marrow biopsy= hypercellular marrow with dysplasia of >1 cell lines. Myeloblasts are increased (but <20% of total cells), and granulocytes show evidence of impaired maturation such as abnormalities in size, granulation or lobulation; RBCs show signs of abnormal development, including large size and nuclear lobation/budding option A= metastatic adenocarcinoma can infiltrate the bone marrow, leading to pancytopenia. Although reticulocytes would be diminished, smear wouldn't show signs of abnormal granulocyte development option C= multiple myeloma; can cause pancytopenia (anemia alone is more common), smear= rouleaux formation due to increased monoclonal proteins; dysmorphic granulocytes would not be seen. In addition, multiple myeloma is not strongly associated with chemotherapy exposure option D= anemia of chronic disease is associated with iron retention in reticuloendothelial macrophages. Although low reticulocyte count is common, patients usually have normocytic anemia and relatively normal peripheral smear findings (e.g., no dysplasia); in contrast, MDS can be associated with ringed sideroblasts option E= myelofibrosis, chronic myeloproliferative disorder that causes anemia and variable platelet/leukocyte count. However, patients typically have marked splenomegaly due to extramedullary hematopoiesis

5 years boy -2 days of dark, low-volume urine, decreased energy; boy had abdominal pain, fever, and bloody diarrhea for 4 days (resolved 3 days ago without treatment); day before symptoms, the patient swam in a lake and ate hamburgers at a family picnic -Exam: pallor, no edema or rashes; labs: anemia, thrombocytopenia, and elevated BUN and creatinine; findings? a)Elevated haptoglobin level b)Elevated serum indirect bilirubin c)Elevated thrombin and prothrombin time d)Low fibrinogen and elevated D-dimer level e)Positive Coombs test f)Positive Streptozyme test

answer: Elevated serum indirect bilirubin -Patient has hemolytic uremic syndrome (HUS), which is most often caused by infection with shiga toxin-producing organisms such as E. coli O157:H7 or shigella dysenteriae. Shiga toxin (verotoxin) enters the circulation from the bowel and induces capillary endothelial damage, resulting in platelet activation with formation of microthrombi. Platelet consumption causes thrombocytopenia whereas the microthrombi lead to RBC damage (forming schistocytes) and the resultant hemolytic anemia (e.g., pallor, weakness, tachycardia). Damage to glomerular endothelial cells also causes acute kidney injury (e.g., oliguria/anuria, increased creatinine) in 50% of cases -Hemolytic anemia in HUS causes decreased Hb and haptoglobin levels (choice A) as well as increased serum LDH and unconjugated bilirubin. The bleeding time may also be increased due to reduced number of platelets. However, other coagulation studies are normal as there are no clotting factor deficiencies or DIC associated with HUS (choices C and D) option E= detects autoantibodies against RBCs if they are present in the serum. The hemolytic anemia of HUS is mechanical rather than autoimmune, thus, it will be negative option F= detects antibodies against group A streptococcus and can be used to retrospectively diagnose streptococcal infection (e.g., when evaluating for post-streptococcal GN); however children with PSGN have edema and hypertension, not hemolytic anemia

30 years man -History of IV drug abuse, and known HIV infection comes to ED because of increasing abdominal distention and anorexia -CT= ascites and large mass around small intestine -Biopsy of mass= uniform, round, medium-sized tumor cells with basophilic cytoplasm and a proliferation fraction (Ki-67 fraction) of >99% -Infectious agent associated with this condition? a)Epstein-Barr virus b)Human papillomavirus serotype 31 or 33 c)Helicobacter pylori d)Human herpesvirus 8 e)Hepatitis B

answer: Epstein-Barr virus -EBV is the etiologic agent most strongly associated with Burkitt lymphoma, with infection being present in 40% of immunodeficiency-associated Burkitt lymphoma and nearly all cases of endemic Burkitt lymphoma. -t(8;14)= overexpression of c-MYC (controls cell proliferation). Chronic EBV infection increases B-cell proliferation and increases risk of a c-MYC translocation. Pre-existing immunodeficiency secondary to HIV infection can potentiate EBV antigen-induced B-cell proliferation -Histology described= classic for Burkitt lymphoma: diffuse medium-sized lymphocytes and a high proliferation index represented by the high Ki-67 fraction (approaching 100%). The classic "starry sky" appearance is usually seen, and is due to presence of benign macrophages option B= HPV 31, 33, 35, and 51 have been identified in invasive squamous cancers of the cervix, although HPV 16 and 18 are most commonly associated option C= plays a role in GI lymphomas, but it more commonly causes chronic gastritis and peptic ulcer disease. Lymphomas associated with H. pylori infection would be expected to have a different type of histology= usually MALT lymphoma option D= causes Kaposi's sarcoma and primary effusion lymphoma (arising from B-cells), most commonly in HIV infected patients/ Histology of primary effusion lymphoma= large cells, big nuclei and prominent nucleoli option E= can cause hepatocellular carcinoma. A CT scan= tumor mass or masses within the liver; histologically and cytologically also different

22 years woman -Worsening dyspnea, heart pounding with exercise (last week) -No chronic medical conditions, reports aches and pain (past weeks) -Takes ibuprofen as needed; temp (37.2), BP (138/86); pulse (90) -Exam: erythematous rash in sun-exposed regions. Midsystolic click and systolic murmur at apex without radiation; mild tenderness of joints -Labs: Hb (7.8g/dl), reticulocytes (6%), platelets (205,000), WBCs (11,200), creatinine (1.4mg/dl); cause of hematologic findings? a)Bone marrow suppression b)Extravascular hemolysis c)Intrinsic factor antibodies d)Thrombotic microangiopathy e)Traumatic intravascular hemolysis

answer: Extravascular hemolysis -Dyspnea, heart pounding w/ exercise= symptomatic anemia. Other findings= joint pain, photosensitive rash, mild renal insufficiency= SLE, autoimmune disease primarily seen in young women. SLE frequently causes anemia due to combination of factors, including chronic inflammation (anemia of chronic disease), GI serositis (iron deficiency from bleeding) and/or autoimmune hemolytic anemia (AIHA) -10% of patients with SLE develop AIHA due to immune dysregulation, which results in IgG autoantibodies formation against RBC membrane; RBCs coated with IgG are subsequently identified by the Fc receptor on splenic macrophages and partially or wholly phagocytized, leading to extravascular hemolysis. Labs: elevated reticulocyte count because interstitial fibroblasts in the kidney sense tissue hypoxia and increase release of EPO, drives bone marrow erythrocytosis= presence of immature RBCs in smear option A and C= reticulocytosis (e.g., >4.5%) rules out impaired bone marrow response, including bone marrow suppression from cancer or infection (e.g., parvovirus), anemia of chronic disease (cytokine-mediated retention of iron), and vitamin deficiency (e.g., vitamin B12 deficiency from intrinsic factor antibodies, iron deficiency) option D= normal platelet count rules this out; associated with intravascular platelet consumption option E= can occur with a mechanical heart valve or severe aortic stenosis; it is uncommon in healthy young individuals. Patient's murmur is consistent with mitral valve prolapse, common in SLE but rarely causes traumatic hemolysis

12 years boy -Excessive bleeding following tooth extraction. Past history: episodes of painful joint swelling from minor trauma -Exam: soft, nontender abdomen with liver span of 10cm, spleen is not palpable -Labs: bleeding time (normal); PT (increased); aPTT (normal) -Deficiency in which factor? a)Factor VII b)Factor VIII c)Factor XI d)Hageman factor e)Von Willebrand factor

answer: Factor VII -Hemarthroses and excessive bleeding following tooth extraction= coagulopathy (i.e., clotting factor deficiency). Coagulopathies generally present with deep-tissue bleeding into joints, muscles and subcutaneous tissue. In contrast, platelet defects typically manifest with mucocutaneous bleeding (e.g., epistaxis, petechiae). The patient's laboratory results show a normal bleeding time, normal aPTT, and prolonged PT. -Normal bleeding time= adequate platelet function, normal aPTT= intact intrinsic coagulation system. However, prolonged PT= defect in extrinsic coagulation pathway. Therefore, patient most likely deficiency in factor VII option B= Hemophilia A, X-linked recessive; would show prolonged aPTT option C= rare autosomal recessive hemophilia C; would show prolonged aPTT option D= factor XII deficiency; autosomal recessive; not clinically significant but prolongs aPTT option E= vWF carries factor VIII and mediator of platelet adhesion to endothelium; vWD presents with prolonged bleeding time and normal/prolonged aPTT

15 years boy -Fever, purulent nasal discharge (10 days); epistaxis (2 days ago, difficult to control); easy bruising for past 2 months -History: cystic fibrosis (doesn't take medication) -Coagulation studies: PT (elevated), PTT (normal); abnormal? a)Factor V b)Factor VII c)Factor VIII d)Factor XII e)Fibrinogen

answer: Factor VII -Has cystic fibrosis and has bruising, epistaxis, prolonged PT= vitamin K deficiency. Patients with CF are at increased risk due to predisposition to pancreatic insufficiency, in which viscous exocrine secretions block pancreatic ducts. Reduced secretion of pancreatic enzymes leads to fat malabsorption and deficiency of fat-soluble vitamins (i.e., A, D, E, K) -Vitamin K= cofactor in hepatic activation of factor II, VII, IX, and X, as well as protein C and S. Vitamin K deficiency results in low levels of activated vitamin K-dependent coagulation factors, and prolonged PT is caused by deficiency of factor VII. Activated PTT is typically normal (it can be prolonged with severe vitamin K deficiency, due to effect on factor II) -Vitamin K deficiency presents with easy bruising and mucosal bleeding (e.g., epistaxis). Treatment is vitamin K administration, and patients with CF also should receive pancreatic enzyme replacement therapy to improve fat soluble vitamin absorption option A= prolonged PT and PTT option C= Hemophilia A; prolonged PTT option D= prolonged PTT option E= coagulation disorders involving fibrinogen include severe liver disease, DIC which are associated with prolonged PT and PTT

50 years man -Long standing alcohol use disorder, difficulty breathing; low BP, high pulse, and RR; pulse oximetry 92% -Exam: bilateral basal crackles, increased jugular venous pressure, hepatomegaly, ascites, and peripheral pitting edema -X-ray= cardiomegaly; scattered ecchymoses across extremities -Labs: platelets (120,000), PT (prolonged), aPTT (normal) -Patient given intramuscular vitamin K, 2 days later, labs unchanged; cause? a)Dietary vitamin K deficiency b)Factor VII deficiency c)Factor VIII deficiency d)Intrinsic platelet dysfunction e)Von Willebrand factor deficiency

answer: Factor VII deficiency -Patient has liver dysfunction due to alcohol use disorder. The liver synthesizes many proteins, including clotting factors. Chronic alcohol use leads to progressive hepatic fibrosis/cirrhosis, resulting in acquired coagulopathy -Clotting factors II, VII, IX, and X are produced initially by the liver in an inactive form and then activated by vitamin K-dependent carboxylation. Factor VII, part of the extrinsic pathway, has the shortest half-life of all coagulation factors -PT assesses the extrinsic and common pathways of coagulation and is the first to become abnormal in liver disease. This patient's PT prolongation is likely due to factor VII deficiency in the setting of cirrhosis. Because clotting factor synthesis is impaired, PT may not improve with vitamin K supplementation as there are insufficient quantities of clotting factors to undergo vitamin K-dependent carboxylation/activation -The liver also synthesizes albumin, and hypoalbuminemia results in ascites and peripheral edema. In addition, severe cirrhosis causes high-output heart failure due to chronic splanchnic vasodilation and development of mesenteric and intrahepatic arteriovenous shunts option A= impairs activation of factor II, VII, IX and X; with malnourishment, fat malabsorption, long-term antibiotic use option C= Hemophilia A; prolonged aPTT option D= cirrhosis is associated with intrinsic platelet dysfunction and thrombocytopenia due to decreased hepatic synthesis of thrombopoietin and increased splenic sequestration (secondary to hypersplenism). Platelet dysfunction and significant thrombocytopenia (e.g., platelets <50,000/mm3) lead to mucocutaneous bleeding/petechiae and prolonged bleeding time, but not PT option E= increased bleeding time, and normal/increased PTT

Question 1 of 2 23 years African American man -Comes to ED due to back and lower extremity pain (2 days) -Several similar episodes over the last 15 years that have required hospitalization -Pain is not responsive to over-the-counter analgesics, prior records shows he required opioids for pain relief -Exam: scleral icterus, and tenderness over his lower back and long bones of his thighs; Labs: Hb (6.7g/dl) -Histology of spleen? a)Congestion b)Cyst formation c)Fibrosis and atrophy d)Infiltration e)White pulp hyperplasia

answer: Fibrosis and atrophy -Patient with frequent episodes of bony pain likely has sickle cell disease (SCD), autosomal recessive condition characterized by hemolytic anemia and vaso-occlusion -Hemolytic anemia causes jaundice (due to unconjugated hyperbilirubinemia) and promotes formation of pigmented gallstones -Vaso-occlusion by sickled RBCs occurs in various tissues, causing hypoxia and acidosis (vaso-occlusive pain episodes); microvascular occlusion nearly always affects the spleen due to the trapping of sickle cells by the rapid splenic cords; Autoinfarction begins in early childhood and can precipitate splenic sequestration crisis in infants -However, repeated infarction eventually leads to significant scarring, fibrosis and atrophy of the spleen, which would likely be present in this adult patient -Asplenic patients are more susceptible to infections with encapsulated bacteria (e.g., group B streptococcus, H. influenzae, Streptococcus pneumoniae, Neisseria meningitidis, Salmonella typhi) option A= occurs during a splenic sequestration crisis (marked hemoglobin decrease, rapidly enlarging spleen), which develops due to vaso-occlusion within the cords of Billroth and splenic pooling of erythrocytes. However, repeated infarction leads to functional asplenia and autosplenectomy (splenic atrophy) by late childhood/adolescence option B= splenic cyst formation occurs due to infection option D= can be seen with metastatic neoplasms, not a feature of SCD option E= composed of lymphoid tissue; lymphoid hyperplasia may represent infection or malignancy

37 years man -Comes to ED due to blood-tinged vomiting and abdominal discomfort -Lost job and began drinking lots of whiskey daily (6 months ago); hospitalized several times with alcohol intoxication -Exam: firm, enlarged liver. Peripheral smear= neutrophils with 6-8 nuclear lobes. Explanation for latter finding? a)Blood lipid abnormality b)Chronic blood loss c)Cobalamin deficiency d)Folate deficiency e)Hypothyroidism f)Myelodysplasia

answer: Folate deficiency -Alcohol use disorder is one of the most common causes of folate deficiency anemia due to poor dietary intake and impaired folate absorption, utilization, and enterohepatic recycling -Normal individuals with folate-deficient diet can maintain normal RBC production for months due to folate recycling, whereas a patient who consumes a large amount of alcohol will experience anemia within a few weeks -Reduced form of folic acid, tetrahydrofoleic acid, is necessary for synthesis of amino acids, thymidine, and purines. Impaired nucleotide synthesis leads to defective DNA production in blood cell precursors, resulting in abnormal cell division and megaloblastic hyperplasia of bone marrow. The peripheral smear shows pancytopenia and hypersegmented neutrophils containing nuclei with >5 lobes. RBC abnormalities include ovalocytosis and macrocytosis, with MCV >100fl option A= extreme hypertriglyceridemia can cause acute pancreatitis option B= can cause iron deficiency anemia option C= associated with pernicious anemia, gastrectomy, certain medications, and fish tapeworm infections. Chronic alcohol use can deplete vitamin B12 levels, however, given the body's large vitamin B12 stores, this depletion would take place over a period of many years not months. In addition, there is no mention of neurologic symptoms that would accompany vitamin B12 deficiency option E= elevated MCV can present in hypothyroidism, but hypersegmented neutrophils are not seen option F= premalignant condition that manifests with pancytopenia, impaired blood cell differentiation, and clonal expansion of mutated hematopoietic cells in the bone marrow. Elevated MCV can be present, but hypersegmented neutrophils are not seen

42 years woman -Fever, sore throat; exam: tonsillar exudate and nontender cervical lymph node 3.5 cm in diameter (antibiotic therapy= symptoms resolved within a week) -Previously enlarged lymph node decreased slightly in size. On follow-up visits over the following year= patient asymptomatic, and size of lymph node fluctuates but doesn't disappear completely -Excisional lymph node biopsy performed; diagnosis? a)Acute lymphoblastic leukemia b)Burkitt lymphoma c)Diffuse large B-cell lymphoma d)Follicular lymphoma e)Hairy cell leukemia f)Mycosis fungoides

answer: Follicular lymphoma -Persistent fluctuating lymphadenopathy, who have had an unrelated pharyngitis (treated with antibiotics) on initial presentation= most likely has follicular lymphoma -Follicular lymphoma is the most common indolent NHL in adults and second most common NHL overall (after diffuse large B cell lymphoma). It derives from germinal center B cells and typically has a long, waxing and waning clinical course -The condition presents in middle aged patients with painless lymph node enlargement or abdominal discomfort from an abdominal mass. Histology= nodular pattern and neoplastic follicles are composed of a mixture of centrocytes (cleaved cells) and centroblasts (noncleaved cells) -The majority of tumors exhibit t(14;18)= overexpression of BCL2 oncogene (blocks programmed cell death) option A= or lymphoma; most common malignancy in children, presents with lymphadenopathy, hepatosplenomegaly, fever, bleeding, and bone pain. Neoplastic cells are pre-B or pre-T cells option B= highly aggressive but chemotherapeutically responsive; B-cell NHL with chronic EBV infection and/or deregulation of the MYC proto-oncogene. Patients typically develop rapidly growing tumor masses in the facial bones, jaw, or abdomen. Tumor doubling time is very rapid and spontaneous tumor lysis can occur option C= typically presents with a rapidly enlarging nodal (neck, abdomen, mediastinum) or extranodal symptomatic masses. The Waldeyer's ring (oropharyngeal lymphoid tissue) and GIT are commonly involved, and systemic "B" symptoms (fever, weight loss, drenching night sweats) can be seen option E= mature B-cell neoplasm, presents with splenomegaly, pancytopenia most often in older men; lymph node enlargement is not characteristic. Leukemic cells have hairlike cytoplasmic projections and are positive for tartrate-resistant acid phosphatase option F= is a cutaneous T-cell lymphoma; proliferating CD4+ atypical lymphocytes infiltrate the dermis and epidermis, where they form Pautrier microabscesses; this condition manifests with plaques (often on trunk or buttocks) that may be confused with eczema or psoriasis. Generalized erythema and scaling and thickening of skin (erythroderma) may result

6 years boy -Fatigue, abdominal distention; patient bruises easily and appears pale -Exam: subconjunctival pallor and several ecchymoses on the boy's extremities -Distended abdomen, massive splenomegaly and moderate hepatomegaly -Labs: Hb (8g/dl), WBCs (3,800 cells/mm3), platelets (90,000 cells/mm3) -Bone marrow aspirate shown; diagnosis? (learn don't solve) a)Acute lymphoblastic leukemia b)Epstein-Barr virus c)Fanconi anemia d)Gaucher disease e)Parvovirus B19

answer: Gaucher disease -Gaucher disease: autosomal recessive, lysosomal storage disease, caused by deficiency of beta-glucocerebrosidase -It breaks down glucocerebroside (in lysosome); deficiency= leads to accumulation of glucocerebroside within lysosomes of macrophages as they engulf and digest cellular debris= results in lipid-laden macrophages (Gaucher cells)= give characteristic "wrinkled tissue paper" appearance and can be found in bone marrow or other affected tissues (e.g., spleen) Clinical manifestations develop as Gaucher cells accumulate and secrete lysosomal proteases and cytokines that disrupt cellular function; onset of symptoms varies from childhood to early adulthood and includes: -Abdominal distention and hepatosplenomegaly due to accumulation of Gaucher cells -Bone pain due to marrow invasion and release of inflammatory mediators, resulting in osteolytic lesions and bone infarction -Pancytopenia due to splenic sequestration and bone marrow infiltration, resulting in easy bleeding and bruising, pallor and fatigue, and leukopenia option A= one of the most common childhood cancers; leukemia can present with pancytopenia and hepatosplenomegaly, but bone marrow would show lymphoblasts option B= presents with fever, pharyngitis, cervical lymphadenopathy, fatigue and splenomegaly; atypical reactive lymphocytes would likely be found on peripheral blood smear option C= inherited cause of aplastic anemia; in addition to pancytopenia, patients often have hypo/hyperpigmented patches, short stature, hypoplastic thumbs, as well as increased risk of malignancy, e.g., AML option E= common cause of erythema infectiosum, can trigger a transient aplastic crisis in those with an underlying hematologic abnormality, e.g., sickle cell disease, hereditary spherocytosis. Bone marrow aspirate would reveal erythroid precursors with intranuclear viral inclusions

56 years man -Worsening fatigue, recurrent epistaxis. Evaluation= anemia, thrombocytopenia, abnormal circulating leukocytes -Bone marrow biopsy= acute myeloid leukemia. Initially treated with induction chemotherapy, and an allogeneic hematopoietic stem cell transplant is recommended -Nonrelated, HLA-matched donor is not found, so the patient's adult son volunteers to be the donor. The plan is for the patient to receive conditioning chemotherapy and radiotherapy followed by bone marrow transplant. Prior to transplant, a procedure is performed to deplete CD3+ cell in the marrow graft. This pretransplant procedure would be most helpful in decreasing the risk of which? a)Cytomegalovirus infection b)Graft versus host disease c)Immunoglobulin deficiency d)Leukemia relapse e)Lymphoproliferative disease

answer: Graft versus host disease Allogenic stem cell transplantation is used in the treatment of leukemia to provide a new source of hematopoietic stem cells (HSCs) after destruction of the patient's immune cells (including malignant cells) with chemotherapy. The donor's HSCs are frequently obtained from bone marrow or peripheral blood and contain differentiated immune cells such as CD3+ T cells These donor CD3+ (CD4+/CD8+) T cells can recognize HLA-mismatched host cells (including residual leukemia cells) as foreign, providing a beneficial graft vs. leukemia/tumor effect. However, they can also attack normal host cells and initiate graft vs. host disease (GVHD), a widespread inflammatory response that characteristically damages the skin (e.g., maculopapular rash), GIT (e.g., diarrhea, abdominal pain) and liver (e.g., elevated serum bilirubin) Depletion of donor CD3+ T cells prior to HSC transplant is performed primarily to decrease GVHD risk, but it also has the following undesirable effects on the host: -Decreased graft vs. leukemia effect, increasing the risk of leukemia relapse (choice D) -Depletion of memory T cells, which increases risk of viral (e.g., CMV, HHV-6) infections (choice A). This also increases the risk of posttransplant lymphoproliferative disease due to reduction in donor T cells that normally attack proliferating B cells infected with EBV (choice E) option C= normal effect of myeloablative chemoradiotherapy that improves over time with engraftment (proliferation and differentiation) of donor HSCs. Removal of donor T cells before the transplant would increase the period of B cells and Ig deficiency by slowing engraftment and reestablishment of host immune function

62 years man -Worsening fatigue. Has long-standing type 2 diabetes complicated by nephropathy -Labs: Hb (9.4g/dl), MCV (90), platelets (200,000), leukocytes (7,500), creatinine (elevated), bilirubin (normal), serum ferritin (elevated), transferrin saturation (normal) -After appropriate treatment is initiated for his anemia, changes to erythroid cells? a)Decrease in free protoporphyrin levels b)Decrease in reticuloendothelial destruction c)Increase in progenitor cell differentiation d)Switch of hemoglobin to the fetal form e)Synchronization of nuclear and cytoplasmic maturation

answer: Increase in progenitor cell differentiation -Patients with chronic kidney disease (CKD) often develop normocytic anemia as the GFR worsens due to inadequate production of EPO, a glycoprotein hormone produced by peritubular fibroblast cells in the renal cortex. EPO is released into the bloodstream in response to renal tissue hypoxia and subsequently acts on erythrocyte progenitor cells (erythroid colony-forming unit cells) in the bone marrow to reduce apoptosis and increase differentiation into mature RBCs -Although healthy individuals respond to anemia by increasing EPO levels up to 10,000 folds, patients with CKD have chronic inflammatory damage to renal EPO-producing cells and are often unable to generate sufficient levels of EPO to maintain adequate erythropoiesis. These individuals are often treated with synthetic forms of EPO (e.g., epoetin, darbepoetin) to supplement intrinsic EPO production and stimulate erythropoiesis. Because synthetic forms of EPO can rapidly deplete iron stores (due to increased RBC production), patients should be tested for iron deficiency prior to treatment with these agents option A= protoporphyrins (heme precursors) are elevated in patients with iron deficiency anemia. Treatment of iron deficiency anemia (e.g., iron supplementation) leads to the conversion of protoporphyrins to heme, thereby reducing free protoporphyrin levels. Patients with iron deficiency anemia usually have low serum ferritin levels and transferrin saturation option B= warm autoimmune hemolytic anemia is characterized by antibodies against antigens on the RBC membrane that result in phagocytosis in the reticuloendothelial system (e.g., spleen). Autoimmune hemolytic anemia is usually treated with immune-suppressing agents (e.g., prednisone), which blunt antibody production and reduce RBC destruction. This patient with normal bilirubin level is unlikely to have hemolytic anemia option D= sickle cell anemia; treated with hydroxyurea which induces formation of HbF which is unaffected by the sickle mutation option E= megaloblastic anemia; treated with vitamin B12/folate supplementation

32 years woman -Several days of fatigue and malaise; thrombocytopenia -Thrombotic thrombocytopenic purpura is suspected; what's required to make diagnosis? a)Abnormal neurologic examination b)Elevated liver aminotransferases c)Fever d)Hemolytic anemia e)Positive Coombs testing f)Renal failure

answer: Hemolytic anemia -TTP is a thrombotic microangiopathy characterized by the formation of platelet-rich thrombi in the microvasculature. It is caused by severe deficiency of ADAMTS13, an enzyme that cleaves ultra-large vWF multimers on the endothelial surface. Loss of ADAMTS13 triggers spontaneous platelet aggregation and activation, which results in thrombocytopenia due to platelet consumption and microangiopathic hemolytic anemia (MAHA) due to mechanical shearing of RBCs -Acquired TTP (autoantibody against ADAMTS13) was initially described as a pentad of fever, neurologic symptoms, kidney disease, MAHA and thrombocytopenia. However, up to 1/3 of patients do not have neurologic abnormalities, half have normal kidney function and 1/5 only have fever (choice A, C and F). In fact, many with acquired TTP have minimal signs of toxicity (as in this case) -Therefore, the only factors required to further evaluation of suspected thrombotic microangiopathy (e.g., TTP, HUS) are the presence of thrombocytopenia and MAHA on peripheral smear (e.g., schistocytes); significantly reduced ADAMTS13 activity can confirm the diagnosis option B= can sometimes be elevated in hemolytic anemia because these enzymes are present in RBCs option E= in TTP, hemolytic anemia is caused by shearing of RBCs in microvasculature, it would be negative

32 years woman -Worsening fatigue, exercise intolerance. Exam: mild mucosal pallor, Hb (7.8g/dl); peripheral smear= polychromatic RBCs, nucleated RBCs, and spherocytes -Cause of anemia? a)Erythropoietin deficiency b)Hemolytic anemia c)Iron deficiency anemia d)Thrombotic microangiopathy e)Vitamin B12 deficiency

answer: Hemolytic anemia This patient with symptomatic anemia (e.g., fatigue, exercise intolerance) has peripheral smear findings indicative of hemolytic anemia, including the following: -Increased reticulocytes: Reticulocytes are larger than mature RBCs, lack central pallor, and have a bluish coloration (polychromatic RBCs) due to presence of ribosomes, which are not found in mature RBCs. Increased reticulocytes in peripheral smear is nonspecific finding that indicates bone marrow is responding appropriately to anemia by increasing production of RBCs -Spherocytes: spherocytes are RBCs with reduced cell membrane, which makes the cell spherical rather than biconcave. In otherwise healthy individuals, spherocytes generally arise in the setting of immune-mediated hemolytic anemia. In this condition, IgG-coated RBCs are partially phagocytized by splenic macrophages, which reduces the surface area of RBC membrane, thereby causing the RBC to assume a spherical shape -Nucleated RBCs: nucleated RBCs are very immature RBCs. The presence of nucleated RBCs in peripheral smear indicates a strong bone marrow response to compensate for the anemia. option A= often seen in patients with ESRD b/c EPO production by the kidneys is impaired. Individuals with EPO deficiency are not able to sufficiently increase erythrocytosis in response to anemia, therefore, smear wouldn't show increased reticulocytes and nucleated RBCs option C= see in young women with heavy menstruation or GI bleeding; smear would typically show hypochromic RBCs and microcytes option D= would show schistocytes option E= megaloblastic anemia

9 years boy -Beta thalassemia major; comes for routine RBC transfusion (diagnosed at age of 6 months and since received numerous transfusions and chelation therapies; been well tolerated) -Exam: mild frontal bossing, hepatosplenomegaly, jaundice -Recent biopsy= Kupffer cells containing coarse, yellowish-brown cytoplasmic granules; these granules composed of? a)Amyloid b)Bilirubin c)Copper d)Glycogen e)Glycolipid f)Hemosiderin g)Lipofuscin

answer: Hemosiderin -Patients with chronic hemolytic anemia (e.g., beta thalassemia major) depend on recurrent RBC transfusions to maintain adequate Hb level. Iron overload (hemosiderosis) from increased iron absorption is a common complication resulting from both primary condition (e.g., hemolysis) and its treatment (frequent RBC transfusions) -Circulating iron is carried by transferrin. Once deposited in cells, iron binds to apoferritin to form ferritin micelles. Because iron is utilized poorly in patients with thalassemia and cannot be excreted actively, the ferritin micelles accumulate in macrophages of the reticuloendothelial system. The resulting iron storage complex is known as hemosiderin and microscopically appears as brown or yellowish-brown pigments in either granular or crystalline form. Hemosiderin can be identified histologically with Prussian blue stain. In the liver, hemosiderin is typically seen in Kupffer cells (hepatic macrophages that line the walls of the sinusoids and participate in RBC breakdown) -Patients receiving regular transfusions should undergo routine iron chelation therapy to reduce the overall iron load within body and improve survival. Eventually, iron burden will overwhelm the reticuloendothelial cells capacity to sequester iron, resulting in parenchymal iron overload in the liver, myocardium, skin and pancreas (e.g., bronze diabetes) option A= amyloid protein has a pale rose color after Congo red staining. Accumulation of amyloid fibrils can result in a variety of sequelae, e.g., cardiomyopathy, nephrotic syndrome option B= primary pigment found within bile and is byproduct of hemoglobin breakdown; hemolytic anemia can cause increased levels and jaundice option C= Wilson disease; copper accumulation= cirrhosis and neurologic complications option D= appears as clear cytoplasmic vacuoles and stains pinkish purple with PAS; glycogen is abundant in the liver shortly after a meal option E= patients with Gaucher disease (glucocerebrosidase enzyme deficiency) have pancytopenia, hepatosplenomegaly, and pathologic fractures; histology= Gaucher cells which are macrophages laden with cerebrosides and other glycolipids= wrinkled paper appearance option G= insoluble yellow-brown pigment composed of lipids and phospholipids complexed w/ proteins. This pigment accumulates with age and "wear and tear" of multiple organs

63 years man -Comes to ED due to abdominal pain; exam: abdominal tenderness without guarding or rebound -Labs: decreased hemoglobin, haptoglobin and platelets; increased total bilirubin and LDH -Magnetic resonance angiography= mesenteric vein thrombosis -Flow cytometry= absence of CD55 on surface of RBCs; most likely pathologic renal finding in this patient? a)Cast nephropathy b)Hemosiderosis c)Interstitial nephritis d)Membranous glomerulonephritis e)Minimal change disease

answer: Hemosiderosis presentation= consistent with paroxysmal nocturnal hemoglobinuria (PNH), disorder due to complement-mediated hemolysis PNH is usually due to mutated phosphatidylinositol glycan class A (PIGA) gene, which helps synthesize the glycosylphosphatidylinositol (GPI) anchor protein; this protein helps attach several cell surface proteins (e.g., CD55 decay accelerating factor, CD59 MAC inhibitory protein) that inactivate complement Absence of these proteins leads to uncontrolled complement-mediated hemolysis Manifestations of PNH include the following: -Fatigue and jaundice due to hemolytic anemia (elevated bilirubin and lactate dehydrogenase, low haptoglobin, hemoglobinuria (which may be nocturnal)) -Thrombosis at atypical sites (e.g., hepatic, portal or cerebral veins) possibly due to release of prothrombotic factors from lysed RBCs and platelets (mesenteric vein thrombosis may explain the patient's abdominal pain) -Pancytopenia due to stem cell injury Chronic hemolysis with breakdown of iron-containing RBCs can also lead to iron deposition in the kidney (hemosiderosis), which can interfere with proximal tubule function and cause interstitial scarring and cortical infarcts; the hemosiderosis combined with microvascular thrombosis can increase the risk of chronic kidney disease option A= usually seen with multiple myeloma and is due to large amounts of monoclonal free chain deposition in the kidney option C= inflammation in the area surrounding the renal tubules; most commonly due to drugs (e.g., analgesics, antibiotics), recent infection, or systemic conditions (e.g., sarcoidosis) option D= can be associated with SLE, drugs (e.g., NSAIDs), hepatitis B and C, or solid tumor malignancies (e.g., lung, prostate, GI) option E= associated with drugs (e.g,. NSAIDs), hematologic malignancies (e.g., lymphoma, leukemia) or allergens (e.g., fungi)

41 years woman -Long history of menorrhagia, comes due to low Hb level identified on normal lab evaluation. Diagnosed with iron deficiency anemia and started on oral ferrous sulfate -After 3 months of therapy, her labs improve. Ask to stop her supplement as she is concerned about iron buildup and damage to vital organs -Physician reassures her and explains that iron homeostasis is maintained by multiple types of cells in the body; which of the following cells secrete a substance that controls iron storage and release by other cells involved in iron homeostasis? a)Biliary lining cells b)Bone marrow macrophages c)Bone marrow stem cells d)Hepatic parenchymal cells e)Intestinal epithelial cells f)Renal tubular cells

answer: Hepatic parenchymal cells Hepcidin is an acute phase reactant synthesized by the liver that acts as the central regulator of iron homeostasis. High iron levels and inflammatory conditions increase the synthesis of hepcidin, while hypoxia and increased erythropoiesis act to lower hepcidin levels. Hepcidin influences body iron storage through its interaction with ferroportin, a transmembrane protein responsible for transferring intracellular iron to circulation. Upon binding hepcidin, ferroportin is internalized and degraded, decreasing intestinal absorption and inhibiting release of iron by macrophages Regulation of intestinal iron absorption is crucial for maintaining iron homeostasis, since blood loss is the only way of removing large amounts of iron from body. Iron absorption from the proximal small intestine is facilitated by DMT-1. Once inside the intestinal cells, iron may take 1 of 2 paths: -Iron may bind to ferritin (primarily intracellular iron binding protein) and remain stored within the enterocyte. The stored iron is excreted in stool as enterocytes slough off and are replaced -Iron may enter the circulation through ferroportin, the basolateral iron transporter of the enterocyte. Free iron released into the circulation is transported through the body by transferrin (an iron binding transport protein), which becomes internalized after interacting with transferrin receptors present on all cells

32 years African American male -Diagnosed with acute prostatitis -Begins treatment with trimethoprim-sulfamethoxazole, but subsequently develops dark urine and anemia with a high reticulocyte count -Explanation? a)Antibody-mediated erythrocyte destruction b)Hereditary erythrocyte membrane defect c)Hereditary erythrocyte enzyme deficiency d)Hemoglobin structure abnormality e)Microangiopathic hemolytic anemia

answer: Hereditary erythrocyte enzyme deficiency -Patient demonstrating signs of hemolysis induced by TMP-SMX (Bactrim). Common way for glucose-6-phosphate dehydrogenase (G6PD) deficiency to present -G6PD deficiency is an X-linked disorder that affects mostly males, and is more common in patients of African, Asian and Mediterranean descent -G6PD is an enzyme of the pentose phosphate pathway that in erythrocytes is essential for maintaining adequate concentrations of NADPH; insufficient NADPH results in an ability to maintain glutathione in the reduced state -This increases the vulnerability of erythrocytes to oxidative stress and manifests with hemolysis induced by infections, drugs (TMP/SMX/sulfa drugs, antimalarials, nitrofurantoin) or other oxidants (e.g., fava beans) -Hemolytic episodes manifest with symptoms of anemia such as malaise and pallor, indirect bilirubinemia (jaundice), hemoglobinemia and hemoglobinuria (dark red urine); the level of haptoglobin decreases and a reticulocytosis develops to compensate for the increased destruction of RBCs; patients are generally asymptomatic between episodes option A= autoimmune hemolytic anemia= result from extrinsic antibody mediated hemolysis and are associated with positive Coombs test; these anemias often accompany SLE and other autoimmune diseases, Hodgkin and non-Hodgkin lymphomas, mycoplasma infections (cold agglutinins) and infectious mononucleosis option B= hereditary spherocytosis, autosomal dominant defect in RBC structural proteins (spectrin, ankyrin, or protein 4.1), characterized by increased erythrocyte osmotic fragility and MCHC is increased above 36 g/dl option D= include sickle cell anemia, HbC disease, and HbE disease, as well as others; heterozygotes usually show minimal to no clinical signs, while homozygotes present with hemolytic anemia in addition to other disease-specific complications option E= MAHA occurs when there is destruction of RBCs within small vessels due to widespread thrombosis; MAHA is associated with DIC, TTP, and HUS

72 years man -Comes to ED due to severe chest tightness, dyspnea (started 20 mins ago, first time); medical conditions: hypertension, hyperlipidemia, type 2 diabetes, and prostate cancer (takes multiple medications, smoker for 30 years but stopped 10 years ago); chest CT shown -Factors that contributed to patient's current condition? (learn don't answer) a)Atherosclerosis b)Fluid overload c)Hypercoagulability d)Intimal tear e)Noninfectious vasculitis f)Pulmonary bleb rupture g)Pulmonary metastasis

answer: Hypercoagulability -sudden-onset chest tightness and dyspnea, CT scan= saddle pulmonary embolism (PE) straddling the bifurcation of the main pulmonary artery. The main pulmonary artery, usually comparable in diameter to the ascending aorta, is significantly dilated due to the proximal increase in hydrostatic pressure from PE. Saddle PE can result in sudden cardiac death or severe hypotension, but many patients can be hemodynamically stable on presentation -Venous thromboembolism (VTE) (i.e., PE or DVT) results from the Virchow triad of endothelial injury, venous stasis, and hypercoagulable state. Malignancy (e.g., prostate cancer) induces a hypercoagulable state and is a strong risk factor for VTE. Active smoking is also a risk factor as it contributes to endothelial injury and a hypercoagulable state. Age plays a role as well, as older individuals tend to be more sedentary and are more prone to venous stasis option A= primarily affects higher pressure arteries in systemic circulation (e.g., aorta, popliteal arteries, carotid arteries) and coronary arteries option B= causes pulmonary edema, typically appearing as alveolar ground glass opacities on CT scan option D= can lead to aortic dissection, which classically manifests as severe chest pain radiating to the back option E= pulmonary vasculitis (e.g., granulomatosis with polyangiitis) could cause dyspnea and chest tightness but would demonstrate an interstitial pattern on CT scan, sometimes leading to pulmonary hemorrhage option F= rupture of pulmonary blebs (thin-walled, air-filled subpleural structures) leads to pneumothorax option G= for pulmonary metastasis to cause significant dyspnea, multiple lesions would likely be present. There is no evidence of lung masses on this patient's CT scan

36 years woman -2 weeks of fatigue and easy bruising; had an upper respiratory tract infection recently. Exam: no lymphadenopathy or hepatosplenomegaly -Labs: Hb (7.0g/dl), RBCs (1.8mill/mm3), MCV (normal), reticulocytes (0.1%), platelets (88,000), WBCs (2,500), PT (normal), PTT (normal) -Peripheral smear: normocytic, normochromic RBCs; other cells are normal too; biopsy would show? a)Hypercellular marrow filled with numerous blast forms b)Hypercellular marrow with megaloblastic hematopoiesis c)Hypergranular promyelocytes with multiple Auer rods d)Hypocellular marrow filled with fat cells and marrow stroma

answer: Hypocellular marrow filled with fat cells and marrow stroma -Patient has anemia, thrombocytopenia and neutropenia (recent respiratory infection) with profound reticulocytopenia, morphologically normal cell lines on peripheral smear and no splenomegaly on physical exam= consistent with aplastic anemia, which in actuality presents as pancytopenia (not simply anemia) that is characteristically without splenomegaly -Bone marrow biopsy= marked hypocellularity replaced by fat cells and marrow stroma, aspiration typically produces a "dry tap". The absence of splenomegaly is key in diagnosing aplastic anemia, a patient with pancytopenia and splenomegaly is unlikely to have aplastic anemia -Other causes of pancytopenia without splenomegaly: severe vitamin B12 and folic acid deficiency, acute leukemias, and certain forms of MDS. Bone marrow examination is helpful in distinguishing among these conditions option A= can be found in myeloproliferative disorders such as primary myelofibrosis (would present with splenomegaly and fatigue) or CML (would present with leukocytosis), MDS (presents in older patients), and certain leukemias (e.g, AML)- blasts on peripheral smear option B= can be seen in MDS or megaloblastic anemia; MDS in older patients (median age=65 years) and has male predominance; megaloblastic anemia would show macrocytosis option C=Acute promyelocytic leukemia is characterized by presence of Auer rods, and numerous coarse azurophilic granules; characterized by DIC (acquired hypercoagulability and bleeding), which not seen in this patient (given her normal PT and PTT time), immature WBC forms would be seen on smear

22 years woman -Comes to ED due to nosebleed; similar episode yesterday, but bleeding stopped with prolonged local pressure -Easy bruising (past months); liver and spleen are normal, scattered ecchymoses over arms and legs; Labs: Hematocrit (45%), platelet count (9,000), leukocytes (5,500), neutrophils (60%), eosinophils (2%), lymphocytes (32%), monocytes (6%); normal fibrinogen, normal PT -HIV and HCV negative; primary mechanism? a)Bone marrow aplasia b)Bone marrow infiltration by malignant cells c)Disseminated intravascular coagulation d)Immune destruction of platelets e)Platelet sequestration f)Von Willebrand disease

answer: Immune destruction of platelets -Recurrent epistaxis, ecchymoses, marked thrombocytopenia; normal hematocrit, leukocyte count and differential, fibrinogen level, PT time; takes no medications, no hepatosplenomegaly= isolated acquired thrombocytopenia -Common causes of acquired thrombocytopenia include increased platelet consumption, sequestration, and/or destruction. In this patient, primary immune thrombocytopenia purpura (ITP) is the most likely diagnosis as, other than thrombocytopenia (with associated ecchymosis), her physical exam and labs are normal (diagnosis of exclusion) -ITP is characterized by autoimmune destruction of platelets by anti-platelet antibodies, likely IgG autoantibodies against the platelet membrane glycoproteins GpIIb/IIIa. In children, ITP is typically acute and self-limited, whereas it tends to run an insidious and chronic course in adults. A peripheral smear= isolated thrombocytopenia and no other platelet abnormalities would help confirm the diagnosis (megakaryocytes sometimes seen). Treatment involves immunosuppression (corticosteroids). Secondary ITP is sometimes associated with HIV or HCV infection option A= -Recurrent epistaxis, ecchymoses, marked thrombocytopenia; normal hematocrit, leukocyte count and differential, fibrinogen level, PT time; takes no medications, no hepatosplenomegaly= isolated acquired thrombocytopenia -Common causes of acquired thrombocytopenia include increased platelet consumption, sequestration, and/or destruction. In this patient, primary immune thrombocytopenia purpura (ITP) is the most likely diagnosis as, other than thrombocytopenia (with associated ecchymosis), her physical exam and labs are normal (diagnosis of exclusion) -ITP is characterized by autoimmune destruction of platelets by anti-platelet antibodies, likely IgG autoantibodies against the platelet membrane glycoproteins GpIIb/IIIa. In children, ITP is typically acute and self-limited, whereas it tends to run an insidious and chronic course in adults. A peripheral smear= isolated thrombocytopenia and no other platelet abnormalities would help confirm the diagnosis (megakaryocytes sometimes seen). Treatment involves immunosuppression (corticosteroids). Secondary ITP is sometimes associated with HIV or HCV infection option B= would cause pancytopenia and extramedullary hematopoiesis option C= fibrinogen would be decreased and PT increased option E= occurs in disorders that produce splenomegaly, i.e., portal hypertension; abnormal bleeding doesn't occur b/c platelets remain >30,000; in ITP platelets can undergo splenic sequestration, but it is not the primary mechanism option F= platelet count would be normal and bleeding time increased

65 years man -Exertional dyspnea, fatigue; no chest pain, fever or cough -History: mild aortic stenosis, GERD, obesity, hypertension, hypercholesteremia; BMI is high -Mucosal pallor, no jaundice, no lymphadenopathy nor jugular vein distention. Exam: 2/6 systolic ejection murmur at 2nd intercostal area -No hepatosplenomegaly; labs: Hb (7.8g/dl), platelets (98,000), leukocytes (3,800); peripheral smear= macrocytes, hypersegmented neutrophils; cause? a)Impaired DNA synthesis b)Impaired heme production c)Inadequate erythropoietin production d)Replacement of bone marrow with fat cells e)Replacement of bone marrow with fibrosis f)Traumatic hemolysis

answer: Impaired DNA synthesis -Exertional dyspnea, fatigue, conjunctival pallor, systolic ejection murmur= symptomatic anemia. Presence of macrocytosis and hypersegmented neutrophil indicate likely megaloblastic anemia -This condition sometimes arises in older individuals who take antacids for GERD, as gastric acid is required for proper vitamin B12 absorption -Vitamin B12 acts as a cofactor in the recycling of 5-methyl-tetrahydrofolate to tetrahydrofolate (reduced form of folate), which serves as a one-carbon donor in the synthesis of amino acids, purines and thymidine monophosphate. Therefore, folate or vitamin B12 deficiency reduces available purines and thymidine, which impairs DNA synthesis and causes cell cycle arrest (in S phase) or apoptotic cell death -Hematopoietic cells are primarily affected due to their high replication rate. In these cells, nuclear division slows relative to cytoplasmic maturation (nuclear-cytoplasmic desynchrony), which results in megaloblastic changes (e.g., macrocytosis, hypersegmented neutrophils); in addition, increased apoptosis causes anemia and often mild thrombocytopenia/leukopenia option B= iron deficiency anemia; microcytic and hypochromic option C= chronic kidney disease; normocytic anemia option D= aplastic anemia; pancytopenia option E= primary myelofibrosis; teardrop cells and nucleated RBCs on smear option F= aortic stenosis; but would show schistocytes

35 years woman -Sudden-onset numbness in left arm and face; generalized headache, dyspnea on exertion, easy fatigability (several days, no weakness) -History: well-controlled asthma -Light touch sensation decreased in left upper extremity and lower left side of face; strength and reflexes are normal -No skin rash; labs: Hb (8.6g/dl), platelets (24,000/mm3), BUN and creatinine are elevated -PT and PTT are normal; smear= schistocytes; urinalysis= mild proteinuria; cause? a)IgA immune complex deposition b)Impaired cleavage of von Willebrand factor c)Plasma cell proliferation d)Rupture of atheromatous plaque e)Systemic coagulation factor activation

answer: Impaired cleavage of von Willebrand factor -Severe thrombocytopenia and schistocytes on peripheral smear indicates microangiopathic hemolytic anemia (MAHA). The presence of normal coagulation studies indicate there is no systemic activation of the coagulation cascade, which makes DIC unlikely (choice E); therefore, a platelet-activated thrombotic microangiopathy such as TTP or hemolytic uremic syndrome more likely -In this case, the patient has 4 of the 5 classic findings of acquired TTP= 1) severe thrombocytopenia which may cause bleeding or bruising 2) MAHA, which may cause symptomatic anemia (e.g., fatigue, dyspnea on exertion) 3) renal damage, which may cause renal insufficiency (e.g., elevated creatinine) and mild proteinuria 4) neurologic damage, which may cause confusion, headache, and transient focal findings (e.g., numbness, weakness) 5) fever (not present) -Acquired TTP occurs due to formation of an autoantibody inhibitor against ADAMTS13, a vWF-cleaving protease. Circulating vWF normally is released as large vWF multimers that are cleaved to their regular size by ADAMTS13. These vWF multimers support hemostasis by bridging platelets and subendothelial components to sites of vascular injury. In TTP, the large uncleaved vWF multimers are significantly more prothrombotic and result in diffuse microvascular thrombosis option A= Henoch-Schonlein purpura (IgA vasculitis) is characterized by IgA immune complex deposition. Typical manifestations include palpable purpura, renal disease, and arthritis/arthralgia; platelet counts are normal option C= multiple myeloma can cause renal failure, anemia; and Waldenstrom macroglobulinemia can cause neurologic symptoms due to hyperviscosity and autoimmune hemolytic anemia; however, microangiopathic hemolytic anemia would be atypical option D= cerebrovascular accident caused by plaque rupture would cause neurologic symptoms but not other symptoms present here

52 years man -Dizziness, headaches, pruritus after showering; smoker (15 years) -Exam: reddish facial complexion and mild splenomegaly -Labs: Hb (19g/dl), hematocrit (59%), RBCs (7.5 million/uL), platelets (550,000/uL), leukocytes (15,600/uL); findings? a)Decreased circulating plasma volume b)Decreased blood oxygen saturation c)Increased bone marrow sensitivity to growth factors d)Increased red blood cell life span e)Increased serum erythropoietin levels

answer: Increased bone marrow sensitivity to growth factors -Polycythemia vera (PV) is a clonal myeloproliferative disease of pluripotent hematopoietic stem cells. Approximately 95% of patients with PV have a V617F mutation involving JAK2 gene, which is involved in signal transduction. This mutation replaces a valine with phenylalanine at the 617 position, rendering hematopoietic cells more sensitive to growth factors such as EPO and thrombopoietin -PV presents with increased RBC mass, increased plasma volume and low EPO levels. Additional manifestations can include an elevated platelet and/or WBC count, thrombotic events (from blood hyperviscosity), peptic ulceration and aquagenic pruritus (due to histamine release from basophils), and gouty arthritis (from increased cell turnover) -Exam: plethoric, reddened face and splenomegaly; Diagnosis= confirm low EPO levels and bone marrow aspiration cytogenetic studies= JAK2 mutation -Treatment: serial phlebotomy as necessary to keep hematocrit< 45% option A= dehydration or excessive diuresis can also cause elevated hematocrit, mild leukocytosis and thrombocytosis due to low effective circulating volume. However, splenomegaly and symptoms such as pruritus would not be seen option B= hypoxia is a strong stimulus for EPO production; SaO2< 92% (PaO2 <65mmHg) appears to be the threshold for development of physiologic secondary polycythemia. Conditions such as COPD and obstructive sleep apnea can cause it. However, leukocytosis, thrombocytosis and splenomegaly would not be seen option D= wouldn't cause leukocytosis or thrombocytosis option E= EPO-producing tumors (e.g., renal cell carcinoma, hepatocellular carcinoma) can cause secondary polycythemia due to abnormal EPO production. Workup will show elevated EPO levels; however, the combination of multiple elevated cell lines and splenomegaly is not typical

question 2 of 2 -Patient is admitted to hospital and receives opioid analgesics and IV fluids (no evidence of infection); his Hb electrophoresis= sickle cell disease (HbSS) -Labs: high MCV, reticulocytes (1.5%, low given patient's severe anemia) -Cause of patient's macrocytosis? a)Extramedullary erythropoiesis b)Increased folic acid requirement c)Iron overload d)Liver disease e)Myelodysplastic syndrome

answer: Increased folic acid requirement -This patient has macrocytic anemia; severe macrocytosis is usually due to megaloblastic anemia, a subtype of macrocytic anemia caused by impaired DNA synthesis -Cytoplasmic components then accumulate within the slowly dividing erythrocytes, producing cells that are larger than normal (macrocytes) -Patients with sickle cell disease or other hemolytic anemias have increased folic acid requirement due to increased erythrocyte turnover; as such, they are prone to developing relative folic acid deficiency and megaloblastic anemia option A= can occur in SCD, and lead to an elevated reticulocyte count and MCV, as reticulocytes are larger than those released by bone marrow. However, this patient's low reticulocyte index (reticulocyte % corrected for degree of anemia) suggests inadequate response to anemia and lack of erythropoiesis option C= patients with SCD may receive multiple transfusions; transfusion related iron overload can cause severe liver disease, but would not produce a high MCV option D= macrocytosis may occur in liver failure due to increase in circulating phospholipids and cholesterol that adsorb onto erythrocytes, resulting in membrane expansion; However, liver-associated macrocytosis is generally mild (<110) and this patient doesn't have any findings of advanced liver disease option E= macrocytosis may occur in liver failure due to increase in circulating phospholipids and cholesterol that adsorb onto erythrocytes, resulting in membrane expansion; However, liver-associated macrocytosis is generally mild (<110) and this patient doesn't have any findings of advanced liver disease

3 years boy -Jaundice, pallor; adopted and his family history is unknown. Labs: anemia, reticulocytosis, increased indirect bilirubin -Peripheral smear: RBCs without central pallor. Additional findings? a)Decreased lactate dehydrogenase b)Increased haptoglobin c)Increased mean corpuscular hemoglobin concentration d)Increased mean corpuscular volume e)Red blood cell inclusions

answer: Increased mean corpuscular hemoglobin concentration -anemia, reticulocytosis, increased indirect bilirubin= hemolytic anemia. His peripheral smear= RBCs without central pallor (i.e., spherocytes), which suggests acquired or hereditary spherocytosis -Hereditary spherocytosis is caused by defective binding of red cell cytoskeleton to the plasma membrane due to mutations involving ankyrin, band 3, or spectrin proteins; acquired spherocytosis is most often caused by autoimmune hemolytic anemia. -Compared to normal RBCs, spherocytes are smaller and have a more intensely staining cytoplasm due to membrane loss and RBC dehydration. The red cell index most specific for spherocytosis is elevation in MCHC option A= increased in hemolytic anemia option B= decreased in hemolytic anemia option D= normal/low in spherocytosis option E= Howell Jolly bodies (DNA) and Pappenheimer bodies (iron), are occasionally associated with hemolytic anemia; but more commonly associated with splenectomy and sideroblastic anemia, respectively

19 years woman -Exertional dyspnea, intermittent lightheadedness; exam: mucosal pallor -Labs: RBCs (3.5mill/mm3) and Hb (8.5g/dl); smear= hypochromic microcytic RBCs and poikilocytosis -Which of the following present in this patient? a)Decreased transferrin binding capacity b)Increased macrophage iron retention c)Increased serum cytokine levels d)Increased serum erythropoietin levels e)Increased serum hepcidin levels

answer: Increased serum erythropoietin levels -dyspnea, lightheadedness, mucosal pallor= symptomatic anemia -Peripheral smear= hypochromic, microcytic RBCs and poikilocytosis (variety of RBC shapes)= iron deficiency anemia (Young women develop IDA from heavy menstruation) -Patients with severe IDA do not have enough iron to generate adequate hemoglobin for each RBC, which results in low MCHC (hypochromia) and small erythrocyte size (microcytosis) -Interstitial fibroblasts in the peritubular capillaries and PCT of the kidney detect reduced O2 carrying capacity of the blood (mild tissue hypoxia) and increase EPO release to drive generation of new RBCs. However, new RBCs cannot be produced at a level to compensate for the anemia due to insufficient iron; therefore, patients typically have minimal reticulocytes and low RBC counts in peripheral blood option A= binds free iron; increased in IDA, but low iron so decreased transferrin saturation options B, C, and E= increased in anemia of chronic disease

65 years man -Worsening fatigue (4 months), tired with simple chores; 15 pack year (smoker), 2-3 beers daily -Exam: late systolic ejection murmur w/ soft S2; no hepatosplenomegaly; labs: Hb (9g/dl), MCV (93); smear= Rouleaux formation; cause of smear? a)Chronic gastrointestinal blood loss b)Excessive cold agglutinin production c)Increased serum paraproteins d)Mechanical erythrocyte injury e)Nutritional vitamin B12 deficiency

answer: Increased serum paraproteins -smear= Rouleaux formation (stacked RBCs). This occurs due to elevated levels of circulating proteins, which disrupts the repulsive electrostatic charge on RBC surface and causes stacked aggregation. Although rouleaux formation can be seen with inflammatory conditions (e.g., infection, rheumatic disease) that increase acute-phase reactants (e.g., fibrinogen), it is classically linked to lymphoproliferative/plasma cell disorders such as multiple myeloma and Waldenstrom macroglobulinemia, which generate high levels of monoclonal paraprotein (immunoglobulins) -Multiple myeloma is a plasma cell malignancy often associated with normocytic anemia, osteolytic bone lesions, and hypercalcemia due to proliferation of neoplastic cells in the bone marrow. Patients also frequently have renal insufficiency (due to immunoglobulin light chain cast nephropathy), nonspecific symptoms (e.g., fatigue). Diagnosis is made using serum/urine protein electrophoresis (monoclonal M-spike) and bone marrow biopsy option A= leading cause of iron deficiency anemia; results in hypochromic microcytic anemia, sometimes with target cells on smear; rouleaux formation is atypical option B= cold agglutinins are cross-reactive IgM antibodies that form with some infections (particularly mycoplasma pneumoniae) and hematologic malignancies. Cold agglutinins typically cause clumping agglutination (not stacked-coin agglutination) and hemolysis option D= common w/ damaged or artificial heart valves; this patient has cardiac murmur that indicates possible aortic stenosis; but smear would show schistocytes option E= megaloblastic anemia; macrocytes with hypersegmented neutrophils

5 years boy -Looks yellow and has been tired lately; had upper respiratory tract infection symptoms (past 3 days, cough, rhinorrhea, sneezing, sore throat); a lot of kids in his daycare are sick -Pallor, scleral icterus, palpable splenomegaly; Labs: Hb (9.0g/dl), reticulocytes (10.8%), platelets (218,000), leukocytes (7,500); total bilirubin (3mg/dl, direct is normal), ALP, AST and ALT (normal) -Recovers spontaneously after few weeks; peripheral smear= spherocytes a)Glucose-6-phosphate dehydrogenase enzyme deficiency b)Imbalance between alpha and beta globin chain production c)Nuclear maturation defect due to defective DNA synthesis d)Polymerization of hemoglobin within red blood cells e)Red blood cell membrane cytoskeleton abnormalities

answer: Red blood cell membrane cytoskeleton abnormalities -Peripheral smear= spherocytes, 2/3 diameter of normal RBCs, more densely hemoglobinized at periphery, and often lack a zone of central pallor. This patient's presentation is consistent with hereditary spherocytosis (HS), autosomal dominant hemolytic anemia due to RBC membrane defect. The mutation in HS most often affects the plasma-membrane scaffolding proteins spectrin and ankyrin. Without this scaffolding, spherocytes are less deformable than normal RBCs and are prone to sequestration and subsequent accelerated destruction in the spleen -Clinical manifestations include hemolytic anemia, jaundice (indirect hyperbilirubinemia), and splenomegaly (have difficulty passing through cords of Billroth and accumulate in the spleen). Infections can trigger hemolysis and lead to an acute hemolytic crisis, as seen in this patient. Age of diagnosis varies considerably, but many patients have jaundice, fatigue in times of increased hemolysis in setting of viral infections option A= G5PD deficiency anemia follows oxidative stress. Common triggers= drugs (sulfonamide, antimalarial), fava beans, infections (viral hepatitis, pneumonia or typhoid), Smear= bite cells and Heinz bodies option B= thalassemia; peripheral smear= hypochromic microcytes and target cells option C= megaloblastic anemia; vitamin B12/folate deficiency; smear: macrocytes and hypersegmented neutrophils option D= sickle cell anemia; missense mutation in the beta globin chain leads to production of HbS, which polymerizes in deoxygenated states; polymerization leads to RBC membrane injury and sickle cells on smear

32 years woman -3 episodes of deep vein thrombosis (6-year period); history of pulmonary embolism -PTT is normal; remains unchanged when activated protein C is added to her plasma -Cause of patient's problem is? a)Immune b)Nutritional c)Inherited d)Infectious e)Neoplastic

answer: Inherited -Young patient with recurrent DVT, features indicative of a hypercoagulable state -Inherited causes of hypercoagulability must be considered in all patients under age 50 who present with thromboses in absence of any obvious explanation for an acquired prothrombotic state -Patient's plasma is resistant to normally antithrombotic effects of activated protein C; thus, the most likely diagnosis is a mutation in factor V gene, which render factor Va resistant to inactivation by activated protein C -2-15% of Caucasians carry a specific factor V mutation, the Leiden mutation; The factor V Leiden mutation and mutations in the prothrombin gene are the most common inherited causes of hypercoagulability -Activated protein C resistance (factor V Leiden mutation) is detected in approximately 20% (range 12-40%) of patients with abnormal venous thromboses option A= antiphospholipid antibody syndrome is a common cause of hypercoagulability; this condition is defined by presence of antiphospholipid antibodies (lupus anticoagulant and/or anticardiolipin antibodies), plus one or more of the following: venous thromboembolism, arterial thromboembolism or frequent fetal loss. Unlike the patient here, patients with antiphospholipid syndrome typically have prolonged baseline aPTT; lupus anticoagulants are most common cause of aPTT prolongation option B= folic acid deficiency can cause hyperhomocysteinemia, which is a prothrombotic state, however, activated protein C resistance is not seen option D= can affect vascular endothelial cells in a way that favors thrombosis; however, infection and inflammatory mediators do not directly promote activated protein C resistance option E= paraneoplastic syndrome of hypercoagulability may be seen in some patients with cancer, especially patients with adenocarcinomas of the pancreas, colon, or lung; mechanism of cancer-induced hypercoagulability is thought to involve release of procoagulant tumor products. Vascular stasis due to obstruction of blood flow by tumor, patient immobility, hepatic involvement and dysfunction, sepsis and advanced age may contribute toward thrombosis in cancer patients; none of these promotes activated protein C resistance

62 years woman -Severe fatigue, progressive exertional dyspnea (6 months) -Several episodes of food getting stuck in throat, especially when attempting to swallow large chunks of solid food -Exam: mucosal pallor, tongue smooth and red, koilonychia; supplementation with? a)Erythropoietin b)Filgrastim c)Folic acid d)Iron e)Pyridoxine f)Vitamin B12 g)Vitamin C

answer: Iron -Plummer-Vinson syndrome (PVS) is a rare disease characterized by dysphagia, esophageal webs and iron deficiency anemia. Patients develop weakness, fatigue, and dyspnea secondary to anemia; dysphagia develops due to esophageal web formation. Other physical examination findings associated with iron deficiency include koilonychia (spoon-shaped nails) and a shiny red tongue secondary to atrophy of lingual papillae. PVS is treated with iron supplementation, which resolves the symptoms related to iron deficiency anemia and often improves dysphagia option A= used to treat anemia associated with chronic renal failure and chemotherapy option B= granulocyte colony-stimulating factor analog used to treat neutropenia in patients following chemotherapy or bone marrow transplant option C and F= vitamin B12/folic acid deficiency results in megaloblastic anemia option E= is required as a cofactor for the first step in heme synthesis. Deficiency causes sideroblastic anemia, neurologic deficits, and dermatitis option G= vitamin C is sometimes added to improve oral absorption of non-heme iron, but would be ineffective without concurrent iron supplementation

35 years woman -Fatigue, exertional dyspnea; history of SLE (noncompliant with therapy). Patient takes naproxen as needed for joint pain. -Vital signs= normal; exam: mild pallor; Labs: Hb (8.6g/dl), MCV (80), creatinine (normal), iron (decreased), Total iron binding capacity (decreased), LDH (normal) -Underlying cause? a)Immune hemolysis of erythrocytes b)Increased cellular use of folic acid c)Intrinsic factor antibodies d)Iron sequestration in macrophages e)Occult gastrointestinal blood loss

answer: Iron sequestration in macrophages -Untreated SLE has normocytic anemia with low serum iron and low TIBC= anemia of chronic disease. Most cases arise in those with chronic elevation of inflammatory cytokines due to underlying rheumatologic disease (e.g., SLE), chronic infection, or malignancy. Although a number of inflammatory cytokines contribute to the development of ACD, the primary mediator is hepcidin, a small peptide released by the liver in response to inflammation -Hepcidin binds to and inactivates iron channels (ferroportin) on enterocytes and reticuloendothelial macrophages, which results in reduced iron absorption in the gut and increased iron sequestration in the reticuloendothelial system. Because reticuloendothelial macrophages recycle senescent erythrocytes and provide >95% of daily iron for erythrocytosis, sequestration of iron in the reticuloendothelial system dramatically reduces serum iron concentration. This limits the amount of iron available for generation of new RBCs and typically results in normocytic (or slightly microcytic) anemia with low reticulocyte response. In ACD, TIBC is generally normal or slightly reduced due to cytokine-mediated suppression of transferrin option A= patients with SLE can develop immune-mediated hemolysis due to generation of autoantibodies against RBC membrane, however, this typically results in extravascular hemolysis, which increases the serum LDH option B and C= megaloblastic anemia option E= patients with SLE can develop GI bleeding from serositis (or chronic NSAID use), which leads to iron deficiency anemia; associated with microcytic anemia and high TIBC, because levels of transferrin are increased

66 years man -Hypertension and stage IV chronic kidney disease; worsened fatigue, low energy (several months), no chest pain or dyspnea -Normal vitals; S4 heart sound; labs: normocytic, normochromic anemia with low reticulocyte count (iron studies normal); treatment with recombinant glycoprotein hormone is begun, and repeat lab testing weeks later show improvement in Hb level -Pathway of described hormone? a)Adenylate cyclase/cyclic AMP b)Arachidonic acid/phospholipase A2 c)Janus kinase 2/signal transducers and activators of transcription d)Nuclear receptors e)Phosphatidylinositol 3-kinase f)Ras/mitogen-activated protein kinase

answer: Janus kinase 2/signal transducers and activators of transcription -Patients with chronic kidney disease (CKD) often develop normocytic anemia as the GFR declines, due to inability to generate adequate quantities of EPO. EPO is a glycoprotein produced by peritubular fibroblast cells in the renal cortex in response to tissue hypoxia (as seen with anemia). Healthy individuals increase EPO production up to 10,000-fold in response to anemia, but patients with CKD have chronic renal inflammation that damages EPO-producing cells, thereby preventing an adequate EPO response. Therefore, patients with CKD are often treated with recombinant EPO agents (e.g., epoetin, darbepoetin) to supplement EPO production -EPO travels from the kidney through the bloodstream to the bone marrow, where it binds to a surface receptor on erythrocyte colony-forming unit cells (erythrocyte precursors). Binding activates the Janus kinase 2/signal transducers and activators of transcription (JAK2/STAT) signaling pathway, which induces the transcription of proteins that inhibit erythrocyte precursor apoptosis and promote survival. Binding also activates signals that increase the differentiation of erythrocyte precursors into mature erythrocytes. Together, these effects increase the production of RBCs and improve circulating RBC counts. -Activating mutations affecting the JAK2 gene are associated with polycythemia vera, a chronic myeloproliferative disorder marked by the clonal proliferation of RBCs (leading to dramatic elevations in hematocrit)

34 years obese man -Fatigue, daytime sleepiness, occasional headaches. Sleeps in a separate room from wife (she complains of his snoring) -Exam: high BP, soft and non-tender abdomen; liver span= 9cm, spleen not palpable -Labs: Hematocrit (57%), WBCs (9,000), platelets (190,000) -Decreased oxygen delivery to which organ caused increased hematocrit? a)Brain b)Liver c)Spleen d)Bone marrow e)Lungs f)Kidneys

answer: Kidneys -Most likely has obstructive sleep apnea (OSA), disease characterized by episodes of asphyxia during sleep, often up to 500 times per night. In patients with OSA, peritubular cells in renal cortex sense hypoxia and respond by releasing EPO into bloodstream (Deficient EPO production is the reason why many patients with chronic kidney disease develop anemia) -EPO stimulates RBC production by binding to EPO receptors on RBC precursors in bone marrow; the response is responsible for secondary polycythemia which occurs in patients with chronic hypoxia secondary to OSA, COPD, right-to-left shunts and high altitude -The liver is the major site of EPO production in the fetus

64 years man -Worsening fatigue, low back pain exacerbated by movement (1 month) -Exam: mucosal pallor, bony tenderness; radiographs: lytic bone lesions -Labs: Hb (8.8g/dl), creatinine (elevated), serum calcium (elevated); urine dipstick is negative for protein, but quantitative measurement reveals elevated 24-hour urinary protein excretion -Urine electrophoresis detected? a)Gamma heavy chains b)Immunoglobulin A c)Immunoglobulin G d)Immunoglobulin M e)Lambda light chains

answer: Lambda light chains -Multiple myeloma (MM) is a lymphoproliferative malignancy characterized by monoclonal plasma cell proliferation and production of monoclonal paraproteins (e.g., immunoglobulins, chain fragments). MM should be suspected in elderly patients who have any combination of osteolytic lesions, hypercalcemia, anemia or acute kidney injury. Other common symptoms: weight loss, constipation (due to hypercalcemia), fatigue, and recurrent infection -Kidney disease in MM is largely due to light chain cast nephropathy (myeloma kidney). Intact immunoglobulins (e.g., IgA, IgG, IgM) are too large to pass through the glomerulus, but a small amount of free light chains (e.g., kappa, lambda) are freely filtered and reabsorbed in the proximal tubules. However, in MM, the excessive quantities of free light chains exceed the resorptive capacity of the kidney. The filtered light chains then combine with Tamm-Horsfall proteins to form obstructive casts that cause tubular rupture and kidney injury -Monoclonal paraproteins are not detected by urine dipstick (which detects only albumin), but quantitative protein concentrations (spot or 24-hour turbidimetric assay) will be elevated. Urine electrophoresis, immunofixation or free light chain immunoassays can be used for diagnosis -MM can occasionally cause monoclonal immunoglobulin deposition disease, in which intact Ig's or heavy and/or light chains deposit on the kidney basement membranes, resulting in glomerular disease and nephrotic syndrome. In this case, large intact Ig's (and heavy chains) can enter the urine, however, albumin (which also passes through the injured glomerulus) would be detected on urine dipstick (choice A, B, C, D)

65 years man -Fatigue (several months), upper abdominal fullness, early satiety, unintentional weight loss; but no fever, night sweats, cough or rash -Exam: mucosal pallor, no scleral icterus; mild hepatomegaly, spleen tip is palpable close to the midline of abdomen -Labs: pancytopenia; peripheral smear= immature granulocytes, nucleated erythrocytes, teardrop cells -Aspiration is attempted but yields no marrow; histology features of spleen? a)Accumulation of macrophages with fibrillary cytoplasm b)Diffuse neutrophilic infiltration and follicular necrosis c)Dilated sinusoids and fibrous nodules with hemosiderin d)Large islands of hematopoietic progenitor cells e)Numerous noncaseating epithelioid granulomas

answer: Large islands of hematopoietic progenitor cells -Primary myelofibrosis is a chronic myeloproliferative disorder marked by clonal expansion of atypical megakaryocytes. These atypical cells produce TGF-beta, which stimulates fibroblasts in the bone marrow to lay down collagen, leading to subsequent bone marrow fibrosis. Because fibrotic bone marrow is inhospitable to hematopoietic stem and progenitor cells, these cells migrate out of the bone marrow to the liver and spleen, the primary sites of extramedullary hematopoiesis -The proliferation of erythroid, myeloid, and megakaryocytic progenitor cells in the spleen leads to marked expansion of the splenic red pulp and usually results in dramatic splenomegaly. Nucleated red cells and immature granulocytes spill into circulation and can be identified on peripheral smear. Dacrocytes (teardrop cells) are also common due to erythrocyte membrane damage during RBC production inf the spleen or fibrotic marrow. Because extramedullary hematopoiesis is less efficient than medullary hematopoiesis= patients typically have one or more cytopenias -The diagnosis= requires bone marrow evaluation; aspiration usually unsuccessful (dry tap) due to fibrosis. Therefore, biopsy= hypocellular marrow with significant fibrosis and atypical megakaryocytes (minority of primary myelofibrosis have been linked to chemical exposure, e.g., toluene, benzene) option A= Gaucher disease; autosomal recessive lysosomal storage disease; hepatosplenomegaly and pancytopenia due to accumulation of glucocerebroside within the cytoplasm of macrophages (crumpled paper appearance). However, most cases present in childhood option B= acute infections of the spleen (e.g., septic emboli from infective endocarditis) are typically associated with this; Follicular necrosis is also common if the infection is due to group A streptococcus option C= congestive splenomegaly due to portal hypertension (e.g., from liver cirrhosis) is associated with dilated sinusoids, fibrosis of red pulp, and hemosiderin-laden macrophages option E= common in sarcoidosis, some malignancies and some infectious disorders

45 years man -Progressive weakness, fatigue (last year); adopted (doesn't know family history); genetic testing= loss of expression mutation in a gene coding for a protein found on the basolateral surface of hepatocytes and enterocytes; this protein is known to interact with transferrin receptor -Conditions at greatest risk of developing? a)Basal ganglia atrophy b)Fat malabsorption and osteoporosis c)Iron deficiency anemia d)Liver cirrhosis and hepatocellular carcinoma e)Pulmonary emphysema

answer: Liver cirrhosis and hepatocellular carcinoma Primary hemochromatosis (i.e., hereditary hemochromatosis) is most commonly caused by mutations affecting HFE protein. This protein interacts with transferrin receptor to form complex that functions as a sensor of iron stores. Mutations that inactivate the HFE protein cause enterocytes and hepatocytes to detect falsely low iron levels. This increases iron accumulation in the body through 2 mechanisms: -1) Enterocytes respond by increasing apical expression of divalent metal transporter 1 (DMT1), increasing iron absorption from intestinal lumen -2) hepatocytes respond by decreasing hepcidin synthesis, low hepcidin levels result in increased ferroportin expression on basolateral surface of enterocytes; this allows increased iron secretion into circulation, leading to iron overload When body iron levels exceed 20g, patients typically develop the classic triad of micronodular cirrhosis, diabetes mellitus, and skin pigmentation (i.e., bronze diabetes). These patients are at increased risk of hepatocellular carcinoma, CHF, and testicular atrophy/hypogonadism option A= Wilson disease option B= exocrine pancreatic dysfunction and vitamin D decrease option C= complication of blood loss, lack of dietary iron, or inability to absorb iron, i.e., celiac option E= complication of alpha-1 antitrypsin deficiency

66 years man -Back and lower extremity pain (several weeks). Pain feels dull and achy and is worsened by movement, causing him to have difficulty bearing weight and ambulating. No bladder or bowel dysfunction, nor extremity weakness or numbness -Takes metformin for type 2 diabetes; drinks 2 alcoholic beverages daily. Exam: no gross abnormality/deformity of back or extremities. No swelling or redness of the legs. Radiograph of lower extremity= radiolucent bone lesions. Labs: Hb (9.6g/dl), MCV (92), Creatinine (high), calcium (high), glucose (high), total protein (high), albumin (normal); cause of bone changes? a)Decreased urinary calcium excretion b)Increased 1,25-dihydroxyvitamin D production c)Increased secretion of parathyroid hormone d)Local tumor cell-mediated activation of osteoblasts e)Local tumor cell-mediated activation of osteoclasts

answer: Local tumor cell-mediated activation of osteoclasts -Mild renal insufficiency, normocytic anemia, hypercalcemia, and bone pain has the classic manifestations of multiple myeloma (MM) (plasma cell malignancy associated with excessive production of monoclonal Ig (paraprotein)). Myeloma cells replicated in the bone marrow and release cytokines that stimulate osteoclasts and inhibit osteoblasts, which leads to generation of purely osteolytic (radiolucent) bone lesions. -Bone remodeling is tightly regulated by osteocytes (stromal bone marrow cells that are embedded within the lacuna-canalicular network) in conjugation with signals form the bone marrow and bone surface. The RANK/RANKL pathway plays a prominent role. RANK is a transmembrane receptor on the surface of osteoclast precursors, which is activated by RANK-Ligand (RANKL) produced by osteocytes. Activation of RANK leads to differentiation of osteoclast precursors into mature osteoclasts -Myeloma cells stimulate osteoclast maturation by producing RANKL and by destroying osteoprotegrin (OPG), mediated by a secreted proteoglycan. They also uniquely inhibit osteoblast activity by secreting cytokines that block osteoblast function (e.g., IL-3, IL-7, Wnt pathway inhibitors). This tilts the balance of bone remodeling strongly toward osteolytic bone destruction, which results in many of the characteristic findings seen in MM, including bone pain, hypercalcemia, and osteolytic (radiolucent) bone lesions option A= can be seen with thiazide diuretic use and can result in hypercalcemia; this effect is associated with increase in bone density option B and C= can be seen with thiazide diuretic use and can result in hypercalcemia; this effect is associated with increase in bone density option D= Many other tumors can do that; but they would produce sclerotic lesions on radiograph

64 years man -Lump in his neck, long history of using chewing tobacco; exam: ulcerative lesion in left floor of mouth -Firm, palpable, nontender left submandibular mass; biopsy- atypical mitotic figures and keratin pearls; route of spread from floor of mouth to submandibular mass? a)Direct invasion through the floor of the mouth b)Field cancerization of the oral cavity mucosa c)Lymphatic drainage to the submandibular lymph nodes d)Perineural spread through the lingual nerve e)Sublingual vein to the submandibular gland

answer: Lymphatic drainage to the submandibular lymph nodes -Patient with ulcerative lesion in floor of mouth and ipsilateral lymphadenopathy in the submandibular area has a biopsy that shows malignant cells with atypical mitotic figures and keratin pearls, findings which are consistent with head and neck mucosal squamous cell carcinoma (HNSCC) -Most carcinomas, including head and neck SCC, first spread from the primary site of the tumor to regional lymph nodes via the lymphatics. In this patient with a floor of mouth SCC, the spread is to the upper anterior cervical lymph nodes, manifesting as submandibular lymphadenopathy -Assessment of regional lymph node status is important for tumor staging based on the TNM staging system, which evaluates the size and characteristics of the Tumor, regional lymph Node involvement, and distant Metastases. The presence of metastases is indicative of advanced cancer and often confers a poor prognosis option A= can occur in head and neck SCC, and result in extension to the submandibular space; However, it is likely to present as a large, fungating mass with local destruction (e.g., bony invasion) option B= field cancerization refers to widespread, premalignant damage to cells in a large area. It is though to be responsible for the high rate of synchronous tumors in patients with head and neck SCC option D= perineural invasions occurs in many cancers, including head and neck cancer, and conveys a poor prognosis. It may contribute to some symptoms of cancer, including numbness or pain, but is not responsible for disease in the lymph nodes of this patient option E= typically follows venous drainage routes and is common in sarcomas and a few carcinomas (e.g., RCC and HCC)

54 years man -2 months of progressive, generalized weakness, easy fatigability -Abdominal discomfort and early satiety; works as a security advisor and has not traveled recently -Afebrile, normal vitals, exam: pallor, abdominal distention, massive splenomegaly (spleen crossing midline); pancytopenia -No peripheral lymphadenopathy; bone marrow aspiration is attempted, but no marrow can be aspirated; findings? a)Intraerythrocytic ring forms b)Lymphocytes with cytoplasmic projections c)Myeloid cells with azurophilic rodlike granules d)Positive heterophile antibodies e)Ringed sideroblasts

answer: Lymphocytes with cytoplasmic projections -Lymphocytes with cytoplasmic projections are seen in hairy cell leukemia, an indolent B-cell neoplasm diagnosed in middle-aged men that infiltrates the bone marrow and reticuloendothelial system; bone marrow infiltration and cytokine production cause fibrosis and bone marrow failure, resulting in pancytopenia -Bone marrow aspirate is usually unsuccessful (dry tap) -Splenic red pulp infiltration can cause massive splenomegaly (i.e., crossing the midline). Common manifestations: left upper quadrant pain, fatigue, weakness, fever and recurrent infections -Diagnosis= bone marrow biopsy and flow cytometry, which has replaced tartrate resistance acid phosphatase (TRAP) activity testing option A= seen in malaria, no travel history here option C= AML option D= EBV induced infectious mononucleosis; presents in young adults with low grade fever, pharyngitis, cervical lymphadenopathy, mild hepatosplenomegaly may be present option E= abnormal erythrocyte precursors found in myelodysplastic syndrome and sideroblastic anemia, characterized by mitochondrial iron accumulation surrounding the nucleus; patients with MDS have petechiae, weakness, recurrent infections, splenomegaly is uncommon

46 years smoker -3 week history of low-grade fever, weakness, neck swelling -Biopsy: large cell with bilobed nucleus, large nucleoli, abundant cytoplasm ("owl eye's appearance) -Abnormality in which of the following tissues is responsible for patient's symptoms? a)Respiratory epithelium b)Thyroid gland c)Lymphoid tissue d)Myeloid precursors e)Skin epithelium

answer: Lymphoid tissue -biopsy= numerous lymphocytes (small cells with dark, round nuclei and a small rim of cytoplasm)= means biopsy from lymph node -Biopsy= giant binucleated cells= Reed-Sternberg (RS) cell -RS cells are derived from germinal center B-lymphocytes and are neoplastic cells of Hodgkin lymphoma (presence is essential for diagnosis)= often described as having appearance of "owl's eyes" option A= pseudostratified columnar ciliated epithelium that extends down to level of bronchioles; at bronchioles it gradually becomes cuboidal then transitions to a flat, single-celled alveolar lining of type I pneumocytes option B= goiter can cause neck swelling; histologically, Hashimoto thyroiditis has appearance of lymphocytes infiltrating between thyroid follicles option D= RS cells are not associated with leukemia; usually use peripheral blood smear and not biopsy of lymph node option E= both melanoma and squamous cell carcinoma can metastasize from the skin causing lymphadenopathy; but biopsy here is different

60 years African American female -Fatigue, had mitral valve repair with mechanical valve (severe mitral insufficiency) -30 pack year history of smoking; exam: pallor -Labs: Hb (9.0g/dl); reticulocytes (6.8%), platelets (218,000/mm3), WBCs (4,500/mm3) -Serum chemistries are normal; peripheral smear= burr cells, helmet cells; cause? a)Microvascular thrombi b)Cytoskeleton abnormality c)Impaired DNA synthesis d)Gastrointestinal blood loss e)Mutation in beta globin chain f)Complement mediated hemolysis g)Mechanical trauma

answer: Mechanical trauma -RBC fragments, burr cells and helmet cells are consistent with traumatic hemolysis, which can result from either microangiopathic hemolytic anemia or mechanical damage (e.g., prosthetic valve) -Prosthetic valves produce excessive shear and turbulence in the cardiac circulation, causing mechanical trauma to the RBCs -Burr cells are RBCs with short, evenly-spaced projections; they can occur as an artifact or in association with uremia, pyruvate kinase deficiency, microangiopathic hemolytic uremia, or mechanical damage option A= pathophysiologic mechanism for microangiopathic hemolytic anemia, which causes thrombocytopenia, in this patient, normal platelet count option B= pathophysiologic mechanism for microangiopathic hemolytic anemia, which causes thrombocytopenia, in this patient, normal platelet count option C= megaloblastic anemia; macrocytes and hypersegmented neutrophils would be seen in smear option D= hypochromic, microcytic anemia due to iron deficiency option E= beta thalassemia, target cells in smear option F= paroxysmal nocturnal hemoglobinuria is an episodic hemolysis caused by complement mediated mechanism; leukopenia and thrombocytopenia (pancytopenia) are commonly associated

7 years boy -Comes to office for follow-up; week ago= began having episodes of bloody diarrhea that have since resolved -Labs studies= elevated BUN and creatinine; CBC= anemia, thrombocytopenia -Coagulation studies= normal; peripheral blood smear= schistocytes -Most likely cause of anemia? a)Autoimmune hemolytic anemia b)Erythropoietin deficiency c)Hemolytic anemia due to oxidative stress d)Bone marrow fibrosis e)Microangiopathic hemolytic anemia

answer: Microangiopathic hemolytic anemia -Clinical presentation and PBS shown= hemolytic uremic syndrome -Most cases of childhood HUS are preceded by bloody diarrhea, often caused by Shiga-like toxin released from enterohemorrhagic E. coli (particularly strain O157:H7); undercooked ground beef is a common cause of HUS -Smear= schistocytes and few platelets; schistocytes are diagnostic of a traumatic mechanism and indicated either microangiopathic hemolytic anemia (e.g., HUS, TTP, DIC) or mechanical damage (e.g., prosthetic valve, severely calcified valves) -HUS and TTP lie on a spectrum; unlike in DIC, the coagulation system is not activated in HUS-TTP, therefore, coagulation studies (PT and aPTT times) are normal, as seen in this patient option A= is due to an altered immune response and production of antibodies against RBCs; AIHA can occur following infections with mycoplasma pneumonia and EBV but not usually following a diarrheal illness; in addition, thrombocytopenia and renal failure are not associated with it option B= normocytic, normochromic anemia; neither schistocytes nor decreased platelets are present; in addition, erythropoietin deficiency is usually seen in chronic renal failure, this patient has acute renal failure option C= G6PD deficiency, not associated with thrombocytopenia or renal failure; would show bite cells and Heinz bodies option D= Leukoerythroblastic peripheral smears (i.e., nucleated RBCs and immature WBCs) are usually seen when the marrow is replaced with fibrosis (or metastatic cancer); the red cells can be squeezed while passing through fibrous strands and appear on PBS as characteristic teardrop cells

34 years man -Enlarged lymph node (anterior cervical chain; 4cm in diameter; few weeks ago, steadily increasing in size since) -Concerned whether or not he has cancer; biopsy reveals abnormal lymph node architecture and numerous lymphocytes. Which of the following is most consistent with malignancy? a)Lymphocyte pleomorphism b)Abundant mitotic figures within the lymph node c)Nuclear changes in lymphocytes d)Monoclonal T-cell receptor gene rearrangements e)Admixture of several lymphoid cell types within the lymph node

answer: Monoclonal T-cell receptor gene rearrangements -Lymphadenopathy can represent inflammatory changes within the lymph node (reactive hyperplasia) or malignant transformation -Reactive hyperplasia encompasses all benign, reversible enlargement of the lymphoid tissue secondary to an antigenic stimulus. The nodal response to antigenic stimuli is highly variable and can be classified as follicular hyperplasia, sinus hyperplasia, or diffuse hyperplasia -Follicular hyperplasia occurs when follicles increase in size and number, whereas sinus hyperplasia occurs when sinuses enlarge and fill with histiocytes. Diffuse hyperplasia is observed when the nodal architecture is diffusely effaced by sheets of lymphocytes, immunoblasts, and macrophages. When malignant transformation occurs (as in lymphomas), the normal lymph node architecture is distorted or effaced by the proliferation of malignant lymphoid cells, often to a greater extent than that seen with reactive hyperplasia. Malignancy associated effacement of nodal architecture may be follicular or diffuse -Reactive lymphoid hyperplasia is polyclonal in that it consists of a proliferation of many different cell types within the lymph node. For each type of lymphocyte responding to an antigenic stimulus, multiple genetically-distinct cells of that variety undergo limited monoclonal expansion, leading to an overall polyclonal response. Malignant transformation, in contrast, is monoclonal in that it results from the unchecked proliferation of a single genetically unique cell from only one cell line Evaluationformonoclonalityofthelymphocytepopulationisimportantwhenlymphoma is suspected. The clonality of a T-cell population is assessed by molecular methods, such as PCR, that examine the rearrangement of T-cell receptor (TCR) genes. If a single allele for the V region of the TCR predominates in a lymphocytic population, monoclonal proliferation is suspected. The same principle applied when assessing for B-cell clonality option A, B and C= pleomorphism, increased mitoses, and nuclear changes are all commonly seen in lymphocytes of a reactive hyperplastic lymph node. These findings do not automatically classify the specimen as malignant option E= suggestive of a benign process. A predominantly monomorphic cell population is indicative of malignancy and is characteristic of non-Hodgkin lymphoma. Hodgkin's lymphoma does demonstrate an admixture of several lymphoid cell types within lymph nodes; however, this finding does not distinguish Hodgkin's lymphoma from reactive hyperplasia; additionally, genetic analysis of Reed-Sternberg cells often reveals monoclonal characteristics

6 years boy -Brought to ED due to bleeding after dental extraction -History: significant for painful swelling of his knee joints after minor trauma -Aspiration of joints during several occasions: frank blood, diagnosed with hemarthrosis -Hemostasis in this patient can be achieved by administration of? a)Factor XII b)Fibrinogen c)Protein C d)Thrombin e)Urokinase

answer: Thrombin -History of prolonged bleeding following procedures and spontaneous hemorrhages into joints (hemarthrosis) is typical for hemophilia, an X-linked recessive bleeding disorder due to decreased levels of factor VIII (Hemophilia A), or factor IX (hemophilia B) -Factors VIII and IX are components of the intrinsic coagulation pathway and activate factor X; activated factor X then catalyzes conversion of prothrombin to thrombin (part of common pathway). In absence of factors VIII, or IX, activation of factor X and subsequent conversion of prothrombin (factor II) to thrombin doesn't occur -Administration of thrombin, however, will make up for deficiency and lead to blood clotting; in practice, prothrombin complex concentrates (containing factors II, VII, IX and X, which lead to thrombin formation) were used for management of hemophilia B (Thrombogenic risks have limited their use) -In both types of hemophilia, bleeding time and platelet count are normal; PT is also normal; however, aPTT is prolonged (assesses intrinsic pathway); diagnosis is made by assessing plasma levels of factors VIII and IX option A= synthesized by liver and activated by endothelial injury; triggers intrinsic coagulation pathway by activating factor XI; its upstream of hemophilia mutations option B= protein synthesized by liver; thrombin mediates cleavage of fibrinogen to fibrin, the main component of thrombi, therefore, without thrombin, fibrinogen administration alone would not be helpful option C= vitamin K dependent factor synthesized in liver; anticoagulant that degrades factor Va and VIIa option E= thrombolytic agent used for treatment of MI and pulmonary embolism; acts by converting plasminogen to plasmin, which breaks down fibrinogen and fibrin into degradation products; it prevents thrombosis

36 years man -Persistent fever, bleeding gums, sore throat. Peripheral smear= cells with Auer rods and bilobed nuclei -Cells contain structures that would stain positive for? a)Major basic protein b)Myeloperoxidase c)Platelet-derived growth factor d)Tartrate resistant acid phosphatase e)Terminal deoxynucleotidyl transferase

answer: Myeloperoxidase -Peripheral smear shows myeloblasts, which are large myelogenous cells with abundant basophilic cytoplasm and large nuclei that can be folded or bilobed. Myeloblasts often have a number of coarse rod/needle-shaped intracytoplasmic granules (e.g., Auer rods) that stain positive for myeloperoxidase. Auer rods indicate myeloid differentiation -Acute promyelogenous leukemia (APL) is a variant of AML, in which immature cells of myeloid origin are unable to differentiate and mature. They proliferate in the bone marrow and suppress the growth and multiplication of other hematopoietic precursors. The clinical manifestation os APL, such as anemia (fatigue, pallor), thrombocytopenia (petechiae, hemorrhages), and neutropenia (fever, opportunistic infections), result from marrow replacement by leukemic cells. option A= found in eosinophil granules and helps to defend against parasites option C= PDGF receptor mutations play a role in GI stromal tumors option D= hairy cell leukemia stains positively for TRAP; the neoplastic cells are B lymphocyte precursors and are CD20+. Hairy cell leukemia presents with splenomegaly, fatigue and pancytopenia option E= TdT is responsible for adding nucleotides to the V, D, and J regions of the antibody gene for antibody diversity. It is a marker of immature lymphocytes, both B and T cells. Neoplastic cells in ALL are TdT positive; ALL affects mainly children

16 years boy -Fever, malaise, sore throat; gets tired even getting out of bed -Exam: palatal petechiae, cervical lymphadenopathy, splenomegaly -Peripheral blood smear= atypical lymphocytes -Horse erythrocyte agglutinate when exposed to patient's serum -Agent causing patient's disease is associated with? a)Acute myeloid leukemia b)Cervical carcinoma c)Hepatocellular carcinoma d)Kaposi sarcoma e)Mucosa-associated lymphoid tissue tumor f)Multiple myeloma g)Nasopharyngeal carcinoma

answer: Nasopharyngeal carcinoma -Sore throat, fever, palatal petechiae, lymphadenopathy (commonly posterior cervical or auricular), splenomegaly, and atypical (reactive) lymphocytosis are classic signs and symptoms of infectious mononucleosis, a disease common among teenagers and young adults; 90% of cases of IM are caused by EBV, however, CMV, HIV, and toxoplasma are some others that can cause IM -EBV infection induces heterophile antibodies, which react to antigens from animal erythrocytes (sheep in the Paul-Bunnell test and horse in the Monospot test). The heterophil antibody tests are sensitive and specific for EBV-associated IM -EBV replicates primarily in B cells and is also associated with malignancies, including Burkitt lymphoma (especially the endemic African type) and nasopharyngeal carcinoma. Many disorders in immunocompromised patients are caused by EBV, including CNS lymphoma in HIV patients and post-transplant lymphoproliferative disorder in allograft recipients option B= strongly associated with HPV 16,18, 31 and 33 option C= HBV or HCV option D= HIV and HHV-8 option E= gastric adenocarcinoma and MALToma are associated with H. pylori

27 years man -Cough, chest discomfort, and dyspnea on exertion (month ago, gradually worsening), weight loss (2 months); smoker (pack a day) -Chest imaging= large, anterior mediastinal mass; subsequent blood work= elevated beta-hCG and alpha-fetoprotein; diagnosis? a)Benign lymphoma b)Bronchogenic cyst c)Hodgkin lymphoma d)Nonseminomatous germ cell tumor e)Seminoma f)Thymoma

answer: Nonseminomatous germ cell tumor Masses that arise within the mediastinum are typically classified according to their compartment. Within the anterior mediastinum, the most common etiologies are the 4 Ts; thymoma, teratoma (and other germ cell tumors (GCTs)), "terrible" lymphoma, and thyroid neoplasm. This patient with a large, anterior mediastinal mass and elevated beta-hCG and AFP most likely has a germ cell tumor In adults, the anterior mediastinum is the most common location for extragonadal GCTs, which may be either benign (e.g., benign teratoma) or malignant (e.g., seminoma, nonseminomatous GCTs). These occur predominantly in young men and cause chest discomfort and compressive symptoms (e.g., cough, dyspnea) due to mass effect on adjacent structures (e.g., trachea). GCT type can usually be distinguished by differences in serum tumor markers: -Seminomas may cause elevated beta-hCG levels in a minority, but AFP is almost always normal (choice E) -Nonseminomatous GCTs (e.g., yolk sac tumor, choriocarcinoma, embryonal carcinoma) cause elevated AFP and/or beta-hCG in majority (85%) -Teratomas are benign (vs. malignant), do not typically elevated AFP/beta-hCG (choice A) option B= Congenital cysts, arise due to abnormal development of bronchopulmonary foregut, and are an uncommon cause of mediastinal mass; they are typically located in the middle (vs. anterior) mediastinum and do not elevated AFP/beta-hCG option C= lymphomas, including HL, can arise in any of the mediastinal compartments, including anterior mediastinum. They can cause weight loss but classically produce additional symptoms (e.g., fever, night sweats), they do not elevate AFP/beta-hCG option F= common cause of anterior mediastinal mass; they do not elevate beta-hCG/AFP levels and are often associated with paraneoplastic syndromes, e.g., myasthenia gravis

59 years woman -Progressive fatigue, occasional heart palpitations (last 6 months) -Lot of stress due to work problems; no dietary restrictions, eats out at restaurants frequently, drinks 2-3 cans of beer on weekends -Postmenopausal and has not noticed any uterine bleeding, dark stools or bleeding with bowel movements -BMI= 25kg/m2; low hemoglobin; Peripheral blood smear= pale, microcytic erythrocytes -Cause of patient's abnormal lab findings? a)Substance use disorder b)Hematologic malignancy c)Hemolysis d)Liver disease e)Occult blood loss f)Poor nutrition

answer: Occult blood loss -Fatigue and heart palpitations= common in all forms of anemia -PBS indicates= hypochromic, microcytic anemia, which most often arises in setting of iron deficiency -Primary and most dangerous cause of iron deficiency= blood loss (should be excluded first) -Women of childbearing age are commonly iron deficient due to menstruation; men (especially old) or postmenopausal women have no physiologic reason to be iron deficient; should be evaluated for blood loss in the GI tract (e.g., due to malignancy); GI blood loss is often occult, so the lack of dark or bright red stools in this patient should not rule out GI hemorrhage -Iron studies (including ferritin) should be obtained, and patient will likely require endoscopic evaluation option B= associated with normochromic, normocytic anemia; decrease in erythropoiesis seen in these patients results from hypersplenism or tumor replacement of bone marrow mass option C= presents with normochromic, normocytic anemia; spherocytes or schistocytes are often seen on PBS option D= usually normocytic or slightly macrocytic with target cells on the PBS; microcytic anemia occurs in <25% of cases, and most patients have only mild anemia with Hb level of 10-11g/dL option F= normal BMI and no factors predisposing her to malnutrition (e.g., dietary restrictions, chronic alcohol use); normal Western diet provides sufficient daily iron; so, men and postmenopausal women usually have no dietary iron shortage

3 years boy -Oral bleeding (1 hour after tooth extraction); bleeding persisted 30 mins despite application of constant pressure with gauze -Exam: slow oozing blood at site of dental extraction; family history is significant for maternal uncle who suffered from recurrent bleeding into his joints; most likely abnormal in this patient? a)Haptoglobin level b)Hemoglobin level c)Partial thromboplastin time d)Platelet count e)Platelet function analyzer f)Prothrombin time g)Thrombin time

answer: Partial thromboplastin time -Abnormal bleeding following tooth extractions or other surgical procedures can signify a quantitative or qualitative platelet defect, coagulopathy, or abnormal fibrinolysis -This patient's male sex and family history of hemarthrosis suggest an X-linked coagulopathy such as hemophilia A (factor VIII deficiency) or hemophilia B (factor IX deficiency, Christmas disease) -Hemophilia A and B are indistinguishable clinically as both demonstrate similar symptoms and inheritance patterns; both conditions are characterized by isolated prolongation of PTT due to defects in the intrinsic coagulation pathway. PT and TT are normal -When endothelial injury occurs, platelets are activated and form a hemostatic plug first, followed by activation of the coagulation cascade. Patients with hemophilia have normal platelet function and can form a platelet plug, so bleeding after procedures can be delayed rather than immediate, in contrast to patients with platelet disorders. Manifestations include intramuscular hemorrhage, hemarthrosis, and prolonged or delayed bleeding after procedures option E= PFA-100 test monitors the time needed for a patient's blood to form a platelet-based plug in an in vitro environment. Abnormal results are seen in vWD, inherited/acquired platelet dysfunction (e.g., NSAID use), and thrombocytopenia option F= elevated in vitamin K deficiency and warfarin use option G= measures conversion of fibrinogen to fibrin; prolonged in patients with deficient or defective fibrinogen, which can be congenital or acquired; most common cause of acquired dysfibrinogenemia is liver disease

20 years man -Recurrent jaundice, no medical history (no parents), exam: pallor, icterus and mild splenomegaly; no lymphadenopathy or hepatomegaly -Labs: Hb (9g/dl), platelets and WBCs (normal), increased LDH, increased total bilirubin (normal direct bilirubin), direct Coombs (negative) -Patient's RBCs are incubated in hypotonic solution with glycerol= Hb is released (control sample doesn't release Hb); greatest risk of developing? a)Autoimmune disease b)Autoinfarction of the spleen c)Avascular necrosis of the femur d)Hemolysis from consumption of fava beans e)Pigmented gallstones

answer: Pigmented gallstones -Anemia, elevated LDH, indirect hyperbilirubinemia= hemolytic anemia; positive acidified glycerol lysis test= hereditary spherocytosis (HS); spherocytes demonstrate increased osmotic fragility in this test due to decreased surface area/volume ratio -HS is the most common hemolytic anemia due to a RBC membrane defect; mutation in HS most often affects spectrin and ankyrin (plasma membrane scaffolding proteins); patients with HS present with hemolytic anemia, jaundice, and splenomegaly -Pigmented gallstones are a complication of any hemolytic anemia; in chronic hemolysis, increased bilirubin from lysed RBCs precipitates as calcium bilirubinate, forming pigmented stones in gallbladder; patients with HS may also present with aplastic crisis due to parvovirus B19 infection option A= -Anemia, elevated LDH, indirect hyperbilirubinemia= hemolytic anemia; positive acidified glycerol lysis test= hereditary spherocytosis (HS); spherocytes demonstrate increased osmotic fragility in this test due to decreased surface area/volume ratio -HS is the most common hemolytic anemia due to a RBC membrane defect; mutation in HS most often affects spectrin and ankyrin (plasma membrane scaffolding proteins); patients with HS present with hemolytic anemia, jaundice, and splenomegaly -Pigmented gallstones are a complication of any hemolytic anemia; in chronic hemolysis, increased bilirubin from lysed RBCs precipitates as calcium bilirubinate, forming pigmented stones in gallbladder; patients with HS may also present with aplastic crisis due to parvovirus B19 infection option B and C= abnormal adhesion of sickle cell RBCs to endothelium and subsequent obstruction of small blood vessels lead to various injuries, which include splenic Autoinfarction and femoral avascular necrosis option D= G6PD deficiency presents with hemolytic anemia and indirect hyperbilirubinemia usually following oxidative stress; common triggers: sulfonamide or antimalarial drugs, fava beans, and infections; PBS= bite cells and Heinz bodies

57 years man -Fatigue, low energy; intermittent back pain that responds to ibuprofen; Family history: father died of heart attack at 60 years -Fecal occult blood is negative; Labs: Hematocrit (36%), MCV (normal), WBC (7,000), platelets (170,000), sodium (136, low border), potassium (normal), AST and ALT normal; bilirubin normal; and creatinine elevated -Plasma protein electrophoresis= high peak in gamma-globulins; diagnosis? a)Iron deficiency b)Cobalamin deficiency c)Chronic lymphocytic leukemia d)Aplastic anemia e)Plasma cell neoplasm f)Hodgkin lymphoma g)Hypothyroidism

answer: Plasma cell neoplasm -In multiple myeloma, neoplastic B-lymphocytes mature into plasma cells that synthesize abnormal (typically large) amounts of monoclonal Ig or Ig fragments (e.g., light chains). Clinical manifestations of MM include: impaired hematopoiesis leading to normochromic, normocytic anemia and weakness, lytic bone lesions classically affecting the vertebral column and causing back pain and pathologic fractures; hypercalcemia and AL amyloidosis, which contribute to renal dysfunction; Severe amyloidosis can cause cardiac and cutaneous findings -Classic lab abnormalities in MM= erythrocyte rouleaux formation on peripheral smear and Bence-Jones protein in urine -Serum protein electrophoresis (SPEP) is more specific and used to determine if excessive monoclonal Ig's are present in serum; an "M peak" on SPEP indicates the presence of such an Ig. An M peak may also be found in Waldenstrom macroglobulinemia and some lymphomas option A= hypochromic microcytic anemia option B= megaloblastic anemia with neurologic signs option C= may cause normochromic normocytic anemia, but Ig production is depressed not increased option D= uniform depression in all hematologic cell lines option F= B-cell malignancy; characterized by presence of Reed-Sternberg cells on lymph node histology; there may be anemia of chronic disease in patients with this condition option G= may be anemic due to iron deficiency or pernicious anemia

66 years woman -Brought to ED due to left leg pain; 5 days ago patient underwent elective right hip arthroplasty for advanced osteoarthritis (recovering well at an acute rehabilitation facility until she woke up today with severe left calf pain and swelling) -Takes daily subcutaneous, low-molecular weight heparin and as needed acetaminophen since surgery. Surgical incision is healing well; left posterior calf is tender and swollen to the knee -3 days ago, CBC was normal, but platelet count now is 70,000. Peripheral smear shows decreased platelet number but no other abnormalities -Which of the following is the most likely cause of this patient's thrombocytopenia? a)Decreased bone marrow platelet production b)Mechanical destruction of platelets c)Non-immune-mediated platelet clumping d)Platelet removal by splenic macrophages e)Uncontrolled activation of the coagulation cascade

answer: Platelet removal by splenic macrophages -Patient on LMWH develop thrombocytopenia and DVT (calf pain/swelling), raising strong suspicion for heparin-induced thrombocytopenia (HIT) type 2. In HIT type 2, heparin induces a conformational change in platelet factor 4, leading to the formation of a neoantigen. IgG antibodies form against the neoantigen (heparin-platelet factor 4), which results in antibody aggregation on the platelet surface -Patients generally develop manifestations 5-10 days after heparin initiation. Thrombocytopenia is the hallmark feature due to the clearance of antibody-coated platelets by splenic macrophages. Antibody aggregation on the platelet surface also result in wide-spread platelet aggregation, which worsens the thrombocytopenia and significantly increases the risk of venous and arterial thrombosis -Patients with HIT-type 2 should immediately stop heparin and be initiated on a nonheparin anticoagulant (e.g., argatroban) to help prevent/treat thrombosis option A= bone marrow suppression can cause thrombocytopenia, it most often occurs in the setting of malignant invasion of the bone marrow (e.g., leukemia) or chemical toxicity (e.g., chemotherapy, excessive alcohol). Thrombosis is uncommon option B and E= bone marrow suppression can cause thrombocytopenia, it most often occurs in the setting of malignant invasion of the bone marrow (e.g., leukemia) or chemical toxicity (e.g., chemotherapy, excessive alcohol). Thrombosis is uncommon option C= HIT type 1 is a non-immune-mediated condition caused by platelet clumping. Thrombocytopenia is typically mild (nadir 100,000/mm3), and most cases arise within 2 days of heparin administration. It does not cause thrombosis and is not considered clinically significant, therefore, heparin can be continued

28 years woman -Comes to ED due to acute onset abdominal pain, nausea, confusion. Doesn't drink or smoke (they make her feel sick) -Serum lipase and liver function tests are normal; urine is reddish in color and darkens on standing for 24 hours; IV dextrose is administered, and her symptoms improve significantly -Dextrose infusion improved symptoms by affecting which pathway? a)Gluconeogenesis b)Fatty acid synthesis c)Ketone body formation d)Porphyrin synthesis e)Protein catabolism f)Purine degradation

answer: Porphyrin synthesis -The patient most likely has acute intermittent porphyria (AIP), autosomal dominant disorder of the heme synthesis pathway caused by porphobilinogen (PBG) deaminase deficiency. In general, enzyme deficiencies in the early steps of porphyrin synthesis cause neurovisceral symptoms (acute porphyrias), deficiencies in the latter steps (after condensation of PBG to HMB) result in photosensitivity (cutaneous porphyria) -AIP is an acute porphyria that causes nervous system dysfunction due to accumulation of early heme pathway intermediates (PBG and delta-aminolevulinic acid (ALA)). It presents acutely with variable GI and neurologic symptoms, most commonly abdominal pain, vomiting, peripheral neuropathy, and neuropsychiatric derangements. A key feature of acute porphyrias is reddish urine that darkens on exposure to light and air due to oxidation of excess PBG -Treatment and prevention of acute porphyria attacks is directed at inhibiting ALA synthases (rate limiting enzyme of heme synthesis) to reduce formation of the toxic intermediate metabolites. ALA synthase is upregulated by CYP450 inducers (e.g., most antiepileptics, griseofulvin, rifampin) and downregulated by heme and glucose. As such, avoidance of alcohol, smoking and other CYP450 inducing drugs is important for preventing acute attacks. Likewise, IV heme administration and carbohydrate loading (such as with dextrose infusion) are useful for ameliorating acute symptoms

6 years boy -Recurrent nosebleeds; exam: diffuse petechiae -Labs: Hb (9.0g/dl); platelets (20,000/mm3); leukocytes (14,500/mm3) -Bone marrow aspiration= shown -The large cells demonstrate positive immunostaining for terminal deoxynucleotidyltransferase (TdT); they express surface CD19 and CD10 as well; diagnosis? (learn don't answer) a)Precursor B-cell leukemia b)Precursor T-cell leukemia c)Mature B-cell leukemia d)Mature T-cell leukemia e)Hodgkin's lymphoma

answer: Precursor B-cell leukemia -Bone marrow smear shows several lymphoblasts with a high nuclear-to-cytoplasmic ratio, variably condensed nuclear chromatin, and prominent nucleoli; presence of these immature cells excludes choice C, D & E -D/D of this acute ALL includes precursor B-cell leukemia and precursor T-cell leukemia -Precursor B-ALL and precursor T-ALL can be distinguished from each other only by immunophenotyping; lymphoblasts in precursor B-ALL are positive for TdT, CD10 (CD10 also sign of immature B cells), CD19 -In contrast, precursor T-ALL are TdT positive, and variably express CD1a, and T cell markers CD2, CD3, CD4, CD5, CD7, CD8 -ALL is primarily a disease of children (75% occurs in children < 6 years); most patients with precursor B-ALL present with evidence of bone marrow failure; thrombocytopenia and/or anemia and/or neutropenia; the leukocyte count may be normal, decreased or increased option B= 15% of all childhood ALL and is more common in adolescents than younger children; patients have high leukocyte count typically and often a large mediastinal mass; lymphoblasts in precursor T-ALL variably express CD1a and T cell markers, CD2-5, CD7, CD8 Option C= chronic lymphocytic leukemia/small lymphocytic lymphoma, CLL/SLL; is a common neoplasm of small, round, monomorphic B lymphocytes in peripheral blood, bone marrow and lymph nodes; these cells express CD19 (B-cell marker) and coexpress CD5 (T-cell marker) on their surface option D= uncommon neoplasms, derived from mature or post-thymic T cells; findings= more mature lymphocytes with T-cell markers option E= can also involved bone marrow, not only lymph nodes; bone marrow finding= Reed-Sternberg cells

67 years man -Severe fatigue (months), can't eat as he used to, weight loss (6 months); exam: mucosal pallor, hepatomegaly, and massive splenomegaly -Evaluation: gain of function mutation of a non-receptor tyrosine kinase protein in hematopoietic cells, leading to persistent activation of signal transducers and activators of transcription (STAT) proteins. Diagnosis? a)Acute promyelocytic leukemia b)Chronic lymphocytic leukemia c)High-grade non-Hodgkin lymphoma d)Mantle cell lymphoma e)Primary myelofibrosis

answer: Primary myelofibrosis -Chronic myeloproliferative disorders are bone marrow diseases characterized by overproduction of myeloid cells. Primary myelofibrosis is caused by atypical megakaryocytic hyperplasia, which stimulates fibroblast proliferation, resulting in progressive replacement of marrow space by extensive collagen deposition. In the early stages, there is marrow hypercellularity with minimal fibrosis, but as the disease progresses, pancytopenia can result. Hepatomegaly and massive splenomegaly develop because the loss of bone marrow hematopoiesis is compensated by extramedullary hematopoiesis. Peripheral smear= teardrop cells and nucleated RBCs -With the exception of CML, chronic myeloproliferative disorders (especially polycythemia vera) frequently harbor a mutation in the nonreceptor cytoplasmic tyrosine kinase, JAK2; this mutation results in constitutive tyrosine phosphorylation activity, and consequently, in the cytokine-independent activation of the STAT pathway. Once they are activated, STAT proteins translocate to the nucleus and promote transcription. A JAK2 inhibitor (ruxolitinib) has been approved for treatment of primary myelofibrosis. option A= t(15;17) leads to formation of PML:RARA fusion protein; blocks differentiation of myeloid precursors option B= lymphoproliferative disorder involving B lymphocytes; lymphocytosis with "smudge cells" on peripheral smear; the majority of cases exhibit increased expression of the proto-oncogene BCL2, a similar finding in follicular lymphomas option C= severe high-grade NHLs are associated with cytogenetic abnormalities; t(8;14) involves c-myc oncogene, found in Burkitt lymphoma, which is associated with EBV infection and classically has "starry-sky" appearance option D= low grade NHL characterized by t(11;14) leading to cyclin D1 overexpression

72 years woman -Routine follow-up; no symptoms and medical history is insignificant -ECG= atrial fibrillation with no ischemic changes; anticoagulation therapy with warfarin is initiated for stroke prevention -2 days later, the patient is hospitalized with severe skin and subcutaneous fat necrosis. Drug effects on which of the following processes are most likely responsible for skin findings? a)Factor IX synthesis b)Factor VIIa activity c)Factor XIa activity d)Fibrinogen conversion e)Protein C activity f)Prothrombin conversion

answer: Protein C activity -This patient has warfarin-induced skin necrosis, a rare but important complication of warfarin. It is thought to be due to transient hypercoagulable state that can occur during the first few days of warfarin therapy -The overall anticoagulant effect of warfarin is due primarily to its inhibition of the vitamin K dependent gamma-carboxylation of clotting factors II, VII, IX, and X ("vitamin K dependent clotting factors"). However, warfarin also decreases carboxylation of proteins C and S, which normally exert an anticoagulant effect (through proteolysis and deactivation of factors V and VIII). Protein C has a short half-life, so its anticoagulant activity is reduced quickly when warfarin therapy is initiated, by about 50% within the first day. During this time, the vitamin K dependent clotting factors II, IX, and X continue to exert a procoagulant effect as they have longer half-lives (factor VII has a short half-life similar to protein C) -the difference in half-lives translates into a transient hypercoagulable state à 1) decreased protein C (anticoagulant) activity= procoagulant effect 2) persistent clotting factor II, IX, and X activity= procoagulant activity -Thrombosis and clot can interrupt blood flow to the skin and lead to skin necrosis. For this reason, overlapping coadministration of heparin ("heparin bridge") is commonly used when warfarin is initiated. The risk of warfarin-induced skin necrosis is increased in patients with a pre-existing protein C deficiency, as well as those started on a large dose of warfarin

50 years woman -Comes to ED due to large, painful skin lesions. She doesn't remember sustaining trauma. Recently diagnosed with atrial fibrillation and started treatment with warfarin. Noticed lesions about a day and half after starting medication (picture shows skin necrosis) -Cause of skin lesion? a)Allergic drug reaction b)Antithrombin deficiency c)Autoimmune phenomena d)Protein C deficiency e)Vitamin K deficiency

answer: Protein C deficiency -Started on warfarin to prevent thromboembolism secondary to atrial fibrillation. Purpuric lesion likely represents warfarin-induced skin necrosis, rare complication that results from an excessive decrease in protein C activity; patients with underlying protein C deficiency are at increased risk -Protein C and S are innate anticoagulants that are vitamin K dependent, as are procoagulant factors II, VII, IX, and X. Warfarin inhibits epoxide reductase (responsible for regenerating vitamin K), which leads to decreased levels of these proteins. Factor VII and protein C are depleted first due to their shorter half-life. The early loss of protein C activity relative to procoagulant factor activity leads to a transient hypercoagulable state that persists until the other procoagulant factors are sufficiently inhibited. In patients with protein C deficiency, this procoagulant state is exaggerated, promoting microvascular occlusion and hemorrhagic skin necrosis. Underlying protein S deficiency also increases the risk of warfarin-induced skin necrosis, though not to the same extent -Treatment of warfarin-induced skin necrosis includes discontinuing warfarin and administering fresh frozen plasma or protein C concentrate. The condition may be prevented by coadministration of heparin during the first several days of warfarin therapy option A= immune-mediated hypersensitivity reactions that can have variable cutaneous findings ranging from diffuse erythematous eruptions to extensive blistering and exfoliation (e.g., Stevens-Johnson syndrome/toxic epidermal necrolysis) option B= can be inherited (autosomal dominant) or acquired (e.g., cirrhosis, nephrotic syndrome) and presents with venous thromboembolism (e.g., DVT, PE) and resistance to heparin option C= heparin-induced thrombocytopenia is an autoimmune phenomenon that causes arterial and venous thrombosis. It can produce skin findings similar to this patient's but would generally be associated with thrombocytopenia and heparin use option E= vitamin K deficiency most often occurs in patients with fat malabsorption or those taking broad-spectrum antibiotics. The deficiency increases the inhibitory effect of warfarin on all vitamin K dependent proteins, so it does not alter the relative coagulability balance to encourage warfarin-induced skin necrosis

26 years Caucasian female -Single amino acid substitution (glutamine to arginine) near the protein C cleavage site in her coagulation factor V gene product; greatest risk for? a)Petechiae and ecchymoses b)Recurrent hemarthroses c)Renal artery stenosis d)Splenic infarction e)Pulmonary thromboembolism

answer: Pulmonary thromboembolism -Major clinical manifestations of factor V Leiden include deep vein thrombosis (DVT), cerebral vein thrombosis, and recurrent pregnancy loss -Because pulmonary thromboembolism occurs in approximately 50% of all individuals with untreated DVTs, the young woman presented here is at significant risk (some point in life) -Factor V Leiden is the most common cause of inherited thrombophilia, with the heterozygote prevalence of this genetic mutation ranging from 1-9% in Caucasian populations worldwide. Heterozygote carriers of factor V Leiden have 5-10 times the risk of developing a thrombosis, while homozygote carriers have a 50-100 times risk of developing thrombosis -Factor V Leiden causes thrombophilia through 2 pathophysiological mechanisms. In normal hemostasis, activated protein C (APC) restricts clot formation by proteolytically inactivating factors Va and VIIIa. Factor Va Leiden has reduced susceptibility to cleavage by APC (APC resistance). Because factor Va is a cofactor in the conversion of prothrombin to thrombin, persistently circulating factor Va Leiden results in increased thrombin production. Additionally, factor V Leiden is unable to support APC anticoagulant activity. This combination of increased coagulation and decreased anticoagulation produces a hypercoagulable state option A= qualitative and quantitative platelet abnormalities option B= Hemophilia option C= atherosclerosis and fibromuscular disease of the arteries option D= sickle cell anemia, infectious endocarditis, and myeloproliferative disorders

26 years woman -Being treated for glomerulonephritis, photosensitive skin rash, and arthralgias -Rapid plasma reagin (RPR) test= repeatedly positive despite negative Treponema pallidum enzyme immunoassay (TP-EIA) testing -Coagulation studies: bleeding time (normal), aPTT (prolonged), PT (normal), platelets (normal) -Greatest risk for which complication? a)Esophageal dysmotility b)Menorrhagia c)Painful hemarthroses d)Recurrent miscarriages e)Sclerodactyly f)Tabes dorsalis

answer: Recurrent miscarriages -Glomerulonephritis, a photosensitive skin rash, and arthralgia in a young woman= SLE -Prolonged aPTT and false-positive rapid plasma reagin (RPR) test further suggest presence of circulating antiphospholipid antibodies -Although antiphospholipid antibodies cause aPTT prolongation in vitro, they act in vivo to produce a hypercoagulable state due to activation of phospholipid-dependent coagulation pathways; these antibodies can also cause false-positive results on nontreponemal serologic syphilis tests (e.g., RP and VDRL) by reacting with cardiolipin (the dominant antigen used in these test) -Antiphospholipid antibody syndrome (APS) is a disorder characterized by presence of antiphospholipid antibodies in setting of venous or arterial thromboembolism and/or recurrent pregnancy loss; APS may be a primary disorder or occur secondary to other autoimmune diseases (e.g., SLE) option A= scleroderma/mixed connective tissue disease option B and C= hypocoagulable state option E= CREST syndrome/scleroderma option F= manifestation of tertiary syphilis, results in sensory ataxia; ruled out here with treponema testing

10 years boy -Recently immigrates as political refugee. Required several blood product transfusions due to anemia (previously, no records) -Exam: conjunctival pallor, moderate splenomegaly -Labs: Hb (9.4g/dl), platelets (240,000/mm3) -Enzyme assays demonstrate low pyruvate kinase activity; most likely cause of splenomegaly? a)Inflammatory infiltration b)Myeloproliferative disorder c)Passive splenic congestion d)Presence of circulating immune complexes e)Red pulp hyperplasia f)Undegraded metabolite accumulation

answer: Red pulp hyperplasia -Pyruvate kinase is a glycolytic pathway enzyme that convers phosphoenolpyruvate to pyruvate, resulting in the generation of a molecule of ATP. Allosteric stimulation of pyruvate kinase (by fructose 1,6-bisphosphate) stimulates glycolysis. Mature RBCs, which do not contain mitochondria, rely on lactate as the main metabolite for glycolysis -Most of the ATP produced is used for transport of cations against a concentration gradient in the RBC membrane. Therefore, pyruvate kinase deficiency, which results in insufficient ATP production, disrupts this gradient, leading to water and potassium loss, defective maintenance of membrane architecture (echinocyte formation), and hemolysis. -As reticuloendothelial cells in the splenic red pulp are involved in removal of damaged RBCs, their increased activity in the setting of pyruvate kinase deficiency causes them to undergo hyperplasia, resulting in splenomegaly option A= the spleen is important in fighting infectious pathogens in the body; therefore, many acute and chronic infections lead to enlargement of the spleen due to proliferation of lymphoid tissue option B= disorders such as leukemia and lymphoma result in splenomegaly from neoplastic proliferation of lymphoid tissue within the spleen option C= occurs in setting of portal hypertension, spleen vein thrombosis, or congestive heart failure; resultant splenic sinusoid dilation can lead to splenomegaly option D= in infective endocarditis, circulating immune complexes can deposit in the kidney and spleen, contributing to splenomegaly option F= splenomegaly in lysosomal storage diseases Niemann-Pick disease and Gaucher disease is due to accumulation of sphingomyelin and glucocerebrosides, respectively

63 years woman -Dyspnea on exertion and generalized fatigue. Hospitalized 3 times during past year for heart failure exacerbations. Medical conditions: essential hypertension (20 years), MI at age 58, hypercholesteremia -36 pack year (smoker), 10 year alcohol abuse. Exam: tachycardia, distended neck veins, bilateral crackles on lung auscultation, 3rd heart sound on cardiac auscultation, pedal edema -Appropriate therapy is initiated. On 3rd day= hematocrit increases to 50%, up from 44% on admission. An arterial blood gas analysis shows an arterial O2 partial pressure of 70mmHg. A 51Cr-tagged RBC infusion indicated normal red blood cell mass. Cause of polycythemia? a)Polycythemia vera b)Hypoxic erythrocytosis c)Renal disease d)Occult neoplasia e)Relative erythrocytosis

answer: Relative erythrocytosis -Polycythemia is defined as hematocrit level >52% in men and >48% in women. It may be the result of a true increase in RBC mass (absolute erythrocytosis) or decrease in plasma volume (relative erythrocytosis) In patient with erythrocytosis, the following tests help determine cause: 1)Absolute vs. relative polycythemia: Direct measurement of RBC mass is necessary. An increased total RBC mass indicates an absolute polycythemia (normal RBC mass= relative polycythemia). Patient doesn't have increased RBC mass (relative polycythemia more likely due to diuretic therapy for CHF) 2)Primary vs. secondary polycythemia: serum EPO levels can be used to differentiate primary from secondary. Primary polycythemia is associated with low EPO levels and is caused by myeloproliferative disorder (i.e., polycythemia vera). Secondary polycythemia is characterized by increased EPO levels due to chronic hypoxia, smoking, COPD or kidney neoplasm. 3)Hypoxic vs. other causes of secondary polycythemia: measurement of arterial oxygen saturation is important to exclude hypoxemia as a cause of polycythemia. SaO2 <92% (PaO2 <65mmHg) can cause secondary polycythemia. option A= would have increased RBC mass option B= would have SaO2 <92% (PaO2 <65mmHg) Option C= would reveal increased RBC mass and increase in EPO levels; in chronic renal disease= decreased EPO production and anemia option D= would reveal RBC mass and increase in EPO level

32 years man -Progressive fatigue, easy bruising, and recurring episodes of gum bleeding -Exam: several ecchymoses in his lower extremities -Labs: Hb (7.8g/dl), platelets (65,000/mm3), WBCs (3,000/mm3); prolonged PT and aPTT; decreased fibrinogen and increased D-dimer -Bone marrow biopsy and fluorescence in situ hybridization studies reveal a balanced translocation between long arms of chromosomes 15 and 17; abnormal protein? a)Epidermal growth factor receptor b)GTP-binding protein c)Platelet-derived growth factor receptor d)Retinoblastoma gene product e)Retinoic acid receptor

answer: Retinoid acid receptor -AML, characterized by failure of immature myeloid precursors (myeloblasts) to differentiate into mature granulocytes, is divided into 8 types (M0 through M7). Acute promyelocytic leukemia (APML), the M3 variant of AML is associated with t(15;17) cytogenic translocation involving promyelocytic leukemia (PML gen) on chromosome 15 and the retinoid acid receptor alpha (RARA) gene on chromosome 17 -Fusion of these 2 genes= PML/RARA, chimeric gene (seen in FISH) that codes for an abnormal retinoid acid receptor, which then inhibits myeloblast differentiation. Abnormal promyelocytes and Auer rods are seen on smear -Clinical manifestations of AML: anemia (pallor, fatigue), thrombocytopenia (petechiae, hemorrhages), and neutropenia (fever, opportunistic infections), result from marrow replacement by leukemic cells -As seen in this patient, APML is associated with DIC, which is characterized by activation of coagulation cascade; labs: thrombocytopenia, elevated D-dimer, prolonged PT and aPTT and low fibrinogen due to compensation -All trans retinoid acid is used for treatment of APML option A= associated with certain lung (ERBB1), breast (ERBB2, also known as HER2/neu), ovarian and gastric tumors option B= involved in cellular signal transduction option C= plays a role in pathogenesis of several cancer, including ovarian cancers option D= predisposes to retinoblastoma and osteosarcoma

43 years Caucasian female -Reduced energy and fatigue; found to have hypochromic, microcytic anemia -History: stable angina treated with metoprolol and aspirin -Iron supplementation is prescribed; weeks later, peripheral smear shows numerous enlarged RBCs that appear blue on Wright-Giemsa stain; bluish color of these RBCs is explained by presence of? a)Hemoglobin precipitates b)Nuclear membrane c)Ribosomal RNA d)Histones e)Mitochondria f)Golgi apparatus

answer: Ribosomal RNA -Patient has hypochromic, microcytic anemia= iron deficiency anemia -Iron-deficient individuals on replacement therapy should experience hemoglobin level increases of approximately 2g/dl per week for the first 3 weeks -This increase in Hb results from enhanced erythropoiesis and accelerated release of both mature RBCs and reticulocytes into the bloodstream -Reticulocyte is an immature RBC that is slightly larger and bluer than a mature RBC; it lacks a cell nucleus but retains a basophilic reticular (mesh-like) network of residual ribosomal RNA. The ribosomal RNA appears blue microscopically after the application of the Wright-Giemsa stain or methylene blue -After spending a day or so in the bloodstream, the reticulocytes are transformed into mature RBCs that have a lifespan of approximately 120 days

1 year African American boy -Brought to ED with 3 hour history of severe swelling and tenderness of the hands and feet (cries when hands or feet are touched). No history of recent illness. Attends daycare, but parents are unsure about sick contacts. Family history= older brother who died of pneumococcal sepsis at age of 6. Exam: bilateral, severe swelling of hands and feet -Abnormal in this patient? a)Left ventricular systolic function b)Serum albumin c)Serum C4 complement fraction d)Serum haptoglobin e)Systemic venous pressure f)Urine cortisol excretion

answer: Serum haptoglobin Sickle cell disease (SCD) is the most common autosomal recessive disorder in African Americans. Its manifestations include: -Hemolysis: repeated sickling of RBCs leads to permanent deformation and premature RBC destruction by macrophages and mechanical stress. This causes intra and extravascular hemolysis, resulting in increased indirect bilirubin and LDH and decreased haptoglobin. Haptoglobin binds circulating Hb and reduces renal excretion of free Hb (preventing tubular injury) -Vasoocclusive symptoms: patients with SCD may experience pain from hypoxic tissue injury and infarctions due to obstruction of small vessels by sickled cells. Microvascular occlusion typically involves the bone marrow, periosteum and deep muscles but can occur in any organ. This patient is presenting with dactylitis (hand-foot syndrome), which results from small infarctions in the bones of the extremities. These infarctions cause swelling, tenderness, and warmth. Dactylitis is common in patients with SCD during the first few years of life as the affected bones still contain hematopoietic bone marrow. In older children and adults, vascular occlusion leads to pain crises, acute chest syndrome, leg ulceration, priapism, autosplenectomy, and stroke -Infections: patients with SCD are predisposed to infections with encapsulated organisms (e.g., Streptococcus pneumoniae) because repeated splenic infarcts causes functional asplenia option A and E= chronic anemia in SCD can result in cardiomegaly due to chronically increased cardiac output (systolic function is largely preserved due to increase in systolic volume). However, this typically does not occur until after infancy. Older patients can develop CHF due to ischemic cardiomyopathy and present with decreased CO and increased systemic venous pressure option C= hereditary angioedema is a rare autosomal dominant disorder that is associated with painless episodes of swelling involving the face, lips, larynx and extremities. It is caused by C1-inhibitor deficiency, not Vasoocclusion

8 years boy -One week history of fever, throat pain= brought to ED due to severe dyspnea, tachypnea, and inspiratory stridor -Also experienced worsening dysphagia with solid foods (last 2 weeks) -Labs: immature hematopoietic cells (blasts) in peripheral smear; neoplastic cells causing this patient's condition normally give rise to? a)B-lymphocytes b)T-lymphocytes c)Monocytes d)Erythrocytes e)Platelets

answer: T-lymphocytes -Blast cells in peripheral smear- leukemia. Most common pediatric malignancy is ALL; the neoplastic cells of ALL arise from lymphocytic precursors that are either pre-B or pre-T lineage -Clinical manifestations of ALL= fever, fatigue, pallor, petechiae, and bleeding; leukemic spread can cause lymphadenopathy, hepatosplenomegaly and bone pain -T-cell ALL is more likely to present with a large anterior mediastinal mass that can compress great vessels, causing superior vena cava syndrome -The mediastinal mass can also compress the esophagus causing dysphagia, while compression of the trachea may lead to dyspnea and stridor. This patient's clinical presentation is therefore most suggestive of T-cell ALL, which most commonly affects males in late childhood through young adulthood -Because pre-B and pre-T lymphoblasts can look identical on peripheral smear, immunohistochemical staining is used to differentiate b/w the two. Both lineages contain TdT (antigen of lymphocyte precursors). Cell surface markers for pre-B lymphoblasts include CD10+, CD19+, CD20+; while pre-T lymphoblasts express CD2+, CD3+, CD4+, CD5+, CD7+, CD8+ option A= B-cell ALL is responsible for 70-805 of all cases of ALL; whereas T-cell ALL accounts for 15-17% of all cases; B-cell ALL manifests with fever, malaise, bleeding, bone pain, and hepatosplenomegaly. Generally, B-cell ALL is not associated with symptoms of mediastinal compression option C= AML subtypes M4 and M5 arise from monocytic precursors. This is rarely seen in children and usually doesn't present as mediastinal mass, though extramedullary disease is common option D= rare AML, M6 subtype arises from erythroid precursors and typically presents in elderly option E= rare AML M7 subtype arises from primitive megakaryoblasts and is associated with t(1;22) and Down syndrome in children

2 weeks girl -Routine visit; born by normal spontaneous vaginal delivery at 39 weeks of gestation; mother is breastfeeding exclusively, and infant has regained her birth weight -Newborn screening (electrophoresis)= HbF (70%); HbA (20%); HbS (10%) -Mother has sickle cell trait, and maternal cousin has sickle cell anemia -Exam: well-appearing infant with no pallor or splenomegaly -Most likely true about patient? a)Life-expectancy will be shorter than average b)Mean corpuscular volume will be decreased c)Reticulocyte count will be elevated d)She has relative protection from Plasmodium falciparum e)She will likely develop pain crises

answer: She has relative protection from Plasmodium falciparum -Patient's hemoglobin electrophoresis= consistent with sickle cell trait; At birth, infants who are heterozygotes for sickle cell trait typically have greatest amount of fetal hemoglobin (HbF), followed by hemoglobin A (HbA), and smallest amount of hemoglobin S (HbS) -HbA continues to be higher than HbS throughout the lifetime of these patients as HbF levels naturally decline, offering protection from sickle cell anemia, aplastic crises, and splenic sequestration -Patients with sickle cell trait=usually asymptomatic with normal hemoglobin level, reticulocyte count, and RBC indices. However, they may develop hematuria, priapism, and increased incidence of urinary tract infections; splenic infarctions at high altitudes has also been reported -Patients with sickle cell trait have relative protection from plasmodium falciparum (Malaria), resulting in lower rates of severe malaria and hospitalization than seen in the general population (mechanism: sickling of parasitized sickle cell trait RBCs and accelerated removal of these cells by the splenic monocyte-macrophage system); these patients are not immune to malaria (should still receive prophylaxis) option A= life expectancy is normal in sickle cell trait; patients with sickle cell disease have shorter life expectancy due to significant complications, e.g., acute chest syndrome, infection from encapsulated organisms option B and C= sickle cell trait= normal RBC indices and reticulocyte counts; sickle cell anemia= increased reticulocytes, but normal MCV still option E= sickle cell trait= HbA predominates over HbS so protected against painful crises; vaso-occlusive pain crises that develop in patients with sickle cell anemia are though to occur when HbS polymerizes and causes the RBCs to assume a sickle shape, typically in response to a trigger, e.g., cold weather, dehydration

14 years girl -Sickle cell disease; admitted to hospital due to fever (2 days) -High pulse and temp.; labs: Hb (10g/dl), MCV (86), reticulocytes (2%), platelets (240,000), WBCs (14,000) -Cultures obtained (pharmacotherapy started); peripheral smear- sickle cells and Howell-jolly bodies; Howell-Jolly bodies due to which processes? a)Bone marrow suppression b)Intracellular parasitic infection c)Leukemic infiltration d)Splenic dysfunction e)Toxic granulations

answer: Splenic dysfunction -Peripheral smear= Howell-Jolly body, a dark-purple remnant of RBC nucleus that is normally removed by the spleen -In patients with asplenia, which can be anatomic (e.g., splenectomy) or function (e.g., sickle cell disease). Howell-Jolly bodies are not removed from circulation and are therefore present on peripheral smear -This patient also has sickle-shaped RBCs and polychromasia (indicative of reticulocytosis) on peripheral smear, consistent with her underlying SCD -Point mutation in the beta-globin gene causes sickling of RBCs and classic manifestations of SCD, including hemolytic anemia, and vasoocclusive episodes. Recurrent infection from the poorly deformable RBCs in the splenic vasculature leads to progressive splenic dysfunction with functional asplenia by age 5 in most patients -Because splenic macrophages are critical in the immune response against encapsulated bacteria (e.g., streptococcus pneumoniae, H. influenzae, Neisseria meningitidis), patients with SCD are at increased risk of potentially fatal infection. Therefore, fever (with or without leukocytosis) in the setting of SCD warrants hospitalizations and empiric antibiotic therapy option A= can occur in patients with SCD and parvovirus (e.g., aplastic crisis). Expected lab findings include acute anemia and absent reticulocytosis option B= peripheral smear with intraerythrocytic parasites include trophozoites (ring forms) in malaria and babesiosis option C= smear would show circulating blasts, immature cells with large nucleus and scant cytoplasm; in addition, leukocytosis and thrombocytopenia are usually seen option E= are purple cytoplasmic granules in the periphery of neutrophils (not RBCs) that are typically seen with infection

3 years boy -Brought to ED due to fevers; 3 weeks ago, patient developed fever, cough and runny nose (children in his daycare have had viral upper respiratory infections). Symptoms had briefly resolved 10 days ago, but fever recurred last week after patient returned to day care -Since then, he has been home due to fever occurring almost daily despite acetaminophen administration. His temperature is high; exam: enlarged lymph nodes in neck region. Lymphadenopathy at which of the following locations is most concerning for malignancy rather than infection? a)Anterior cervical b)Posterior cervical c)Submandibular d)Submental e)Supraclavicular

answer: Supraclavicular Lymph node enlargement (lymphadenopathy) of the head and neck is common in children due to frequent exposure to infections (e.g., pharyngitis, upper respiratory infections). The lymphatic system drains bacteria and cellular debris through local lymph nodes, which filter foreign material and contain lymphocytes to fight infection. Lymph then drains to the thoracic and right lymphatic ducts, which empty into the subclavian veins Although lymphadenopathy usually occurs secondary to the proliferation of inflammatory cells in response to infection, it is less commonly due to a clonal expansion of malignant cells. Malignancy should be suspected when lymph node enlargement is associated with any oof the following features: -Generalized or supraclavicular location -Firm, immobile, nontender quality ->2 cm or increasing size -Systemic features (e.g., weight loss, night sweats) The supraclavicular nodes drain the chest (i.e., pulmonary system, mediastinum) and abdomen (i.e., GIT and genitourinary tracts). Therefore, enlargement of these lymph nodes is unlikely to be caused by common childhood infections and always warrants further workup (CBC, ultrasound..) option A and B= common with upper respiratory infections because these nodes drain the neck and oropharynx option C and D= the submandibular nodes drain cheeks, lips, gums, and part of the tongue; the submental nodes drain the floor of the mouth and part of lower lip; these nodes are commonly enlarged with infections affecting the oral cavity, including dental infections and common viral infections, e.g., coxsackievirus

26 years woman -History of immune thrombocytopenia that failed to improve with glucocorticoid therapy. Underwent splenectomy 6 months ago, which resulted in improved platelet counts. She received vaccinations after surgery and feels well. -Exam: normal; smear= target cells -Increase in which of the following features causes these cells? a)Aggregated iron deposits b)Clusters of remnant DNA c)Denatured hemoglobin d)Fragility to osmotic stress e)Precipitated ribosomes f)Surface area-to-volume ratio

answer: Surface area-to-volume ratio Target cells are RBCs with a dark center, a surrounding halo of pallor, and a dark peripheral ring. They form in RBCs with an excessive surface area-to-volume ratio, which results in redundant RBC membrane folding upon itself (thereby creating appearance of a target). Target cells occur in the following conditions: -Reduced RBC volume from deficient Hb synthesis (e.g., thalassemia, iron deficiency) or structural mutations to hemoglobin (sickle cell) -Excessive RBC membrane due to greater cholesterol to phospholipid ratios (e.g., obstructive liver disease) or splenectomy Macrophages in the red pulp of the spleen remove excess membrane from red cells, a process called "splenic conditioning". Patients who undergo splenectomy are unable to prune erythrocytes and generally develop target cells a few weeks following the procedure. However, over time, macrophages in nonsplenic tissue (e.g,. Liver) take up the task of pruning RBCs, which typically eliminates (or diminishes) the presence of target cells in circulation. option A= Pappenheimer bodies; often seen in sideroblastic anemia option B= Howell-Jolly bodies; hyposplenism of asplenism option C= Heinz bodies; in patients with G6PD deficiency or thalassemia option D= spherocytes have low surface area-to-volume ratios and more susceptible to osmotic stress; seen in hemolytic anemia and hereditary spherocytosis option E= Basophilic stippling; thalassemia, alcohol use disorder, and lead/heavy metal poisoning

35 years woman -Weakness, fatigue, pallor. She denies heavy menses or melena. Exam: conjunctival pallor -Labs: Hb (7.2g/dl), RBCs (1.8mill/uL); MCV (90), reticulocytes (0.1%), platelets (280,000), WBCs (6,700); iron and vitamin B12/folic acid levels= normal -Bone marrow biopsy= absence of erythroid precursors but preserved myeloid and megakaryocytic elements; Further workup would reveal? a)Hepatocellular carcinoma b)Renal cell carcinoma c)Thymic tumor d)Cerebellar hemangioblastoma e)Uterine fibroid

answer: Thymic tumor -Pallor, fatigue= anemia, confirmed by low erythrocyte count, low Hb and low percentage of reticulocytes (deceased RBC production). Importantly, her WBC and platelets counts are normal, and their bone marrow precursors appear normal. Most likely diagnosis= pure red cell aplasia (PRCA), a rare form of marrow failure characterized by severe hypoplasia of marrow erythroid elements in setting of normal granulopoiesis and thrombopoiesis -Pathogenesis of PRCA often involves inhibition of erythropoietic precursors and progenitors by IgG autoantibodies or cytotoxic T lymphocytes. It has been associated with immune system diseases such as thymomas and lymphocytic leukemias. When a thymoma is present, removal can occasionally cure PRCA. Thus, all patients with PRCA should undergo a chest CT scan. -PRCA can also result from parvovirus B19 infection. This virus preferentially attacks and destroys proerythroblasts. Recent parvovirus infection can be confirmed via the detection of anti-B19 IgM antibodies in serum option A= can cause anemia of chronic disease, in which total iron binding capacity (TIBC) is low; however, this patient's iron studies are within normal limits; HCC also secrete erythropoietin, although polycythemia is less common option B= RCC can cause both anemia of chronic disease and iron deficiency anemia due to chronic hematuria; some renal cell tumors can produce EPO and cause polycythemia. RCC is also rare in younger patients option D= can produce EPO and are often associated with polycythemia option E= can cause bleeding and may result in microcytic hypochromic anemia with iron studies showing low iron and increased TIBC; they can also produce EPO and cause polycythemia

8 years boy -Mass in right mandible (few months ago, rapidly growing); no fever, chills, cough or weight loss; family recently immigrated from East Africa -Exam: large tumor on right mandible with palpable regional lymphadenopathy; biopsy= diffuse infiltrate of lymphoid cells with numerous mitotic figures; interspersed macrophages with clear cytoplasm are seen -Lymphoid cells have chromosomal translocation that alters? a)Apoptotic inhibition b)DNA repair c)Regulation of G1 to S-phase transition d)Transcription activation e)Tyrosine kinase upregulation

answer: Transcription activation -Likely has endemic African form of Burkitt lymphoma, which primarily affects children and typically presents as a maxillary or mandibular mass. This type of lymphoma is strongly associated with EBV infection -Histology: Burkitt lymphoma consists of monomorphic, intermediate-sized lymphoid cells with round nuclei, multiple nucleoli, and basophilic cytoplasm. A high mitotic index and high cell death rate are typically seen. Benign macrophages that phagocytize the resulting cellular debris ("tangible body macrophages") are distributed throughout the malignant tissue. The clear cytoplasm of macrophages contributes to characteristic "starry sky" appearance of BL -Most BL's demonstrate a translocation between c-Myc oncogene (long arm of chromosome 8) and Ig heavy chain region (chromosome 14)= t(8;14)= this results in Myc overexpression due to highly active Ig promoter in B cells; the product of c-myc is a nuclear phosphoprotein that functions as a transcription activator, stimulating cell growth and proliferation option A= follicular lymphoma; translocation b/w long arm of chromosome 18 near BCL2 gene and Ig heavy chain gene, chromosome 14= t(14;18)= overexpression of apoptosis inhibitor protein BCL2 option B= mutations of DNA repair enzymes (e.g., BRCA1 and 2) are associated with breast cancer, ovarian cancer, Lynch syndrome, xeroderma pigmentosum, Fanconi anemia, and others option C= translocation between cyclin D1 on chromosome 11 and Ig heavy chain locus on chromosome 14= t(11;14) is characteristic of mantle cell lymphoma=overproduction of cyclin D1= promoter of G1 to S phase transition in cell cycle option E= t(9;22)= Philadelphia chromosome= BCR-ABL fusion= encodes protein with increased tyrosine kinase activity that inhibits apoptosis while promoting mitogenesis= CML

12 years African American boy -Brought to ED with high fever, chest pains, and dyspnea. Past medical history: two prior hospitalizations for abdominal pain, which resolved with analgesics and hydration -Evaluation: Hematocrit (23%), reticulocytes (9%); patient dies several hours after admission; autopsy= spleen is firm and brown; finding most likely related to? a)Work hypertrophy b)Follicular hyperplasia c)Vascular occlusion d)Pressure atrophy e)Dysplasia f)Lipid accumulation

answer: Vascular occlusion -African American with anemia (w/ reticulocytosis) and recurrent episodes of abdominal pain that resolve with hydration= sickle cell anemia -Today this patient has presented with symptoms of acute chest syndrome (ACS), which is vaso-occlusive crisis localized to the pulmonary vasculature that can occur in patients with sickle cell anemia. ACS is commonly precipitated by pulmonary infarction. -In patients with homozygous hemoglobin S disease, vaso-occlusive crises can also cause splenic infarctions. Repeated infarctions over time produce a spleen that is shrunken, discolored and fibrotic. By the time they reach childhood, most patients with sickle cell anemia have undergone "autosplenectomy" as a result of these infarcts and are left with only a small, scarred splenic remnant. The spleen may demonstrate brownish discoloration (hemosiderosis) due to extensive ingestion of sickled RBCs by splenic macrophages (Extravascular hemolysis) option A and B= hyperplasia and hypertrophy of normal splenic elements (especially macrophages and lymphoid cells) found in systemic infections and other diseases option D= of splenic parenchyma is seen when there is a tumor infiltrating it option F= in the spleen= lysosomal storage disorders like Gaucher's disease

70 years female -Easy fatigability, exertional dyspnea, weight loss, complains of frequent falls -Exam: symmetrically decreased vibratory sensation to lower extremities -Hb (7.8g/dl), peripheral smear= hypersegmented neutrophils; best treatment? a)Iron preparations b)Vitamin B12 c)Pyridoxine d)Vitamin C e)Folic acid f)Erythropoietin g)Filgrastim h)Interleukin-2 i)Antithymocyte globulin

answer: Vitamin B12 -Deficiencies in folic acid and vitamin B12= megaloblastic anemia; both vitamins are required for DNA synthesis in erythropoiesis -When there is deficiency of either, cell division is delayed, though the cytoplasm develops normally; thus, the cells enlarge (megaloblasts) but do not divide -Bone marrow is hypercellular in megaloblastic anemia, but the megaloblastic precursor cells are rapidly destroyed in the bone marrow and few differentiated RBCs are released into circulation; in severe megaloblastic anemia, 90% of erythroid precursors are destroyed without ever entering circulation -Vitamin B12 and folic acid deficiencies cause similar hematological pictures. However, neurological dysfunction is only seen in patients with vitamin B12 deficiency. This is because B12 deficiency also causes axonal demyelination and degeneration. In late disease, the neurological dysfunction may be irreversible -The main sites of neurological involvement include the peripheral nerves, spinal cord (posterior and lateral columns), and cerebrum. Decreased vibratory and position sense are early signs of vitamin B12 deficiency. Patients experience ataxia and recurrent falls because of compromised proprioception -Importantly, neurologic abnormalities may occur in vitamin B12 deficiency in absence of frank anemia. Treating megaloblastic anemia due to vitamin B12 deficiency with folate alone could worsen the neurologic symptoms -An RBC MCV of more than 100fl is suggestive of megaloblastic anemia. However, RBC macrocytosis can also occur in liver disease, hypothyroidism, and alcohol-induced liver disease. MCVs greater than 110fl are typically seen only with vitamin B12 and folic acid anemias. On peripheral smear, there is macrocytosis, hypersegmented neutrophils, large bizarrely shaped platelets; presence of one single neutrophils with more than 6 lobes should raise suspicion.

9 years girl -Brought to ED due to prolonged epistaxis; she picked her nose immediately before bleeding started (epistaxis persisted for 20 mins despite constant compression) -Frequent nosebleeds that last >10 mins; family history: grandfather who had unspecified bleeding disorder -Labs: hematocrit (43%), bleeding time (prolonged), aPTT (prolonged), PT (normal), thrombin time (normal), D-dimer (normal); diagnosis? a)Disseminated intravascular coagulopathy b)Dysfibrinogenemia c)Factor XIII deficiency d)Hemophilia A e)Hemophilia B f)Vitamin K deficiency g)Von Willebrand disease

answer: Von Willebrand disease -Normal PT and TT times and prolonged aPTT indicating defect in intrinsic pathway (coagulation factors VIII, IX, XI, or XII). Bleeding time is a test of platelet function and is prolonged by qualitative and quantitative platelet defects. The term "bleeding time" refers to this particular test and not the duration of bleeding, which can be prolonged from other coagulopathies -vWD will cause both prolonged aPTT and bleeding time. vWF is produced by endothelial cells and megakaryocytes and functions as a carrier protein for factor VIII and as a mediator of platelet adhesion to the endothelium. Absence of vWF leads to impaired platelet function and coagulation pathway abnormalities. vWD is inherited in an autosomal dominant fashion with variable penetrance and is the most common heritable bleeding disorder option A= DIC is a consumptive coagulopathy most commonly seen in septic shock; PT, PTT, bleeding time are prolonged, D-dimer is elevated option B= inherited abnormalities in the fibrinogen molecule that can cause excessive bleeding or thrombophilia. TT, PT and PTT are abnormal in this condition, but bleeding time is unaffected option C= factor XIII is a transglutaminase that cross-links fibrin polymers, thereby stabilizing clots. Factor XIII deficiency causes spontaneous or excessive bleeding, but it would not prolong bleeding time, PT or PTT option D= X-linked deficiency of factor VIII that causes coagulopathy with prolonged PTT but shows normal bleeding time option E= X-linked deficiency of factor IX that causes coagulopathy with prolonged PTT but shows normal bleeding time option F= required for activation of factors II, VII, IX and X. it causes coagulopathy that prolongs PT mainly, and PTT prolongation in severe cases; bleeding time is not affected

Question 1 of 2 24 years woman -Chronic fatigue; history of heavy menstrual periods since menarche, and frequent nosebleeds as a child -Labs: Hb (9.2g/dl), MCV (72), decreased levels of ferritin; diagnosis? a)Antiphospholipid syndrome b)Factor VIII deficiency c)Factor XIII deficiency d)Immune thrombocytopenia e)Protein C deficiency f)Von Willebrand factor deficiency

answer: Von Willebrand factor deficiency -Microcytic anemia with low ferritin= iron deficiency anemia; In developed countries, IDA is most commonly caused by overt or occult blood loss. The patient's history of heavy menstrual periods (menorrhagia) since menarche and frequent nosebleeds (epistaxis) as ac child further suggests von Willebrand disease (vWD) as the most likely etiology of her anemia -In vWD, a mutation results in impaired synthesis in vWF, leading to a qualitative defect in platelet binding and aggregation. As a result, patients with vWD experience easy bleeding from skin and mucosal sites, including gingivae, nasal mucosa, GI tract, and endometrium option A= causes hypercoagulable state that increases risk of thromboses and spontaneous abortion option B= Hemophilia A; severe forms of vWD can also result in factor VIII deficiency as factor VIII is protected by degradation by vWF. However, this patient does not have the characteristic deep-tissue bleeding (e.g., bleeding into joints/muscles, GI bleeding, hematuria) typically seen with clotting factor deficiencies option C= extremely rare; autosomal recessive; affected patients have delayed, recurrent bleeding after trauma or surgery. Hemophilia-like bleeding (e.g., deep tissue bleeding) also seen option D= produces isolated thrombocytopenia with episodic bleeding that is typically mucocutaneous (e.g., petechiae, epistaxis, purpura); diagnosis of exclusion option E= anticoagulant protein; deficiency leads to hypercoagulable state, most commonly manifests with recurrent DVT

72 years man -Brought to hospital by paramedics after found disoriented and wandering in the street; past medical history= dementia, hypertension, stroke -Exam: disoriented, thin-appearing male; gingivae are swollen and bleed easily; ecchymoses on legs -Labs: Hb (9.6g/dl), platelets (152,000/mm3), WBC (4,500/mm3), BUN (28mg/dl), serum creatinine (0.8mg/dl), prothrombin time (11 secs), partial thromboplastin time (30 secs) -Cause of ecchymoses? a)Platelet dysfunction b)Endothelial cell dysfunction c)Hepatic dysfunction d)Water soluble vitamin deficiency e)Fat soluble vitamin deficiency f)Zinc deficiency

answer: Water soluble vitamin deficiency -Vitamin C (ascorbic acid) is a water-soluble vitamin that functions as a cofactor in numerous biochemical processes, including collagen synthesis; deficiency in vitamin C ("scurvy") is most commonly due to insufficient intake, which tends to occur in setting of chronic drug or alcohol abuse, malnourishment, or old age -Cutaneous signs= phrynoderma (perifollicular hyperkeratosis), coiled hair, poor wound healing -Mucosal findings= bleeding gums and loose teeth; hemorrhagic complications include ecchymoses and petechiae, hemarthrosis, intramuscular bleeding, and bleeding within the CNS, GI tract, genitourinary system -Normocytic, normochromic anemia is typical while coagulation studies (including PT and PTT) are normal; treatment with vitamin C supplementation, and symptomatic improvement is expected within a few days option A= common cause of platelet dysfunction in elderly include uremia, and NSAID use; this patient has mild dehydration and prerenal azotemia, which should not cause platelet dysfunction; similarly, while this patient might be taking aspirin because of his stroke history, aspirin alone would not cause bleeding gums option B= can cause vWD; presents earlier in life and is associated with prolonged PTT when severe option C= can cause prolongation of both PT and PTT or prolongation of PT and normal PTT option E= deficiency in vitamin K results in prolonged PT option F= a)deficiency in vitamin K results in prolonged PT) Zinc deficiency (characterized by an erosive dermatitis of the hand and feet and the perioral, perianal and periocular regions; other findings include: diarrhea, conjunctivitis, and impaired wound healing

28 years African American man -Prophylactic medications and vaccinations before a business trip to Africa. Five-days later, he comes to the ED due to jaundice and dark urine -Labs: Hb (8.2g/dl), reticulocytes (8%), total bilirubin (high, normal direct); LDH (high), low haptoglobin -Peripheral smear: RBCs with dark inclusions stained with supravital stain. Inheritance pattern? a)Autosomal dominant inheritance b)Autosomal recessive inheritance c)Mitochondrial inheritance d)X-linked dominant inheritance e)X-linked recessive inheritance

answer: X-linked recessive inheritance -Classic history of hemolytic anemia due to medication-induced oxidative stress -Antimalarial drugs usually prescribed for travelers to affected regions (e.g., certain parts of Africa) are notorious instigators of hemolysis in individuals with G6PD deficiency -Lab studies reflect intravascular and extravascular hemolysis with anemia, reticulocytosis, indirect hyperbilirubinemia, low haptoglobin (due to removal of free Hb-haptoglobin complexes); peripheral smear= RBCs with Heinz bodies, dark, intracellular inclusions that stain with supravital stains and represent Hb that has denatured/precipitated from oxidative stress. Heinz bodies are strongly associated with G6PD deficiency and other unstable hemoglobinopathies -G6PD= X-linked recessive (males are affected and females are generally carriers)

23 years man -Rapidly enlarging left jaw mass (developed over past weeks); political refugee from East Africa -Exam: large left-sided tumor on his jaw with surrounding lymphadenopathy but no erythema or warmth (everything else normal; HIV negative) -Biopsy= numerous mitotic figure and apoptotic bodies -Genetic feature in this tissue? a)BCL2 overexpression b)BCR-ABL rearrangement c)c-Myc oncogene overexpression d)n-Myc oncogene overexpression e)Tyrosine kinase activation

answer: c-Myc oncogene overexpression -Typical presentation of endemic (African-type) Burkitt lymphoma, of which jaw involvement is a characteristic feature; almost all cases of endemic Burkitt lymphoma are associated with EBV infection -Histology= "starry sky" appearance due to presence of macrophages and apoptotic bodies in a sea of medium-sized lymphocytes -Up to 90% of Burkitt lymphoma cases are associated with translocation of the c-Myc gene on chromosome 8, usually onto the Ig heavy chain region of chromosome 14; t(8;14) -This translocation leads to overexpression of the c-Myc oncogene and tumor growth; it is a high-grade lymphoma (very aggressive, but responds well to short-term, intensive high-dose chemotherapy)= excellent prognosis option A= follicular lymphoma; overexpression of antiapoptotic BCL2; t(14;18) translocation; causes generalized lymphadenopathy and tends to affect the elderly; jaw involvement is unusual option B and E= CML and some cases of ALL; translocation increases tyrosine kinase activity option D= seen in neuroblastoma

24 years man -Progressive generalized weakness (2 weeks); significant bruising on trunk (spontaneous, without trauma); high pulse (sinus tachycardia) -Conjunctival pallor, truncal ecchymoses; labs: Hb (6.8g/dl) -Bone marrow aspiration= grossly pale, histologically diluted due to high lipid content; labs? a)Erythropoietin (high); reticulocytes (high); MCV (normal); haptoglobin (low) b)Erythropoietin (high); reticulocytes (low); MCV (normal); haptoglobin (normal) c)Erythropoietin (high); reticulocytes (low); MCV (low); haptoglobin (high) d)Erythropoietin (low); reticulocytes (high); MCV (low); haptoglobin (low) e)Erythropoietin (low); reticulocytes (low); MCV (normal); haptoglobin (normal)

answer: choice B (Erythropoietin (high); reticulocytes (low); MCV (normal); haptoglobin (normal)) -Patient has low Hb level and has several symptoms of anemia (generalized weakness, tachycardia, and conjunctival pallor), and thrombocytopenia (spontaneous bruising without trauma). Bone marrow aspirate= abundance of lipids instead of cells, which is usually seen with bone marrow aplasia or hypoplasia= all suggest aplastic anemia -AA= caused by cytotoxic T cell destruction of multipotent hematologic stem cells. Because multipotent stem cells produce all mature blood cells, patients usually develop pancytopenia (not just anemia) and have manifestations of anemia, thrombocytopenia, and leukopenia -As with most forms of anemia, anemia-induced tissue hypoxia stimulates interstitial cells in the kidney to increase the release of erythropoietin. However, in AA, erythropoietin is unable to stimulate significant new red blood cell production due to the reduced population of functioning hematologic stem cells; therefore, reticulocytes are inappropriately low. Because the blood cells produced by the remaining undamaged stem cells are normal in morphology, erythrocytes are normal in size and appearance; therefore, MCV is normal. Because there is no intravascular hemolysis, haptoglobin (binds free hemoglobin in the blood) is also normal

20 years man -Increasing tiredness, exertional dyspnea (2 days). Yellowish discoloration of the eyes and dark urine; 5 days ago he went to urgent care due to right leg cellulitis (7 days course of Trimethoprim-sulfamethoxazole was prescribed) -Patient is an exchange student from Israel. High pulse; exam: mild scleral icterus, pale oral mucous membranes, faint systolic ejection murmur, nontender abdomen with no organomegaly -Hb (7.8g/dl), MCV (90); additional findings? a)Reticulocyte count (decreased), LDH (decreased), haptoglobin (normal) b)Reticulocyte count (decreased), LDH (normal), haptoglobin (normal) c)Reticulocyte count (increased), LDH (increased), haptoglobin (decreased) d)Reticulocyte count (increased), LDH (normal), haptoglobin (normal) e)Reticulocyte count (normal), LDH (increased), haptoglobin (decreased)

answer: choice C (Reticulocyte count (increased), LDH (increased), haptoglobin (decreased)) Anemia, signs of hemolysis (e.g., dark urine, scleral icterus) after taking a sulfa drug (i.e., TMP-SMX), findings consistent with G6PD deficiency. G6PD is a RBC enzyme that helps neutralize ROS by producing NADPH. In the absence of G6PD, oxygen radicals accumulate during times of oxidative stress (e.g., infection, sulfa drugs, fava bean-ingestion), causing damage to RBCs and intravascular hemolysis Characteristic findings of hemolysis include: -1) normocytic anemia (i.e., dyspnea, pallor, fatigue, flow murmur-systolic ejection murmur due to hyperdynamic circulation) 2) increased production of reticulocytes (immature RBCs) 3) increased breakdown products of RBCs, including LDH and Hb (Increased urinary excretion of Hb occurs with intravascular hemolysis and causes dark urine) 4) decreased haptoglobin because it binds free circulating Hb 5) degradation of Hb (heme à biliverdin à bilirubin), resulting in elevated indirect bilirubin, which manifests as jaundice and scleral icterus G6PD deficiency= Heinz bodies (inclusions of denatured Hb) and bite cells (phagocytosis of Heinz bodies) on peripheral smear

18 years woman -Collapsed while practicing for a marathon in hot, humid weather. Patient subsequently vomited 3 times and had a tonic-clonic seizure -Fever (>40C), low BP, high pulse and RR; patient is profusely sweating and confused. Pupils are sluggishly reactive to light -Labs: Hb (15.8g/dl), platelets (90,000), leukocytes (10,000); PT (high); PTT (high); findings? a)Fibrinolysis (decreased); thrombin production (decreased); plasmin generation (decreased) b)Fibrinolysis (decreased); thrombin production (decreased); plasmin generation (increased) c)Fibrinolysis (decreased); thrombin production (increased); plasmin generation (increased) d)Fibrinolysis (decreased); thrombin production (increased); plasmin generation (decreased) e)Fibrinolysis (increased); thrombin production (increased); plasmin generation (increased) f)Fibrinolysis (increased); thrombin production (increased); plasmin generation (decreased)

answer: choice E (Fibrinolysis (increased); thrombin production (increased); plasmin generation (increased)) -Extreme hyperthermia and seizures has exertional heat stroke, a life-threatening multisystem disorder caused by inadequate body heat dissipation. Severe hyperthermia increases tissue oxygen demand and metabolic rate and shunts blood away from the central organs (e.g., brain, kidneys, liver, GIT) to the skin to dissipate heat. This can lead to tissue ischemia/necrosis and the release of procoagulant proteins (e.g., tissue factor), which can trigger disseminated intravascular coagulation (DIC) -DIC is marked by excessive activation of extrinsic (tissue factor) coagulation cascade, leading to the generation of thrombin and cross-linked fibrin clots. Fibrinolysis is then activated to clear the intravascular thrombi; plasminogen is then converted to plasmin, which cuts the cross-linked fibrin-fibers and generates fibrin-degradation products (e.g., elevated D-dimer). Other anticoagulant proteins (e.g., protein C, protein S) are also rapidly consumed. -Patients with acute DIC usually have bleeding (e.g., oozing from venipuncture sites) and end organ damage (e.g., confusion, lung injury, renal insufficiency); labs are diagnostic and show thrombocytopenia (due to consumption of platelets) and prolonged PT/PTT (due to consumption of coagulation factors)

6 years girl -Abdominal pain, vomiting. Patient has Down syndrome but has been otherwise healthy. Exam: liver edge 3 cm below costal margin and spleen tip extending past the umbilicus -Leukocyte count= 104,000/mm3, and subsequent peripheral blood smear= numerous lymphoblasts -Diagnosed with acute lymphoblastic leukemia and combination cytotoxic chemotherapy is planned; lab changes following treatment initiation? a)Potassium (decreased); uric acid (increased); phosphorus (increased); LDH (increased) b)Potassium (decreased); uric acid (increased); phosphorus (decreased); LDH (increased) c)Potassium (increased); uric acid (decreased); phosphorus (decreased); LDH (increased) d)Potassium (increased); uric acid (increased); phosphorus (decreased); LDH (decreased) e)Potassium (increased); uric acid (decreased); phosphorus (increased); LDH (decreased) f)Potassium (increased); uric acid (increased); phosphorus (increased); LDH (increased)

answer: choice F (Potassium (increased); uric acid (increased); phosphorus (increased); LDH (increased)) ALL is generally associated with a high tumor cell burden and rapid turnover rate. Therefore, initiation of cytotoxic chemotherapy often leads to massive lysis of tumor cells, which spills intracellular contents into the blood (tumor lysis syndrome), resulting in the following: -hyperphosphatemia: intracellular phosphate concentration is much higher than serum phosphate levels. Therefore, serum phosphate levels typically spike following chemotherapy initiation. Because excess phosphate is excreted via the kidneys, patients are at risk for calcium-phosphate deposition in the renal tubules and collecting system, which can lead to hypocalcemia and acute renal failure -Hyperuricemia: purine nucleic acids released from lysed tumor cells are metabolized by xanthine oxidase into uric acid. Because uric acid is excreted through the kidneys and is poorly soluble, patients are at risk for obstructing uric acid stones, which can lead to acute renal injury -Hyperkalemia: potassium is primarily an intracellular ion. Therefore, significant tumor lysis is generally associated with elevations of serum potassium, which can lead to cardiac arrhythmia -Elevated lactate dehydrogenase (LDH): LDH, an enzyme involved in cellular respiration, is found in almost all human cells. It is an important lab marker for cell lysis in a wide range of disease states (e.g., hemolytic anemia). Levels are typically significantly elevated in TLS

23 years Italian American man -Mild anemia (routine check-up 3 months ago, started on iron supplementation) -Has been feeling well, energy levels are normal, he has run several half-marathons in the last 2 years -Patient's initial Hb (12.3g/dl); labs now: Hb (12.2g/dl), RBCs (5.8mill/mm3), MCV (70), WBCs (6500), platelets (180,000), HbA2 (6%, normal= 1-3%); smear= hypochromic microcytic RBCs, and target cells -Initial step in pathogenesis of patient's disorder? a)Cell membrane stability b)DNA replication c)Heme synthesis d)Intracellular oxidant detoxification e)mRNA formation f)Protein folding g)Protein solubility

answer: mRNA formation -Patient has a microcytic (MCV low) anemia and increased HbA2 (2 alpha, 2 delta) concentration. Smear shows hypochromia and Poikilocytes (including spherocytes and target cells)= asymptomatic adult of Mediterranean heritage suggest beta-thalassemia minor. -Beta-thalassemia is the most common thalassemia in patients of Mediterranean descent, alpha-thalassemia is more common in individuals from Southeast Asia. The microcytic anemia is often incorrectly attributed to iron deficiency, but iron supplements do not benefit these patients as they are not iron deficient -Normal adult hemoglobin (HbA) contains 2 alpha and 2 beta chains in combination with heme. Beta-thalassemia affects beta-chain production and can be caused by a variety of DNA mutations affecting the transcription, processing, and translation of beta-globin mRNA. Most commonly, these mutations cause aberrant precursor mRNA splicing or premature chain termination during mRNA translation. In some cases, point mutations prevent RNA polymerase from binding to the promoter region. The resulting beta-globin deficiency occurs in the setting of normal heme and alpha-chain synthesis, leading to increased formation of HbA2 and (in some patients) hemoglobin F (2 alpha and 2 gamma chains) option A and G= decreased beta-chain synthesis in beta-thalassemia results in the formation of insoluble alpha-chain tetramers that precipitate within RBCs. This can affect the membrane cytoskeleton and reduce membrane stability. However, the initial pathogenic abnormality is deficiency in beta-chain synthesis option B= in beta-thalassemia, DNA replicates normally despite mutations in the beta-globin gene. It is not until the DNA is transcribed into mRNA that Hb production becomes abnormal. Anemias due to DNA synthesis defects= megaloblastic anemia option C= Porphyrias option D= G6PD deficiency option F= Z mutation in the gene coding for alpha-1 antitrypsin; and cystic fibrosis chloride channel

54 years woman -3 months of progressive pain and prolonged morning stiffness in her hand joints. She has tried over the counter naproxen without relief -Swelling and tenderness of proximal interphalangeal joints and metacarpophalangeal joints is present in both hands -Labs: Hb (9.8g/dl), MCV (86); patient prescribed methotrexate; Labs 6 months later show Hb (12g/dl); changes before starting methotrexate therapy? a)Circulating iron level (low); bone marrow iron (high) b)Circulating iron level (low); bone marrow iron (low) c)Circulating iron level (high); bone marrow iron (low) d)Circulating iron level (high); bone marrow iron (high) e)Circulating iron level (normal); bone marrow iron (low) f)Circulating iron level (high); bone marrow iron (normal)

answer: option A (Circulating iron level (low); bone marrow iron (high)) -Polyarticular hand joint pain, swelling, prolonged morning stiffness= rheumatoid arthritis -Patients with chronic rheumatological diseases= high circulating inflammatory cytokines= triggers anemia of chronic disease (ACD) due to maladaptive changes in iron utilization -ACD is primarily mediated by hepcidin= binds & inactivates iron channels (ferroportin) on enterocytes and reticuloendothelial macrophages= reduced iron absorption and reduced iron recycling by reticuloendothelial system= reduce circulating iron levels= impairs heme synthesis -Bone marrow would show increased sequestration of iron within macrophages and reduced quantities of RBC precursor cells (e.g., normoblasts, reticulocytes) -Normocytic (or slightly microcytic) anemia with low reticulocyte count; choice B is iron deficiency anemia

25 years woman -Brought to ED after motor vehicle collision -Pneumothorax, multiple rib fractures, right femur and tibial fracture; chest tube is placed -Receives resuscitation and undergoes repair of femur and tibial fractures; following day, develops respiratory distress, decreased urine output, persistent bleeding from chest tube drains and around IV lines; labs? a)Fibrinogen (decreased), protein C (decreased), factor VII (decreased) b)Fibrinogen (decreased), protein C (decreased), factor VII (increased) c)Fibrinogen (decreased), protein C (increased), factor VII (increased) d)Fibrinogen (decreased), protein C (increased), factor VII (decreased) e)Fibrinogen (increased), protein C (increased), factor VII (increased)

answer: option A (Fibrinogen (decreased), protein C (decreased), factor VII (decreased)) -Multiple injuries following severe MVA developed respiratory distress, reduced renal output, and oozing from catheter and venipuncture sites= DIC, a consumptive coagulopathy associated with trauma, sepsis, malignancy, obstetrical complications -In DIC, due to trauma, the coagulation cascade is triggered by damage to the vascular endothelium (exposes tissue factor) and tissues (releases procoagulant proteins and phospholipids). This leads to generation of fibrin and platelet rich microvascular thrombi, which consumes platelets, coagulations factors (e.g., low factor VII) and fibrinogen. Fibrinolysis and anticoagulant proteins then dissolve clots, which depletes protein C, protein S, and antithrombin -Most patients with acute DIC develop bleeding complications such as oozing from catheters/venipuncture sites, ecchymosis, and petechiae. End-organ damage to the kidneys (e.g., renal insufficiency) and lungs (e.g., pulmonary hemorrhage) is common. Many patients also have microangiopathic hemolytic anemia (e.g., schistocytes) due to shearing of RBCs by intravascular thrombi

40 years man -End-stage renal disease due to type 1 diabetes is hospitalized for initiation of hemodialysis -Tunneled dialysis catheter is inserted into the right internal jugular vein; before he is able to undergo dialysis treatment, patient develops bleeding around the catheter exit site that is difficult to control (hasn't been treated with anticoagulants) -Lab abnormalities? a)PT (normal); aPTT (normal); platelet count (normal); bleeding time (prolonged) b)PT (normal); aPTT (prolonged); platelet count (normal); bleeding time (prolonged) c)PT (normal); aPTT (prolonged); platelet count (normal); bleeding time (normal) d)PT (prolonged); aPTT (prolonged); platelet count (decreased); bleeding time (prolonged) e) PT (prolonged); aPTT (prolonged); platelet count (normal); bleeding time (normal)

answer: option A (PT (normal); aPTT (normal); platelet count (normal); bleeding time (prolonged)) -Excessive bleeding is common in patients with significant renal dysfunction due in part to accumulation of uremic toxins in the circulation -These toxins impair platelet aggregation and adhesion, resulting in a qualitative platelet disorder characterized by prolonged bleeding time with normal platelet count, PT and aPTT -Uremic bleeding can be improved with dialysis as it removes the toxins and partially reverses the bleeding abnormality -Choice B= severe vWD with VIII deficiency; choice C= hemophilia A and heparin use; choice D= DIC; choice E= warfarin therapy

16 years girl -Fatigue (past months); plays on soccer team but her endurance has decreased; sleeps 9 hours a night, vegetarian (past year). -Reached menarche at age 13, regular menses (past 6 months). Exam: well-nourished teenage girl with pale conjunctivae. Hb (9.2g/dl). Labs? a)Serum ferritin (normal); circulating transferrin (normal); MCV (74); hypersegmented neutrophils (none); serum folate (normal) b)Serum ferritin (low); circulating transferrin (high); MCV (76); hypersegmented neutrophils (none); serum folate (normal) c)Serum ferritin (low); circulating transferrin (low); MCV (84); hypersegmented neutrophils (none); serum folate (normal) d)Serum ferritin (high); circulating transferrin (low); MCV (95); hypersegmented neutrophils (none); serum folate (normal) e)Serum ferritin (normal); circulating transferrin (normal); MCV (108); hypersegmented neutrophils (present); serum folate (normal) f)Serum ferritin (normal); circulating transferrin (normal); MCV (115); hypersegmented neutrophils (present); serum folate (low)

answer: option B (Serum ferritin (low); circulating transferrin (high); MCV (76); hypersegmented neutrophils (none); serum folate (normal)) Conjunctival pallor= anemia; diagnosis confirmed by low Hb. Women of childbearing age are at risk of iron deficiency anemia due to menstrual cycle blood loss, especially teenage girls who have higher iron requirements due to growth. In this patient, decreased consumption of dietary iron (e.g., vegetarianism) is an additional risk factor. As iron deficiency develops, the following sequence can be seen with lab testing: -Decreased bone marrow iron stores (ferritin and hemosiderin) -Decreased serum ferritin -Increased serum total iron-binding capacity, reflecting increased transferrin -Decreased serum iron concentration; decreased Hb -Appearance of microcytic, hypochromic RBCs (low MCV/MCH) Ferritin is an intracellular iron-storage protein that is used as a serum marker of total body iron stores. It is decreased in iron deficiency and elevated in iron overload or during infection/inflammation (acute phase reaction). Transferrin transports iron through the plasma. When iron levels are normal, approximately 1/3 of circulating transferrin is bound to iron. In iron deficiency, hepatic synthesis of transferrin increases but transferrin saturation drops due to decreased release of iron into plasma from intracellular stores. Option A= alpha/beta thalassemia; option D= anemia of chronic disease; option E and F= megaloblastic anemia

35 years man -Brought to ED after collapsing in his garage. He collapsed during reparation of his car. Patient was inside the garage but had the door half-open (lost consciousness after working on car for 2 hours, while engine was running). Combination of findings on arterial blood sample? a)Carboxyhemoglobin, normal= 0-2% (2); PaO2, mmHg, normal= 85-100 (95); Methemoglobin, normal= 0-1% (30) b)Carboxyhemoglobin, normal= 0-2% (35); PaO2, mmHg, normal= 85-100 (45); Methemoglobin, normal= 0-1% (1) c)Carboxyhemoglobin, normal= 0-2% (35); PaO2, mmHg, normal= 85-100 (45); Methemoglobin, normal= 0-1% (25) d)Carboxyhemoglobin, normal= 0-2% (40); PaO2, mmHg, normal= 85-100 (95); Methemoglobin, normal= 0-1% (1) e)Carboxyhemoglobin, normal= 0-2% (40); PaO2, mmHg, normal= 85-100 (95); Methemoglobin, normal= 0-1% (25)

answer: option D= Carboxyhemoglobin, normal= 0-2% (40); PaO2, mmHg, normal= 85-100 (95); Methemoglobin, normal= 0-1% (1) Carbon monoxide (CO) is a colorless, odorless gas product of the combustion of carbon-containing compounds. CO poisoning can occur after exposure to automobile exhaust (especially in older cars without catalytic converters), fire smoke, or improperly vented natural gas appliances. CO has 2 major effects on oxygen delivery to tissues: 1)CO binds to Hb with an affinity approximately 250 times that of oxygen, reducing the number of heme binding sites available to oxygen. As a result, levels of CO-bound Hb, represented by carboxyhemoglobin (HBCO), increase. By decreasing the fraction of Hb available for oxygen binding, CO decreases the oxygen-carrying capacity and oxygen content of blood, but not the amount of oxygen dissolved in plasma, reflected by the partial pressure of oxygen (PaO2). 2)CO poisoning causes a leftward shift of the Hb-oxygen dissociation curve, reflecting a decreased tendency for oxygen to unload in tissues CO's effects on arterial blood oxygen content are seen in the graphs, which show lower oxygen content with CO poisoning and leftward shift of the dissociation curve that becomes increasingly more pronounced at increasingly higher HBCO concentrations

42 years man -Fever, persistent sore throat; exam: fever, several bruises on trunk, blood oozing from his IV catheter venipuncture sites -Blood fibrinogen is reduced; bone marrow biopsy= predominance of immature myeloid cells with azurophilic needle-shaped cytoplasmic granules. Chromosomal analysis of these immature cells? a)t(8;14) b)t(9;22) c)t(14;18) d)t(15;17) e)inv(16)

answer: t(15;17) -Fever (suggesting infection and possibly low WBC count), and bruising/oozing from venipuncture sites (suggesting low platelet count) associated with low fibrinogen levels and immature myeloid cells= likely has DIC in setting of acute promyelocytic leukemia (APL); APL, a subtype of AML is characterized by presence of bone marrow of promyelocytes (immature myeloid cells) that typically contain Auer rods (fused lysosomal granules, described in question) -Classic presentation is bleeding in setting of DIC, a consumptive coagulopathy characterized by decreased fibrinogen levels, increased fibrin degradation products, prolonged PT and PTT, and thrombocytopenia -APL is most commonly associated with a t(15;17) chromosomal translocation that causes fusion of the retinoid acid receptor-alpha (RARA) gene to the promyelocyte leukemia (PML) gene. APL is referred to as AML M3 in the FAB classification of leukemia and is responsive to all-trans retinoid acid therapy option A= 80% of cases of Burkitt lymphoma; c-myc overexpression= increases transcription option B= CML; Philadelphia chromosome; BCR-ABL fusion= constitutively active tyrosine kinase option C= follicular lymphoma; BCL2 overexpression= inhibits apoptosis option E= observed in M4Eo (eosinophilic) subtype of AML

43 years man -Recurrent episodes of fever, sore throat (despite antibiotic therapy) -Fatigue, bruises easily, bleeding gums -Exam: fever, mucosal pallor, pharyngeal erythema, multiple ecchymoses on his extremities -Peripheral blood smear (coarse rod-shaped eosinophilic intracytoplasmic granules); chromosomal abnormality that is likely present in affected cells? a)t(8;14) b)t(9;22) c)t(11;14) d)t(15;17) e)13q-

answer: t(15;17) -Recurrent infections (possibly reflecting neutropenia), pallor (anemia), and ecchymoses (thrombocytopenia) + PBS= several abnormal myelocyte precursors containing coarse rod-shaped intracytoplasmic granules called Auer rods -Characteristic for acute myelogenous leukemia; The M3 variant of AML, acute promyelocytic leukemia (APML), characterized by presence of promyelocytes on smear, associated with DIC (bleeding, thrombocytopenia, prolonged PT and aPTT) -Affected cells exhibit t(15;17). Translocation between retinoic acid receptor (RAR-alpha) gene on chromosome 17 and promyelocytic leukemia (PML) gene on chromosome 15 -Fusion of these 2 genes produces a chimeric gene product, PML/RAR-alpha, which codes for an abnormal retinoic acid receptor= triggers development of APML; management with all-trans retinoic acid option A= associated with Burkitt lymphoma, translocation between c-Myc protooncogene on chromosome 8 and Ig heavy chain region on chromosome 14; this leads to increased production of the oncogene due to frequency with which the Ig gene is transcribed option B= BCR-ABL fusion= protein has tyrosine kinase activity= characteristic of CML option C= mantle cell lymphoma; B-cell malignancy; translocation results in activation of cyclin D gene option E= one of molecular defects seen in CLL


Conjuntos de estudio relacionados

Principles Quiz 3 (Ch. 20,21,43,46,31,36)

View Set

Testing and Remediation Advanced Test

View Set

HESI practice notes - nursings sciences

View Set

Psychology of workforce diversity UWF Exam 1

View Set

FNAN 320: Chapter 11 - The economics of financial intermediation

View Set

Government Regulation of The Economy

View Set

Chapter 25 & 26 (Vision and Hearing and Endocrine Function)

View Set